UWorld Review 3

Lakukan tugas rumah & ujian kamu dengan baik sekarang menggunakan Quizwiz!

A 46-year-old HIV-positive man is hospitalized with fever, cramping abdominal pain, and watery diarrhea. Evaluation shows high fever, hypotension, tachycardia, and lower abdominal distension and tenderness. Abdominal x-ray reveals free intraperitoneal air, and the patient is taken for urgent exploratory laparotomy. Operative findings include an erythematous and dilated colon. A focus of bowel wall necrosis with perforation is resected. Histopathologic analysis reveals acute inflammatory changes, epithelial necrosis, and a layer of denuded epithelium, fibrin, and inflammatory cells overlaying the mucosa. Which of the following pathogens is most likely responsible for this patient's current condition?

Clostridioides difficile - This patient with diarrhea, abdominal pain, and fever developed a colonic perforation. Given his histologic findings consistent with pseudomembrane formation, this presentation is highly suggestive of Clostridioides difficile infection (CDI) (formerly Clostridium genus). C difficile is an anaerobic, spore-forming, toxigenic, gram-positive rod that causes infectious colitis. The most significant risk factor for the development of CDI is antibiotic use. Antibiotics (particularly clindamycin, fluoroquinolones, and cephalosporins) alter the normal bowel flora, thereby predisposing to C difficile overgrowth. Other risk factors include hospitalization, older age, and proton pump inhibitor use. C difficile produces 2 toxins: Toxin A (enterotoxin) and Toxin B (cytotoxin). These toxins disrupt cellular cytoskeletons and intercellular tight junctions, leading to colonocyte apoptosis. The colonic mucosa responds to toxin exposure by forming yellow-white, patchy pseudomembranes, which consist of a neutrophil-predominant inflammatory infiltrate, fibrin, bacteria, and necrotic epithelium. Patients with severe disease may develop a nonobstructive colonic dilation known as toxic megacolon, which leads to increased risk of colonic perforation (as seen in this patient).

A 55-year-old man comes to the office due to intermittent shortness of breath, cough, and chest tightness over the past 3 months. The patient had asthma as a child that improved in adolescence, and he has not required inhalers or steroids since. He feels that his current symptoms are similar to his prior asthma symptoms. The patient was recently diagnosed with essential tremor, which has improved with medical therapy; he has no other chronic medical conditions. He is a lifetime nonsmoker and does not use alcohol or illicit drugs. Blood pressure is 125/75 mm Hg, pulse is 58/min, and respirations are 16/min. Physical examination reveals bilateral scattered expiratory wheezes. Heart sounds are normal and there is no extremity edema. This patient's symptom recurrence is most likely attributable to a medication that affects which of the following steps?

Binding of norepinephine to the postsynaptic cell - This patient has developed symptoms of intermittent asthma (eg, cough, wheeze, dyspnea) associated with a new medication for his essential tremor (ET). ET is a common movement disorder, and beta blockers are often used as first-line therapy for debilitating or distressful symptoms. Beta blockers prevent the interaction of epinephrine and norepinephrine with receptors at adrenergic synapses; in ET, this is thought to help dampen the sensitivity of muscle spindles, which improves tremor symptoms by smoothing out the force of contraction. Propranolol, the most commonly used medication for treatment of ET, is a nonselective beta blocker (ie, inhibits both beta-1 and beta-2 receptors). In patients with reactive airway diseases (eg, asthma, chronic obstructive pulmonary disease), inhibition of beta-2 receptors can cause bronchoconstriction with resultant asthma symptoms. Selective beta-1 blockers are often better tolerated in patients with asthma; however, selectivity is dose-dependent and patients can develop bronchoconstriction at high doses.

As part of an experiment, radiolabeled ATP is injected into skeletal muscle. During muscle contraction, the labeled ATP is observed to attach to the sarcomere. This attachment causes immediate:

Myosin head detachment from the actin filament - According to the model posited by Rayment et al., the role of ATP in skeletal and cardiac muscle contraction may be to release the myosin head from its actin binding site and then to energize a conformational change that resets the myosin head to "contract" again the next time it binds to actin.

A 45-year-old previously healthy man comes to the office for evaluation of a neck lump. The patient first noticed the swelling a month ago while shaving, and it has progressively enlarged. He has had no fever, neck pain, hoarseness, or difficulty breathing or swallowing. The patient does not use tobacco, alcohol, or illicit drugs. Temperature is 36.7 C (98 F), blood pressure is 110/70 mm Hg, and pulse is 78/min. On physical examination, there is a nontender, firm nodule in the upper portion of the left thyroid lobe. Laboratory studies reveal a normal serum TSH and elevated serum calcitonin levels. Neck ultrasonography shows a 2-cm hypoechoic thyroid nodule with microcalcifications. Fine-needle aspiration of the nodule reveals spindle-shaped cells in an amorphous background. Which of the following gene alterations most likely contributed to this patient's current condition?

Activation of RET - This patient has medullary thyroid cancer (MTC) presenting with a palpable nodule and elevated serum calcitonin. MTC is a neuroendocrine tumor that arises from calcitonin-secreting C (parafollicular) cells. Microscopy shows nests or sheets of polygonal or spindle-shaped cells, often with extracellular amyloid deposition (consisting of full-length calcitonin). MTC is often seen in the context of multiple endocrine neoplasia type 2 (A and B), although the majority of cases are sporadic. Paraneoplastic symptoms (eg, diarrhea, flushing) can occur due to elevated calcitonin levels; however, calcium is usually normal, possibly due to the downregulation of calcitonin receptors. MTC is associated with activating mutations in the RET proto-oncogene; mutations are commonly found in sporadic MTC, and germline mutations are present in more than 95% of patients with familial MTC. The RET proto-oncogene codes for a membrane-bound tyrosine kinase receptor involved in cell cycle regulation. In patients with MTC, constitutive activation of the receptor leads to unregulated cellular proliferation

An 18-year-old man is admitted to the hospital with cough, fever, and hypoxemia. Despite multiple intravenous antibiotics and frequent chest physiotherapy, the patient decompensates and is transferred to the intensive care unit for mechanical ventilation. Sputum cultures grow >100,000 colony-forming units of a non-lactose-fermenting gram-negative rod. After several days, the patient dies of overwhelming pneumonia complicated by respiratory failure. Since age 1, he had been hospitalized for recurrent episodes of multi-lobar pneumonia, sinusitis, and poor growth. On autopsy, both vas deferens are absent. Which of the following findings is also likely to be present on autopsy?

Distension and obstruction of pancreatic ducts - Cystic fibrosis transmembrane conductance regulator (CFTR) normally allows for transport of chloride into gastrointestinal and respiratory lumens, followed by sodium and water, leading to thin mucus. In cystic fibrosis (CF), the dysfunctional or absent CFTR gene leads to decreased water within the lumen, resulting in thick, dehydrated, and viscous mucus within organs. This patient's recurrent sinopulmonary infections and congenital absence of the vas deferens are consistent with CF. Pneumonia due to a non-lactose-fermenting, gram-negative rod (Pseudomonas aeruginosa or Burkholderia cepacia) is very common. The most common gastrointestinal manifestation of CF is pancreatic insufficiency (PI). Pancreatic duct obstruction and distension due to viscous mucus and subsequent inflammation develop in utero, eventually leading to fibrosis. PI is present from birth in most patients with CF. The loss of pancreatic tissue prevents these patients from developing pancreatitis. PI results in inability to absorb fats and fat-soluble vitamins (ADEK), leading to steatorrhea and failure to thrive. Patients with PI require pancreatic enzyme supplementation.

A 63-year-old man comes to the hospital due to sudden onset chest pain and dyspnea. He has a history of glioblastoma multiforme, for which he has been undergoing radiation treatment. His temperature is 36.1 C (97 F), blood pressure is 110/80 mm Hg, pulse is 118/min, and respirations are 26/min. Pulse oximetry shows 90% on room air. A CT pulmonary angiogram demonstrates a left-sided pulmonary embolism. The patient is started on continuous unfractionated heparin infusion. Several hours later, the patient begins to experience severe headache and quickly becomes unconscious. A CT of the head reveals bleeding into the tumor. After stopping the heparin infusion, administration of which of the following is indicated for immediate anticoagulant reversal?

Protamine - Unfractionated heparin (UFH) and low-molecular weight heparins (LMWHs) are commonly used anticoagulants in the hospital setting. Heparin increases the effect of the naturally occurring anticoagulant antithrombin III (ATIII). Activated partial thromboplastin time (aPTT) is measured to monitor the therapeutic effect of heparin. Bleeding and heparin-induced thrombocytopenia are serious complications of heparin therapy. Bleeding due to heparin toxicity is treated with protamine sulfate, a specific antagonist. Protamine sulfate is a peptide that binds to heparin, forming a complex that has no anticoagulant activity. Although protamine is often used in the management of LMWH toxicity, it does not completely reverse the anti-Xa activity of LMWH.

A 50-year-old man comes to the emergency department due to intermittent palpitations over the last 6 months. The patient describes them as "very uncomfortable" and says the episodes occur randomly, last for several hours, and resolve spontaneously or with sleep. The current episode started an hour ago and has been accompanied by lightheadedness. He has no medical history. Temperature is normal and blood pressure is 122/75 mm Hg. BMI is 30 kg/m2. Chest examination reveals no murmurs and the lungs are clear on auscultation. There is trace peripheral edema. Cardiac rhythm strip obtained in the emergency department is shown in the exhibit. Aberrant electrical activity in which of the following anatomic structures is the most likely trigger for this patient's current condition?

Pulmonary veins - This patient's presentation with intermittent palpitations and cardiac rhythm strip showing an irregularly irregular rhythm with varying R-R intervals and no discernible P waves is consistent with paroxysmal atrial fibrillation (AF). The development of AF involves structural and electrical changes that predispose to the initiation and maintenance of electrical reentrant circuits and/or ectopic foci. These pathologic changes, termed atrial remodeling, can occur as a result of normal aging or diseases that cause atrial dilation (eg, hypertension, mitral regurgitation). The remodeling leads to shortening of the atrial refractory period and heterogeneity in localized electrical conduction between atrial myocytes, creating a substrate for the development of AF. In predisposed patients, AF can spawn from specific triggers such as focal electrical discharges, increased atrial stretch, elevated sympathetic activity (eg, sepsis, emotional stress), and ingestion of alcohol. The most common trigger is aberrant electrical foci in the pulmonary veins near their ostia into the left atrium; therefore, catheter ablation of pulmonary vein trigger sites (pulmonary vein isolation) is used for the treatment of symptomatic, paroxysmal AF.

A 36-year-old woman is brought to the emergency department with sudden-onset right side weakness and speech difficulty. During the last 3 weeks, the patient has experienced progressive fatigue, malaise, and low-grade fevers. Despite the symptoms, she did not seek medical attention and did not take any medications. She had a dental extraction 5 weeks ago, which was uncomplicated. The patient has never previously been significantly ill or hospitalized. She works as a receptionist at a legal firm and has never traveled outside the United States. She does not use tobacco, alcohol, or illicit drugs. The patient is admitted to the hospital, but despite adequate resuscitative measures, she dies 2 hours later. Gross autopsy shows large, friable irregular masses attached to the atrial surface of a valve. Which of the following underlying conditions most likely predisposed this patient to developing these cardiac changes?

Regurgitant mitral valve prolapse - This patient had subacute infective endocarditis (IE) complicated by an embolic stroke. The large, friable irregular masses seen on autopsy are most likely vegetations and are the most probable source of this patient's left middle cerebral artery embolus (eg, right-sided weakness, speech difficulty). The mitral valve is the valve most frequently affected by IE. Mitral valve prolapse (MVP) is the most common underlying valvular disease predisposing to the development of IE in developed countries, particularly when it is associated with coexistent mitral regurgitation. Microscopic deposits of platelets and fibrin occur spontaneously in individuals with valvular disease secondary to endocardial injury from turbulent blood flow. These deposits then become colonized by microorganisms (eg, viridans streptococci) during episodes of bacteremia (eg, following dental extraction). In developing countries, the most common acquired heart disease leading to IE development is rheumatic heart disease (RHD). RHD is a late (10-20 years) complication of acute rheumatic fever (eg, migratory arthritis, carditis/valvulitis, erythema marginatum). RHD was historically considered a frequent predisposing factor for IE; however, MVP, valvular sclerosis, and mechanical valves are now more common causes in developed nations due to the recognition and treatment of group A streptococcal pharyngitis.

A 35-year-old pregnant woman at 8 weeks gestation comes to the emergency department due to persistent nausea and vomiting. The patient has had intermittent nausea for the past week and vomiting for the past 3 days. Now, she is unable to tolerate solids or liquids. Temperature is 36.7 C (98 F), blood pressure is 90/64 mm Hg, pulse is 108/min, and respirations are 14/min. Mucous membranes are dry and capillary refill time is delayed. Cardiac examination shows sinus tachycardia and no murmurs. The abdomen is nontender and nondistended. Compared to her baseline, which of the following sets of serum electrolyte concentration abnormalities are most likely present in this patient?

Sodium ↓ Potassium ↓ Chloride ↓ Bicarbonate ↑ This patient has had severe vomiting (suggestive of hyperemesis gravidarum) and now has multiple signs of volume depletion (eg, dry mucous membranes, delayed capillary refill time, tachycardia). Vomiting causes a significant loss of gastric H+ from the body, which leads to increased serum HCO3− (metabolic alkalosis). There is also loss of water and salt (relatively more Cl− is lost than Na+ due to high gastric quantity of HCl), leading to volume depletion that perpetuates the metabolic alkalosis and causes other electrolyte abnormalities. Intravascular volume depletion decreases renal perfusion, resulting in activation of the renin-angiotensin-aldosterone system. Aldosterone stimulates Na+ reabsorption and a lesser degree of passive Cl− reabsorption in the distal tubules of the kidneys in an effort to increase blood volume. The relatively greater loss of Cl− compared to Na+ from both the stomach and kidneys leads to a large Cl− deficit and characteristic hypochloremia. The aldosterone-mediated increase in Na+ reabsorption comes at the expense of increased K+ and H+ excretion via the principal and alpha intercalated cells in the collecting duct. This leads to hypokalemia and exacerbation of the metabolic alkalosis. Chloride depletion then perpetuates the metabolic alkalosis because low tubular Cl− concentration impairs HCO3− excretion via the pendrin pump on beta intercalated cells. If the hypovolemia persists, it also provides nonosmotic stimulus for the secretion of antidiuretic hormone (ADH). The ADH secretion is considered appropriate because the body's priority is to restore itself to euvolemia; however, it leads to free water retention with hyponatremia and more profound hypochloremia.

An unvaccinated 20-month-old girl is brought to the emergency department with a rash. Three days ago, she developed a fever along with cough, congestion, and red eyes. The rash appeared on her face yesterday and spread to her trunk, arms, and legs today. The patient's temperature is 39.4 C (103 F). She is lethargic and ill appearing. Physical examination shows conjunctival injection and a diffuse, maculopapular, erythematous rash. She is admitted to the hospital for further management. Deficiency of which of the following is associated with a high rate of complications from this patient's condition?

Vitamin A - Vitamin A supplementation is recommended, especially in patients with vitamin A deficiency. Acute measles infection depletes vitamin A stores, resulting in a risk of keratitis and corneal ulceration. Vitamin A helps prevent and treat these ocular complications. In addition, it reduces risk of other comorbidities (eg, pneumonia, encephalitis), recovery time, and length of hospital stay. No other vitamin is linked to measles.

A 9-year-old girl is brought to the emergency department due to prolonged epistaxis. The girl says that she picked her nose immediately before the bleeding started. Her parents decided to bring her to the emergency department after the epistaxis persisted for 20 minutes despite constant compression of the nasal alae. The patient has had frequent nosebleeds that often last >10 minutes. Her family history is significant for a grandfather who had an unspecified bleeding disorder. Given the history of prolonged, recurrent nosebleeds, laboratory tests are ordered, and results are as follows: Hematocrit 43% Bleeding time prolonged Partial thromboplastin time (PTT) prolonged Prothrombin time (PT) normal Thrombin time (TT) normal D-dimer normal Which of the following is the most likely diagnosis?

Von Willebrand disease - This patient has a normal prothrombin time (PT) and thrombin time (TT) and a prolonged partial thromboplastin time (PTT), indicating a defect in the intrinsic pathway (coagulation factors VIII, IX, XI, or XII). Bleeding time is a test of platelet function and is prolonged by qualitative and quantitative platelet defects. The term "bleeding time" refers to this particular test and not the duration of bleeding, which can be prolonged from other coagulopathies. von Willebrand disease (vWD) will cause both a prolonged PTT and bleeding time. von Willebrand factor (vWF) is produced by endothelial cells and megakaryocytes and functions as a carrier protein for factor VIII and as a mediator of platelet adhesion to the endothelium. Absence of vWF leads to impaired platelet function and coagulation pathway abnormalities. vWD is inherited in an autosomal dominant fashion with variable penetrance and is the most common heritable bleeding disorder.

A researcher is studying the Fas receptor (FasR), a protein widely expressed on cell surfaces. The signaling cascade of programmed cell death is initiated when FasR binds to its ligand (FasL), which is expressed on cytotoxic T cells. In an experiment, cancer cells that escaped elimination by the immune system were found to contain soluble Fas proteins that did not promote apoptosis. The soluble Fas proteins were shorter and lacked the transmembrane domain. DNA analysis of these cells revealed no FAS gene mutations. Which of the following is the most likely explanation for the formation of altered Fas proteins in these cancer cells?

Alternative splicing - Alternative splicing is a normal process that allows production of alternate sets of proteins in different tissues. It has also been implicated in various human diseases. Cancers in particular can use alternative splicing to evade innate defense mechanisms. The Fas receptor-Fas ligand interaction drives programmed cell death via the cytotoxic T-cell mediated extrinsic pathway. Cancer cells may develop the ability to splice out a particular exon that codes for the transmembrane domain of the Fas receptor (FasR), converting it to a soluble form that is not expressed on the cell surface, which allows the cells to evade apoptosis.

A 56-year-old man comes to the emergency department due to palpitations that started several hours ago. He has not had similar symptoms before. The patient has had no chest pain, shortness of breath, lightheadedness, or syncope. He drinks alcohol on weekends and does not use illicit drugs. He has no family history of heart disease. Blood pressure is 145/78 mm Hg. BMI is 40 kg/m2. ECG is shown in the exhibit. The patient is treated medically in the emergency department. He is then taken to the electrophysiology laboratory for radiofrequency catheter ablation to terminate his arrhythmia. The ablation procedure should create a conduction block through which of the following areas?

Area between the tricuspid valve and inferior vena cava in the right atrium - This patient's ECG strip shows rapid and regular atrial activity in a saw-toothed pattern (flutter or F waves), consistent with atrial flutter. Narrow QRS complexes are typically present and the ventricular rhythm appears regular. Because the atrial rate is approximately 300/min, typical ventricular rates in atrial flutter are 150/min, 100/min, or 75/min due to 2:1, 3:1, or 4:1 conduction through the atrioventricular node, respectively. Typical atrial flutter is caused by a large reentrant circuit that traverses the cavotricuspid isthmus, the region of right atrial tissue between the inferior vena cava and the tricuspid valve annulus. This region is identified during electrophysiologic study and is the target for radiofrequency ablation to interrupt the reentrant circuit and abolish atrial flutter.

A 50-year-old woman with a history of systemic lupus erythematosus is admitted with fever, chills, vomiting, and burning pain when urinating. Temperature is 39.3 C (102.7 F), blood pressure is 80/50 mm Hg, pulse is 120/min, and respirations are 20/min. On examination, costovertebral angle tenderness is present. The patient is given intravenous antibiotics. Over the next several hours, she has decreased urine output despite aggressive intravenous hydration. Blood is oozing around the central venous catheter. Laboratory results are as follows: Complete blood count Hemoglobin 9.0 g/dL Platelets 68,000/mm3 Leukocytes 24,500/mm3 Coagulation studies Prothrombin time 23 sec Partial thromboplastin time 60 sec Plasma fibrinogen 100 mg/dL (150-350) Which of the following is the most likely cause of this patient's hematologic abnormalities?

Consumptive coagulopathy - This patient's costovertebral angle tenderness, dysuria, and leukocytosis are indicative of acute pyelonephritis complicated by septic shock (hypotension, tachycardia). Her laboratory results show a consumptive coagulopathy (thrombocytopenia, prolonged PT/PTT, low fibrinogen), which, along with bleeding from the central catheter site, are likely due to disseminated intravascular coagulation (DIC). DIC is a common complication of sepsis (particularly with gram-negative organisms) because lipopolysaccharide is a procoagulant that triggers the coagulation cascade. This leads to the formation of fibrin- and platelet-rich thrombi in the vasculature, which consumes platelets (thrombocytopenia), coagulation factors (prolonged PT/PTT), and fibrinogen. Fibrinolysis is then triggered to degrade the clots, which elevates D-dimer (a fibrin degradation product) and depletes protein C, protein S, and antithrombin. The thrombi also often shear red blood cells, leading to microangiopathic hemolytic anemia (MAHA). Most patients with acute DIC have signs of bleeding, such as oozing from vascular catheters, mucocutaneous bleeding, ecchymosis, or petechiae. End organ damage (eg, renal insufficiency, pulmonary hemorrhage) is also common.

A 24-year-old woman comes to the physician with a 3-week history of progressive fatigue. Physical examination shows lesions involving the nail beds. Further evaluation would most likely reveal additional abnormalities during which part of the physical examination?

Cardiac auscultation - These lesions are subungual splinter hemorrhages, splinter- or flame-shaped hemorrhagic streaks in the nail bed that appear as a consequence of microemboli. Janeway lesions (small, macular, erythematous or hemorrhagic, nontender lesions on the palms and soles) are also a sign of microembolism. Infectious endocarditis (IE) is the usual cause of such microemboli, and progressive fatigue may be the presenting symptom of subacute bacterial endocarditis. Chronically destructive, infected vegetations involving the cardiac valves are the source of the microemboli in bacterial endocarditis. Patients with suspected IE should therefore be evaluated for a regurgitant murmur during cardiac auscultation.

A 30-year-old woman comes to the emergency department with sudden-onset abdominal pain and ascites. Laboratory studies show anemia, reticulocytosis, leukopenia, and thrombocytopenia. Flow cytometry of the patient's peripheral blood cells using the appropriate monoclonal antibodies shows CD55 and CD59 deficiency. CT scan of the abdomen shows hepatic vein thrombosis. Which of the following is the most likely cause of this patient's anemia?

Complement activation - This patient's combination of hemolytic anemia, hypercoagulability (eg, hepatic vein thrombosis), and pancytopenia are characteristic of paroxysmal nocturnal hemoglobinuria (PNH). Despite the name, hemolysis has been shown to occur at low baseline levels throughout the day, with exacerbations caused by a variety of inflammatory triggers (eg, infections, surgery). PNH is caused by an acquired mutation of the PIGA gene within a clonal population of multipotent hematopoietic stem cells. This gene is involved in the synthesis of the glycosylphosphatidylinositol (GPI) anchor, a glycolipid necessary for the attachment of several cell-surface proteins, including CD55 (decay-accelerating factor) and CD59 (MAC inhibitory protein). These proteins help inactivate complement and prevent the membrane attack complex from forming on normal cells. Absence of the GPI anchor results in CD55 and CD59 deficiency and complement-mediated hemolysis. The hemolysis occurs more often at night because complement activity is increased during sleep due to lower blood pH. Patients also develop thrombotic complications (eg, Budd-Chiari syndrome), likely due to release of free hemoglobin and other prothrombotic factors from lysed red blood cells and platelets. PNH is often associated with pancytopenia and aplastic anemia (possibly due to autoimmune attack against GPI-antigens on stem cells). Flow cytometry is the gold standard for the diagnosis of PNH and will show absence of the GPI anchor and CD55 and CD59 deficiency.

A 35-year-old primigravida is found to have gallstones at 38 weeks of an uncomplicated pregnancy. Ultrasonography performed one year ago failed to demonstrate any abnormalities. Which of the following pathogenetic components most likely contributed to this patient's condition?

Estrogen-induced cholesterol hypersecretion and progesterone-induced gallbladder hypomobility - Pregnancy and the usage of oral contraceptives predispose to gallstone formation, with 5-12% of all women developing gallstones during pregnancy. Estrogenic influence increases cholesterol synthesis by upregulating hepatic HMG-CoA reductase activity, which causes the bile to become supersaturated with cholesterol. Progesterone reduces bile acid secretion and slows gallbladder emptying. When the gallbladder is hypomotile or there is more cholesterol than bile salts, the cholesterol precipitates into insoluble crystals that eventually form to make gallstones.

A 53-year-old man with nocturnal cough undergoes an upper endoscopy with esophageal biopsy. Light microscopy of the tissue specimen shows elongation of the lamina propria papillae with several eosinophils and neutrophils scattered within the squamous epithelium. The patient takes metformin and enalapril for diabetes mellitus and hypertension, respectively. He is started on pantoprazole, and after 2 weeks his cough symptoms resolve. Which of the following was the most likely cause of his symptoms?

Gastroesophogeal junction incompetence - Gastroesophageal reflux disease (GERD) is a very common condition that usually manifests with heartburn, regurgitation, and dysphagia. However, patients can have extraesophageal symptoms (eg, nocturnal cough, hoarseness) in the absence of heartburn ("silent GERD"). Gastroesophogeal junction incompetence is the primary pathophysiologic mechanism responsible for GERD. This incompetence is most commonly caused by excessive relaxation of the lower esophageal sphincter, resulting in the reflux of acidic gastric contents back into the esophagus. Irritation of the esophageal mucosa leads to an inflammatory reaction with mucosal erythema and edema; erosions/ulcerations may develop in severe cases. Characteristic histologic findings include basal zone hyperplasia, elongation of the lamina propria papillae, and scattered eosinophils and neutrophils.

A 19-year-old college student comes to the office with new-onset fatigue. The patient has had difficulty getting up in the morning for the past week and states he would sleep 10-12 hours if his schedule allowed. He also reports vivid dreams. The patient was diagnosed with attention-deficit hyperactivity disorder at age 10 and has been treated with dextroamphetamine since then. However, his prescription ran out early recently after he doubled the dosage to help him study during exams. Which of the following findings is most likely to be present in this patient?

Hyperphagia - This patient's new-onset fatigue, hypersomnia, and vivid dreams following abrupt cessation of prolonged stimulant (eg, dextroamphetamine) use is most likely due to stimulant withdrawal. Decreased appetite and insomnia are the most common adverse effects of stimulant medications used to treat attention-deficit hyperactivity disorder, and abrupt stimulant withdrawal is associated with the opposite symptoms of fatigue, hypersomnia, and hyperphagia. Dysphoric mood may also occur.

A 22-year-old woman comes to the office because of a 6-month history of increasing neck and low back pain that is most severe in the morning. Her pain improves as the day passes. The patient has no history of trauma, rashes, or urinary symptoms. Vital signs are within normal limits. Examination shows tenderness over the lumbosacral area and at the insertion site of the Achilles tendon. Forward flexion is reduced. This patient's symptoms are most likely attributable to upregulation of which of the following cytokines?

IL-17 and tumor necrosis factor alpha - This patient with inflammatory back pain (ie, age <40, insidious onset, worse with rest/better with activity) and enthesitis (ie, tenderness at insertion of tendons/ligaments on bone) most likely has ankylosing spondylitis (AS). AS is characterized by bridging syndesmophytes in the vertebral column and ankylosis (bony fusion) of the apophyseal and sacroiliac joints. AS is an inflammatory disorder thought to originate largely in the gut. Defects in the mucosal barrier and an abnormal intestinal microbiome induce an IL-17-mediated inflammatory response via innate lymphoid cells and T helper cells (eg, Th1, Th17). The risk is greater in individuals with human leukocyte antigen B27 (HLA-B27), likely due to altered antigen presentation. IL-17 stimulates production of additional inflammatory factors, primarily tumor necrosis factor-alpha (TNF-alpha) and prostaglandins, which have synergistic effects with IL-17 and induce bony erosions and abnormal bone regrowth. Skeletal manifestations of AS occur primarily at sites of mechanical stress, such as the entheses, likely due to the migration of activated immune cells to these areas. Treatment options include nonsteroidal anti-inflammatory drugs (eg, ibuprofen, naproxen), which inhibit prostaglandin formation, and anti-TNF-alpha (eg, etanercept, infliximab) and anti-IL-17 (eg, secukinumab) agents.

A 45-year-old man comes to the hospital with acute onset of severe chest pain and diaphoresis. The patient describes the pain as "squeezing," different from any discomfort he has ever had. He has a past medical history of hypertension, and his father underwent coronary artery bypass grafting at age 50. The patient is diagnosed with acute ST-elevation myocardial infarction and undergoes an urgent coronary intervention with stent placement into the right coronary artery. He is also started on high-intensity atorvastatin therapy, along with antiplatelet therapy and appropriate medications to control blood pressure. Four weeks later, the patient's total cholesterol level is 140 mg/dL, down from 200 mg/dL before discharge. Which of the following has most likely increased as the result of the therapy?

LDL receptor density - HMG-CoA reductase inhibitors (statins) inhibit the rate-limiting enzyme in hepatic cholesterol synthesis. In response to these changes, hepatocytes increase their surface expression of the LDL receptor to increase uptake of circulating LDL. Treatment generally produces a 20%-50% reduction in serum cholesterol and LDL concentration.

A 62-year-old woman comes to the emergency department due to difficulty walking that started 3 hours ago. The patient also cannot feel anything on the right side of her body. She has a history of hypertension and diabetes mellitus, and has smoked a pack of cigarettes daily for the past 30 years. The patient's father had a myocardial infarction at age 60. Physical examination shows loss of touch, temperature, and vibratory sensation affecting the entire right side of the body. Sensation is also diminished over the right side of the face. Muscle strength is 5/5 throughout. Speech, vision, and hearing tests are normal. This patient most likely experienced a stroke affecting which of the following brain structures?

Left thalamus - The thalamic ventral posterior lateral nucleus receives input from the spinothalamic tract and dorsal columns, and the ventral posterior medial nucleus receives input from the trigeminal pathway. Damage to these nuclei (eg, ischemia, hemorrhage) may result in contralateral complete sensory loss (eg, touch, pain/temperature, vibration/proprioception). In addition to sensation loss of touch, vibration, and temperature, this patient's unsteady gait despite normal muscle strength suggests a severe proprioception deficit; her complete right-sided hemisensory loss with intact motor strength most likely indicates a lesion affecting the left thalamus.

A 72-year-old man comes to the office due to constipation. His stools have become increasingly hard, small-volume, and difficult to pass. This has been associated with bloating but not vomiting. Symptoms have not improved despite fiber supplementation, polyethylene glycol, and bisacodyl. The patient was recently diagnosed with metastatic pancreatic cancer and was prescribed palliative chemotherapy 2 months ago. His cancer causes severe abdominal pain, which requires high-dose oxycodone to control. Vital signs are within normal limits. The abdomen is mildly distended with decreased bowel sounds. Which of the following medications acts as a µ-opioid receptor antagonist that could alleviate this patient's constipation without inducing withdrawal symptoms?

Methylnaltrexone - This patient likely has opiate induced constipation. Those with refractory OIC may benefit from methylnaltrexone, a peripherally acting µ-opioid receptor antagonist that reverses the anti-peristaltic effect of opiates. It does not cross the blood-brain barrier; therefore, it does not induce opiate-related withdrawal symptoms, allowing for treatment of OIC without disrupting analgesic effects.

A 61-year-old man comes to the emergency department due to fever, chills, and a productive cough with thick, blood-tinged sputum for the past several days. His temperature is 38.8 C (102 F), blood pressure is 90/60 mm Hg, and pulse is 110/min. On examination, the patient is lethargic and ill appearing. Bronchial breath sounds and crackles are present in the right lung. Blood and sputum cultures grow Klebsiella pneumoniae. It is determined that the bacteria express a lipopolysaccharide on their outer membrane surface that stimulates toll-like receptors in the inflammatory cells. This in turn leads to degradation of the IκB inhibitor protein, which normally binds to a latent transcription factor found in the cytoplasm. Which of the following factors is most likely to be directly activated by the removal of this inhibitor protein?

Nuclear factor kappa B - Nuclear factor-kappa B (NF-κB) is part of a family of transcription factors that perform a critical role in the immune response to infection and inflammation. In inflammatory cells, NF-κB is normally present in a latent, inactive state bound to its inhibitor protein, IκB. As part of the classical activation pathway, an extracellular signal, such as the binding of bacterial antigens to a toll-like receptor, causes activation of IκB kinase. This results in ubiquitination and subsequent destruction of IκB with the release of free NF-κB. Once free, NF-κB enters the nucleus and promotes the synthesis of a number of inflammatory proteins such as cytokines, acute phase reactants, cell adhesion molecules, and leukocyte-related growth factors. The inflammatory cascade is self-limiting as NF-κB also stimulates the transcription of more IκB, ultimately rebinding the freed NF-κB.

A nuclear waste disposal plant suffers a containment failure and several grams of iodine-131 are released into the local environment. A number of workers are believed to have been exposed to dangerous levels of the radioactive material. Which of the following should be immediately administered to help prevent tissue damage in these individuals?

Potassium iodide - Like non-radioactive iodine, 131I is preferentially taken up and concentrated within the thyroid gland. There it can cause significant tissue damage that may result in radiation-induced hypothyroidism and thyroid carcinoma (particularly in younger patients). In the event of a nuclear accident, potassium iodide is given prophylactically to protect the thyroid from excessive accumulation of radioactive 131I. Energy-dependent transport of inorganic iodide into the thyroid follicular cell (iodide trapping) is accomplished by the sodium-iodide symporter located on the basolateral membrane. Because this transporter takes up all isotopes of iodide (as well as other ions such as perchlorate and pertechnetate), high serum levels of nonradioactive iodide can competitively inhibit radioactive I131 from entering thyroid follicular cells. Large increases in serum iodide levels also inhibit iodine organification (Wolff-Chaikoff effect) and reduce thyroid hormone release.

An elderly patient is diagnosed with aggressive non-Hodgkin's lymphoma. The tumor cells are shown to stain strongly for the CD20 marker. Which of the following biologic agents can be added to this patient's chemotherapy regimen to help improve the treatment response?

Rituximab - The patient described above has a lymphoma that expresses surface CD20, a B-cell marker. Rituximab is a monoclonal antibody used in lymphoma immunotherapy that specifically targets the CD20 surface immunoglobulin. Another monoclonal antibody used in cancer treatment is trastuzumab (Herceptin), which is used in the treatment of breast cancer.

A 38-year-old woman, gravida 1 para 0, at 34 weeks gestation comes to the office due to increasing swelling in her hands and legs. The patient first noticed the swelling a few weeks ago but now cannot stand for more than an hour without needing to elevate her legs. She has no orthopnea, dyspnea, or chest pain. The patient has no chronic medical conditions and takes no daily medications. Vital signs are normal. Cardiac examination is within normal limits. Lungs are clear to auscultation. Bilateral lower extremities have 2+ pitting edema to the midshin. Urine dipstick shows no protein. Compared to a healthy nonpregnant woman, which of the following parameters is most likely to be decreased in this patient?

Systemic vascular resistance - During pregnancy, the maternal cardiovascular system undergoes physiologic adaptations to increase cardiac output and optimize fetal perfusion. The most significant hemodynamic change is a marked decrease in systemic vascular resistance (SVR), which occurs due to increased release of peripheral vasodilators (eg, nitric oxide, prostacyclin) and decreased vascular sensitivity to vasoconstrictors (eg, norepinephrine, angiotensin II). The formation of a high-flow, low-resistance circuit within the uterus and placenta also contributes to decreased SVR.

A 3-day-old girl is brought to the emergency department by her parents due to persistent vomiting and refusal to feed. The vomiting began early in the morning and has been a greenish-yellow color. The emesis does not contain blood. The infant appears dehydrated. Her heart rate is 175/min and blood pressure is within normal limits. After initial evaluation is complete, the infant undergoes laparotomy. Findings include a normal-appearing duodenum, the absence of a large segment of jejunum and ileum, and the remainder of the distal ileum winding around a thin vascular stalk. Which of the following intrauterine processes is most likely responsible for this patient's condition?

Vascular occlusion - Intestinal atresias of the midgut (eg, jejunum, ileum, proximal colon) are the result of vascular occlusion in utero. Diminished intestinal perfusion leads to ischemia of a segment of bowel, with subsequent narrowing (stenosis) or obliteration (atresia) of the lumen. If a major vessel (eg, the superior mesenteric artery) is occluded, the area of intestinal necrosis is large. The result is a proximal segment that ends in a blind pouch; followed by an area of absent small bowel and associated dorsal mesentery; and, finally, a distal segment of ileum that assumes a spiral configuration around an ileocolic vessel. This specific pattern is known as an "apple-peel" or "Christmas tree" deformity.

A 44-year-old woman comes to the office due to indigestion. The patient says she often develops right upper quadrant abdominal discomfort and nausea with fatty meals, which subside spontaneously after several hours. She does not use tobacco, alcohol, or illicit drugs. The patient immigrated to the United States from Nepal 10 years ago. Abdominal ultrasound reveals numerous gallstones, and she undergoes elective laparoscopic cholecystectomy. The stones in her gallbladder have very low cholesterol content and appear small, dark, and spiculated. Which of the following conditions most likely predisposed this patient to gallstone formation?

Chronic hemolysis - Gallstones are formed by the aggregation of bile constituents and are categorized as cholesterol stones, pigment stones, or mixed stones. Pigment gallstones, which account for only 10%-25% of gallstone cases in the United States, are most common in rural Asian populations. These stones can be brown to black and arise from conditions that increase the amount of unconjugated bilirubin in bile, which promotes calcium bilirubinate precipitation. Brown pigment stones are associated with biliary tract infections (microbes producing β-glucuronidases), whereas black stones occur in the setting of chronic hemolysis (eg, sickle cell anemia, β-thalassemia, hereditary spherocytosis) and increased enterohepatic cycling of bilirubin (eg, ileal disease).

A 69-year-old hospitalized woman is evaluated for new-onset weakness of the right foot. The patient has a history of chronic hip pain due to osteoarthritis and underwent left total hip arthroplasty under general anesthesia while in the right lateral decubitus position 2 days ago. Since the surgery, she has had difficulty moving her right foot. Other medical conditions include lumbar spinal stenosis, hypertension, type 2 diabetes mellitus, and a small ischemic stroke with no residual deficit. On examination of the right lower extremity, the patient is unable to dorsiflex or evert the foot or extend the toes. Sensation is decreased over the dorsum of the right foot. The rest of the neurologic examination of the right lower extremity is normal. Injury to which of the following structures is the most likely cause of this patient's current symptoms?

Common fibular nerve - The common fibular (peroneal) nerve originates in the popliteal fossa as the lateral branch of the sciatic nerve. From there, it courses laterally around the head and neck of the fibula before dividing into the deep and superficial fibular nerves. Because of its proximity to the lateral neck of the fibula, the common fibular nerve is particularly susceptible to compression from a leg cast or during prolonged time in the lateral decubitus position (eg, during surgical procedures). Common fibular neuropathy characteristically presents with deficits involving both the deep fibular (eg, weakness in foot dorsiflexion and toe extension) and the superficial fibular (eg, weakness in foot eversion, sensory loss over the dorsal foot) nerves. In an ambulatory patient, gait examination typically shows foot drop due to loss of dorsiflexion, with the affected foot lifted high off the ground to compensate (ie, steppage gait).

A 56-year-old woman comes to the emergency department due to 3 days of fever, chills, and retrosternal chest pain. She has end-stage kidney disease related to previous uncontrolled hypertension and receives intermittent hemodialysis through a tunneled catheter. Temperature is 39 C (102.2 F), blood pressure is 108/64 mm Hg, and pulse is 120/min. The patient is ill-appearing. The lungs are clear to auscultation, but a pericardial friction rub is present. Echocardiography reveals a moderate-sized pericardial effusion. Pericardiocentesis yields turbid fluid with a large number of neutrophils. Microbiologic analysis of this patient's pericardial fluid is most likely to reveal which of the following pathogens?

Gram positive cocci growing in clusters - This patient with fever and chest pain has a turbid pericardial effusion with a high number of neutrophils, indicating purulent pericarditis. Most cases are caused by bacteria or fungi and develop due to hematologic spread, direct extension from an adjacent pneumonia or cardiac infection, or complication of a penetrating chest injury of cardiothoracic surgery. Staphylococcus aureus, a gram-positive cocci that grows in clusters, is the most frequently isolated organism. It is particularly likely in patients who have portals from the skin to the bloodstream (eg, tunneled dialysis catheter) or from the skin directly to the pericardium (eg, chest injury, recent cardiothoracic surgery). Streptococcus pneumoniae is the most common organism in patients with adjacent pneumonia.

A 34-year-old woman comes to the office due to progressive exertional dyspnea for the past 6 months. She has no chest pain, lightheadedness, or syncope. Medical history is unremarkable and the patient takes no medications. She does not use tobacco, alcohol, or illicit drugs. The patient's mother died of pulmonary arterial hypertension at age 32. Blood pressure is 125/74 mm Hg and pulse is 75/min. BMI is 23 kg/m2. On physical examination, breath sounds are clear without wheezes or crackles. There is a loud S2 at the left upper sternal border. Chest x-ray reveals clear lungs. ECG shows right axis deviation. If this patient's condition is inherited, which of the following is the most likely direct cause of her disease?

Vascular smooth muscle proliferation - This patient's presentation and family history are suggestive of pulmonary arterial hypertension (PAH). Hereditary PAH is most often due to an inactivating mutation in BMPR2, which is transmitted in an autosomal dominant manner with variable penetrance. Patients with this mutation have a predisposition for dysfunctional endothelial and smooth muscle cell proliferation. An insult (eg, infection, drugs) is thought to then activate the disease process, which involves increased levels of vasoconstrictive, proliferative mediators (eg, endothelin) and decreased levels of vasodilative, antiproliferative mediators (eg, nitric oxide, prostacyclin). The end result is vasoconstriction with vascular smooth muscle proliferation, intimal thickening and fibrosis, increased pulmonary vascular resistance, and progressive pulmonary hypertension. Increased pulmonary artery pressure can cause dyspnea and fatigue and can often be detected as a loud pulmonic component of S2. Significant pulmonary hypertension leads to compensatory right ventricular hypertrophy, which may cause right axis deviation on ECG. Right-sided heart failure can eventually occur, evidenced by elevated jugular venous pressure, hepatic congestion, and peripheral edema. The lungs remain clear in PAH as the lung parenchyma is unaffected.

A 68-year-old man comes to the office for follow-up of type 2 diabetes mellitus, hypertension, and hyperlipidemia. The patient's diabetes and hypertension are inadequately controlled, requiring multiple modifications to his medication regimen. Extended counseling is provided regarding today's changes and the overall long-term care plan. The patient's next appointment is in a month for a routine health maintenance examination. As the physician is getting ready to leave the room at the end of the scheduled appointment time, the patient states, "Doc, my urinary stream has been slow for the past 6 months, and I've been waking up twice overnight to use the bathroom. Can we talk about this today?" Which of the following is the most appropriate response by the physician?

"It sounds like we should talk about this; however, we are out of time today. Please make another appointment so I can appropriately evaluate you." - Sometimes, patients raise unexpected concerns that require additional time beyond what is allotted for their appointments; evaluation of this patient's urinary symptoms would extend his appointment, which has already been spent addressing his chronic conditions. Extending the appointment without having adequate time would make subsequent patient visits late and can lead to rushed and incomplete assessments (for this and/or subsequent patients), potentially resulting in misdiagnosis, mismanagement, or poor patient-physician communication. So physicians should direct the patient to schedule another appointment to address the concern, unless it is urgent or likely serious.

A pharmaceutical researcher is trying to develop a vaccine against the hepatitis C virus. She infects a chimpanzee with hepatitis C virus of known genotype and subtype. Several weeks later, a liver sample is obtained, and viral RNAs are extracted from the hepatocytes. A genetic study of the viral genomes reveals that the extracted RNA sequences vary significantly from that of the original infecting virus. This genetic instability is most likely due to the lack of which of the following features during the viral replication process?

3' to 5' exonuclease activity - Hepatitis C virus (HCV) has >6 genotypes and multiple subgenotypes. These variant strains differ primarily at hypervariable genomic regions, such as those found in the sequences encoding its 2 envelope glycoproteins. Moreover, there is no proofreading 3' → 5' exonuclease activity built into the virion-encoded RNA polymerase. As a result, the RNA polymerase makes many errors during replication, and several dozen subspecies of HCV are typically present in the blood of an infected individual at any one time.

A 4-year-old male dies in the hospital from overwhelming infection. Autopsy findings include bone deformities and hepatosplenomegaly. Clumps of erythroid precursor cells are found in the liver and the spleen. The presence of these precursor cells is most likely related to which of the following conditions?

Chronic hemolysis - The presence of erythroid precursor cells in the liver and spleen is indicative of extramedullary hematopoiesis, a condition characterized by erythropoietin-stimulated, hyperplastic marrow cell invasion of extramedullary organs. Extramedullary hematopoiesis is most frequently caused by severe chronic hemolytic anemias, such as β-thalassemia (which the child depicted here may well have had). Extramedullary hematopoiesis can cause a range of skeletal abnormalities. The expanding mass of progenitor cells in the bone marrow thins the bony cortex and impairs bone growth. Pathologic fractures are common in the most symptomatic of children. Maxillary overgrowth and frontal bossing are associated with the characteristic "chipmunk facies" observed in the pediatric population.

A 52-year-old woman comes to the office for follow-up of type 2 diabetes mellitus. The patient was diagnosed with diabetes 6 months ago and initiated intensive lifestyle modification with increased exercise and a calorie-restricted diet; since that time, she has felt better subjectively but has lost only 2 kg (4.4 lb). Vital signs are normal. BMI is 35 kg/m2. Fasting glucose is 155 mg/dL and hemoglobin A1c is 8.0%. Serum renal and liver function markers are normal. After discussion with the patient, metformin therapy is initiated. Which of the following physiologic changes are most likely to result as a direct effect of this medication?

Hepatic gluconeogenesis: decreased Muscle glucose uptake: increased Endogenous insulin production: unchanged Metformin has a number of important metabolic effects: reduced hepatic gluconeogenesis; increased insulin dependent peripheral glucose uptake; reduced circulating lipid levels. Circulating levels of endogenous insulin are unchanged or slightly decreased by metformin; therefore, metformin carries low risk of hypoglycemia. In addition, it reduces caloric intake due to decreased appetite and decreased absorption of glucose, leading to modest weight loss in most patients.

A 67-year-old woman with a known history of lung cancer comes to the office due to hoarseness and difficulty swallowing. She has no disturbances in vision or hearing. On examination, there is loss of the gag reflex on the left side; when the patient is prompted to say "ah," the uvula deviates to the right side. Her left shoulder is drooped and strength is reduced during left shoulder shrug testing. Chest x-ray shows a right lower lobe lung mass and several osteolytic rib lesions. MRI of the head also demonstrates multiple lesions consistent with metastasis. A lesion involving which of the following anatomical structures is most likely responsible for this patient's symptoms?

Jugular foramen - This patient's symptoms are consistent with a lesion involving the jugular foramen , a large aperture located in the base of the skull behind the carotid canal. It is formed by the petrous portion of the temporal bone and the occipital bone. Passing through the jugular foramen are cranial nerves (CN) IX, X, and XI. Lesions of the jugular foramen (eg, due to tumors, trauma, or infection) can result in jugular foramen (Vernet) syndrome, which is characterized by CN IX, X, and XI dysfunction. Symptoms are related to the nerve affected: loss of taste from the posterior 1/3 of the tongue (IX), loss of gag reflex (IX, X), dysphagia (IX, X), dysphonia/hoarseness (X) soft palate drop with deviation of the uvula to the normal side (X), and sternocleidomastoid and trapezius muscle paresis (XI).

A 15-month-old boy is brought to the emergency department due to fever and foot swelling. The patient first developed swelling of the right foot and fever 4 days ago. Today, swelling of the left foot also developed. Medical history is significant for a perianal abscess at 9 months of age. Imaging studies reveal osteomyelitis affecting the metatarsal bones of both feet. Intravenous antimicrobials are administered, but the patient still has fever and bilateral foot swelling 2 weeks later. Laboratory workup shows persistent neutrophilic leukocytosis. Bone biopsy culture grows Staphylococcus aureus. Evaluation for an underlying diagnosis is initiated; via use of a rhodamine derivative, the patient's peripheral blood is stimulated with phorbol myristate acetate (PMA) and compared with a control. Results of the test are shown. Which of the following processes is most likely impaired in this patient?

Phagocytic metabolism - This young patient has multifocal osteomyelitis resistant to treatment, as well as a previous cutaneous (perianal) abscess. This history of multiple, invasive infections is worrisome for underlying immunodeficiency. Laboratory results show a normal white blood cell response to bacterial infection (ie, neutrophilic leukocytosis), but dihydrorhodamine (DHR) testing, which evaluates for phagocytic oxidative response, is abnormal. These findings are consistent with chronic granulomatous disease (CGD). CGD is a disorder of phagocytic metabolism characterized by defective NADPH oxidase, an enzyme in the respiratory burst pathway. Without this enzyme, neutrophils cannot form the reactive oxygen species required for intracellular killing of phagocytized organisms. Patients are predisposed to recurrent infections with certain catalase-positive organisms (eg, Staphylococcus aureus). The DHR assay detects the conversion of DHR (a colorless substance) to rhodamine (fluoresces green) by free radicals produced when the NADPH oxidase pathway is stimulated (eg, by a protein kinase C agonist such as phorbol myristate acetate). In unaffected patients, stimulated cells show increased intensity of fluorescence (ie, rightward shift along the x-axis), indicating appropriate NADPH activity, whereas unstimulated cells remain colorless. In patients with CGD, the stimulated cells show no increased fluorescence (ie, no oxidation of DHR).

A study is conducted to assess the effect of beta-blocker therapy in patients with acute myocardial infarction (MI). The study found that 20 patients out of 90 who took a beta-blocker during the week prior to developing an MI had a major arrhythmia event during hospitalization. The study also determined that 30 patients out of 70 who did not take any beta-blocker developed a major arrhythmia. What was the odds ratio of developing major arrhythmia in patients who took beta-blockers compared to those who did not take beta-blockers?

(20 x 40) / (70 x 30) -The odds ratio (OR) is a measure of association that compares the odds of an outcome occurring based on exposure status. In this case, it represents the odds that the outcome (eg, major arrhythmia) occurred in the presence of the exposure (eg, beta blocker use) compared with the odds that the outcome occurred in the absence of that exposure. Odds of developing major arrhythmia in exposed group = 20/70Odds of developing major arrhythmia in unexposed group = 30/40. Therefore, the OR of developing a major arrhythmia in patients who took beta blockers compared with those who did not is as follows: (a/b) / (c/d) = (20/70) / (30/40) = (20 × 40) / (70 × 30) = 0.38

A young couple has undergone a successful in vitro fertilization procedure. The father has cystic fibrosis and the mother has a sister with cystic fibrosis. The father as well as the mother's sister are both known to have ΔF508 mutations, but the mother's carrier status is unknown. Before making the decision to conceive, the couple underwent extensive genetic counseling regarding the potential risks of having a child with cystic fibrosis. The family pedigree is diagrammed below.

1/3 - Cystic fibrosis (CF) results from an autosomal recessive defect in the CF transmembrane conductance regulator (CFTR) gene. Although most men with CF are infertile due to congenital absence of the vas deferens, they are not sterile and can have children via assisted reproductive technology. Because the father is homozygous for the mutant CFTR allele, he will always transmit the mutant allele to his offspring. Because the mother has an affected sibling and neither of her parents is affected, she most likely had 2 heterozygous carrier parents. Therefore, the mother's 4 possible genotypes are: homozygous for the normal allele, heterozygous with her mother's mutant allele, heterozygous with her father's mutant allele, and homozygous for the mutant allele. However, the mother does not have CF and therefore is not homozygous for the mutant allele. This leaves 3 possible genotypes for the mother. Two of the 3 remaining genotypes result in her being a carrier for the mutant CFTR allele, while the last one results in her being homozygous normal. Therefore, the mother's probability of being a carrier equals 2/3. If the mother is a carrier (2/3 chance), the probability that she will transmit the mutant allele to the child is 1 in 2. As a result, the probability that the child will inherit a mutant allele from the mother (and therefore have CF as the father will always contribute a mutant allele) is: 2/3 x 1/2 = 1/3.

A new antibiotic developed for the treatment of infections caused by resistant gram-positive cocci has a volume of distribution of 11L. It is eliminated by first-order kinetics and has a half-life of 10 hours. If given by a continuous infusion, approximately how much time would it require for the drug to achieve a 95% plasma steady state concentration?

40 hours - During continuous infusion of a drug metabolized by first-order kinetics (i.e. a constant fraction of the drug is eliminated per unit time), the steady state concentration is reached in 4 to 5 half-lives. Thus, it would take approximately 40 hours, or four times the half-life of 10 hours, for the drug in question to reach approximately 95% steady-state concentration.

Geneticists are studying a malfunctioning protein that causes impaired endothelial cell migration and angiogenesis. They have noticed that the amino acid sequence of the protein is truncated compared to normal controls. The mutated mRNA strand is isolated, and analysis shows a nonsense mutation located near the 3' end of the coding region. The 3' terminal coding sequence of the abnormal mRNA strand is shown. 5'--ACG-CUA-CCA-UUG-UAA-CAA-GUU-AGC-UAG--3' Which of the following tRNA anticodons is responsible for adding the last amino acid to the truncated polypeptide during protein translation?

5'-CAA-3' - One codon (AUG) signals initiation of protein synthesis and 3 codons (UAA, UAG, and UGA) stop protein synthesis. Stop codons function only to terminate translation; they do not add amino acids to the polypeptide chain. Transfer RNA molecules (tRNA) transport amino acids to the ribosome and ensure placement of the proper amino acid. One end of the molecule serves as the amino acid binding site; the opposite end contains a specific nucleic acid sequence known as the anticodon that is complementary to one or more mRNA codons (due to base wobbling). The type of amino acid that is bound to each tRNA molecule is determined by its anticodon sequence; this ensures that the proper amino acid is added at each mRNA codon during protein synthesis. Translation of the mRNA template proceeds in the 5' to 3' direction. The mRNA of the dysfunctional protein in the question stem contains an extra stop codon (UAA) before the normal stop codon (UAG) at the end of the template. During protein translation, the first stop codon encountered will bind a release factor, halting protein synthesis. Therefore, the codon just prior to the first stop codon will be the last codon to add an amino acid. In this case, (5'-UUG-3') is the last codon to add an amino acid to the truncated protein, and this amino acid will be carried by the 5'-CAA-3' anticodon (codon-anticodon binding occurs in opposite directions [ie, 5' to 3' binds 3' to 5']).

A group of psychiatrists conducts a double-blind placebo-controlled randomized trial to determine the efficacy and safety of cariprazine for the treatment of depressive episodes associated with bipolar I disorder (bipolar depression) in adults. The treatment response rate was 46% for cariprazine versus 32% for placebo, and the rate for all adverse events was 7% for cariprazine versus 5% for placebo. The differences in rates were statistically significant. Which of the following is the approximate number of patients who must be exposed to cariprazine to cause harm to 1 person who otherwise would not have been harmed?

50 - The number of people who must be exposed to a treatment to cause harm to 1 person who otherwise would not have been harmed is known as the number needed to harm (NNH). It is calculated in a manner similar to the number needed to treat (NNT) but using the absolute risk increase (ARI) instead of the absolute risk reduction: NNH = (1/ARI) ARI is the difference in the rate (risk) of the adverse event (AE) between the treatment group (eg, cariprazine) and the control group (eg, placebo): ARI = (Rate AE cariprazine - Rate AE placebo) In this study, the rate of any AE in the treatment group (ie, cariprazine) is 0.07 (ie, 7%) and in the control (ie, placebo) group is 0.05 (5%). Therefore, the ARI in adverse events of cariprazine versus placebo is: ARI = 0.07 - 0.05 = 0.02 Consequently, the NNH is: NNH = 1/0.02 = 50 Therefore, approximately 50 patients would need to be treated with cariprazine to cause harm to 1 person who otherwise would not have been harmed. The lower the NNH, the more risk of harm; an NNH of 1 means that every patient treated is harmed.

A 28-year-old man comes to the office with a 2-month history of dull low back pain and morning stiffness. The pain had an insidious onset, and the patient does not recall any back trauma. He attempted treatment with acetaminophen and spinal manipulation, which did not provide significant relief. The patient had been a back sleeper, but now must lie on his side to fall asleep easily. Past medical history is unremarkable. Vital signs are normal. Physical examination shows limited anterior flexion of the spine. There is no swelling or warmth of any peripheral joints. X-rays reveal narrowing of the sacroiliac joints. Which of the following is most strongly associated with this patient's condition?

A specific human leukocyte antigen class I allele - Ankylosing spondylitis is a chronic inflammatory disorder of the sacroiliac joints and axial skeleton. It is most common in young and middle-aged men and presents with morning stiffness and low back pain. Ankylosing spondylitis is characterized by destruction of articular cartilage with resulting stiffness and fusion of axial joints. The sacroiliac joints are often tender to palpation, and the spine may have decreased range of motion. X-ray of the sacroiliac joints may reveal erosions, sclerosis, and narrowing; ultimately fusion of the joint spaces. Spine x-rays reveal sclerosis, ligamentous calcification, and vertebral fusion ("bamboo spine"). Ankylosing spondylitis, reactive arthritis, arthritis associated with inflammatory bowel disease, and psoriatic arthritis are seronegative spondyloarthropathies, so-called due to the absence of serum rheumatoid factor. Patients with these diseases have a higher incidence of the human leukocyte antigen (HLA) B27 allele, which encodes a specific HLA class I molecule, compared to the general population (although most patients with HLA B27 will not develop spondyloarthropathies).

A 23-year-old man is evaluated for excessive bruising since childhood. He says some of his family members "bleed easily" and sometimes require blood transfusions. Temperature is 36.7 C (98.1 F). Physical examination is unremarkable. Complete blood cell count is unremarkable. Platelet aggregometry is abnormal. Genetic testing identifies a defect in a platelet surface glycoprotein receptor that normally binds fibrinogen to support platelet aggregation. The abnormal glycoprotein receptor found in this patient is most likely targeted by which of the following medications? A. Abciximab (66%) B.Argatroban (4%) C.Aspirin (2%) D.Clopidogrel (21%) E.Dabigatran (2%) F.Heparin (2%) G.Warfarin (0%)

Abciximab - This patient likely has Glanzmann thrombasthenia, an autosomal recessive disorder that is caused by a deficient or defective glycoprotein (GP) IIb/IIIa on platelet surfaces and that typically presents in childhood with mucocutaneous bleeding. Peripheral smear shows no platelet clumping (an important clue for diagnosis). Platelets are responsible for formation of platelet plugs that stop bleeding from injured vessels (primary hemostasis). Vessel wall injury exposes the subendothelial collagen and matrix. Platelet attachment to exposed collagen is strengthened by GP Ib binding to von Willebrand factor on the vessel wall. The resulting platelet activation leads to the following: Release of mediators (eg, ADP, thromboxane A2 [TXA2]) into circulation, which in turn activates other platelets Conformational structural change of GP IIb/IIIa on platelet surfaces; this allows thousands of copies of GP IIb/IIIa to bind fibrinogen, thereby forming a platelet plug. Abciximab, a GP IIb/IIIa receptor antagonist, inhibits binding of this receptor to fibrinogen. Abciximab and other GP IIb/IIIa inhibitors are useful for treatment of unstable angina and acute coronary syndrome, particularly in patients undergoing percutaneous coronary intervention.

A 42-year-old woman with a history of schizophrenia is brought to the clinic by her parents after attempting to drink rubbing alcohol in response to voices telling her to kill herself. The patient believes that a television newscaster is sending her secret messages and that the devil injects her with poison at night while she is sleeping. She first developed symptoms at age 23 and has had 7 previous psychiatric hospitalizations and 2 suicide attempts. Trials of haloperidol, risperidone, and olanzapine have yielded minimal improvement. Her medical conditions include hypertension and mild obesity. Physical examination is normal. Medication therapy with clozapine is initiated. Which of the following should be regularly monitored in this patient?

Absolute neutrophil count - The second-generation antipsychotic clozapine is the only antipsychotic that has consistently shown superior efficacy in treatment-resistant schizophrenia and schizophrenia associated with persistent suicidality. Clozapine has affinity for multiple dopamine and serotonin receptors, but the precise pharmacological mechanism responsible for its superior efficacy is unknown. Clozapine binds loosely and transiently to dopamine D2 receptors, causing significantly fewer extrapyramidal symptoms compared to first-generation antipsychotics. Neutropenia (<1000 cells/mm3) and the potential for life-threatening agranulocytosis are the major adverse effects of clozapine. The risk of agranulocytosis is approximately 1%; therefore, treatment requires regular monitoring of the patient's absolute neutrophil count. Treatment should be stopped if neutropenia occurs. Seizures and myocarditis are other important adverse effects that require provider vigilance.

A 63-year-old man comes to the office due to fatigue and easy bruising. He has no lymphadenopathy on physical examination. Laboratory results are as follows: Complete blood count Hemoglobin 8.0 g/dL Platelets 40,000/mm3 Leukocytes 20,500/mm3 The patient's peripheral blood smear is shown. Which of the following is the most likely diagnosis?

Acute myeloid leukemia - This patient's peripheral blood smear shows very large nucleated cells (see red cell size for comparison) with scant cytoplasm. These are blast cells, the finding of which makes acute leukemia likely. Closer examination of the smear reveals linear, purple-red inclusions in some of the cells, called Auer rods. These represent fused granules and may be single or multiple within immature myeloid precursors. Auer rods are highly suggestive of acute myeloid leukemia (AML) but not of acute lymphoblastic leukemia (ALL). Although Auer rods are most commonly associated with the M1, M2, and M3 subtypes of AML, they may be found in any type of AML.

A 45-year-old man comes to the office due to progressive dyspnea on exertion and cough for the past several weeks. He has had no fever, chills, chest pain, or leg swelling. The patient is a former smoker with 5-pack-year history. Temperature is 37.2 C (99 F), blood pressure is 120/70 mm Hg, pulse is 82/min, and respirations are 16/min. The patient is hypoxemic at rest, with an oxygen saturation of 88% on room air. Jugular venous pressure is normal. Lung auscultation shows scattered crackles. Heart sounds are normal. Chest x-ray reveals bilateral midalveolar and lower alveolar opacities. Transbronchial lung biopsy reveals lipoproteinaceous material that is positive on periodic acid-Schiff stain and fills up the terminal bronchioles and alveoli. Electron microscopy of the substance shows lamellar bodies. Impaired function of which of the following most likely caused excessive accumulation of the substance in this patient's alveoli?

Alveolar macrophages - This patient's presentation is consistent with pulmonary alveolar proteinosis (PAP), a rare condition characterized by progressive respiratory dysfunction due to the accumulation of surfactant debris within alveolar spaces. Surfactant is a lipoproteinaceous material that appears pink with periodic acid-Schiff staining; it forms concentrically laminated structures (ie, lamellar bodies) that can be seen on electron microscopy. PAP is caused by an imbalance between surfactant production and clearance. In healthy lungs, surfactant is secreted by type 2 pneumocytes and eventually cleared by alveolar macrophages. In PAP, surfactant removal is impaired due to compromised alveolar macrophage function (eg, usually because of defects in granulocyte-monocyte colony-stimulating factor [GM-CSF] signaling). Treatment involves therapeutic whole-lung lavage to wash away surfactant and inhaled GM-CSF replacement therapy.

A 76-year-old woman comes to the emergency department due to fever and burning with urination for 2 days and right-sided back pain for 1 day. Her temperature is 38.3 C (101 F), blood pressure is 110/80 mm Hg, pulse is 94/min, and respirations are 18/min. Examination is notable for right-sided flank tenderness to palpation. Urinalysis is positive for nitrites, leukocyte esterase, and bacteria. Urine and blood cultures are obtained, and the patient is started on antibiotic therapy. The next day, urine culture grows >100,000 colony-forming units/mL of Escherichia coli. The organism is found to have a methyltransferase that methylates ribosomal RNA. This enzyme most likely confers resistance to which of the following classes of antibiotics?

Aminoglycosides - Aminoglycosides interfere with the aminoacyl binding site on the 30S ribosomal subunit. Binding of aminoglycoside causes the cell to misread mRNA and, as a result, it is unable to perform protein synthesis. An important mechanism of resistance to aminoglycosides is the methylation of the aminoglycoside-binding portion of the ribosome, which inhibits the ability of aminoglycosides to interfere with protein translation. Other common resistance mechanisms are the production of enzymes that inactivate the drug by altering its chemical structure or the production of an efflux pump that decreases the drug's intracellular concentration.

A 32-year-old woman comes to the office at 28 weeks gestation due to dyspnea. She has been feeling short of breath when she is supine but has no other symptoms. The patient has had no prenatal care during her pregnancy. She has a history of epilepsy that is well-controlled with medication. She lives with her husband and does not use tobacco, alcohol, or illicit drugs. Her immunizations are up to date, and she has no allergies. Physical examination shows a uterine size that is larger than expected for gestational age. Sonographic assessment shows markedly elevated amniotic fluid levels. Which of the following fetal anomalies would most likely account for this patient's polyhydramnios?

Anencephaly - Polyhydramnios is the excessive accumulation of amniotic fluid. Moderate-to-severe polyhydramnios causes uterine enlargement (eg, increased abdominal circumference) out of proportion to gestational age. Potential complications include preterm labor, placental abruption, and uterine atony due to uterine overdistention. Polyhydramnios also increases the risk of maternal respiratory compromise as the abdominal cavity impairs lung expansion. Polyhydramnios can be due to decreased fetal swallowing or increased fetal urination. Fetal anomalies associated with impaired swallowing include gastrointestinal obstruction (eg, duodenal, esophageal, or intestinal atresia) and anencephaly (a defect of the cranial neural tube). Causes of increased fetal urination include high cardiac output due to anemia or twin-to-twin transfusion syndrome. Maternal diabetes and multiple gestations tend to cause milder polyhydramnios compared to the aforementioned major fetal anomalies. This patient's prenatal use of anti-epileptic therapy (eg, valproate, carbamazepine, phenytoin) is a substantial risk factor for fetal neural tube defects such as anencephaly. Like anencephaly, spina bifida is a group of neural tube defects (eg, occulta, meningocele, myelomeningocele) that can result from maternal folic acid deficiency and use of folic acid antagonists (anti-epileptic drugs, trimethoprim). However, spina bifida occulta, the mildest form, is not associated with polyhydramnios as fetal swallowing of amniotic fluid remains normal.

A 14-year-old girl is brought to the office by her parents for a sports participation examination. The patient is a competitive figure skater and practices before and after school 5 days a week. Her parents are concerned because she recently announced that she will eat only a vegetarian diet, and they are having difficulty providing an adequate variety of meat-free food options around her busy school and athletic training schedule. The patient is otherwise healthy; menarche occurred at age 12 and she has regular menstrual cycles. Vital signs are normal. BMI is 17 kg/m2. Physical examination is normal for age. Which of the following is the most appropriate next step in management of this patient?

Ask the patient to explain her reasons for wanting to change to a vegetarian diet - This adolescent girl has recently adopted a vegetarian diet, which is causing concern for her parents. Such changes in eating habits are common in adolescents and are often due to benign reasons (eg, wanting to eat "healthy" or improve athletic performance, concerns about animals, desire to identify with a role model or peer group). However, an eating disorder (eg, anorexia nervosa) should also be considered, especially in a patient with a low or low-normal BMI (or involvement in an activity that emphasizes leanness). The next step in evaluating this patient is to directly but nonjudgmentally inquire about her motivations for vegetarianism. As when discussing any sensitive topic with adolescents, she may best be interviewed alone, without her parents present. If no worrisome features (eg, preoccupation with weight, abnormal body image) are found or new complications (eg, amenorrhea) develop, she can be followed clinically without additional specific intervention.

A 34-year-old Russian immigrant comes to the office with a 2-month history of exertional dyspnea and progressive lower extremity swelling. He also reports abdominal distension and decreased appetite. The patient has gained 4.5 kg (9.9 lb) over the last 2 months. Medical history is significant for hypertension and recurrent lung infections requiring prolonged antibiotic therapy. The patient has no history of coronary artery disease. He has smoked a pack of cigarettes daily for the past 15 years. The patient undergoes noninvasive cardiac testing, followed by cardiac catheterization. A jugular venous pressure tracing is shown. The waveform shown most likely corresponds to which of the following?

Atrial contraction - The jugular venous pulsation (JVP) waveform is representative of pressure changes in the right atrium throughout the cardiac cycle; the normal waveform consists of 3 peaks (A, C, and V) and 2 descents (x and y). The A wave is caused by atrial contraction at the end of right ventricular diastole, which occurs just before tricuspid valve closure (indicated by S1). The C wave then occurs due to right ventricular contraction against a closed tricuspid valve, which bulges into the right atrium. There is an ensuing decrease in pressure (x descent) as the emptied right atrium relaxes, followed by an uptrend as the right atrium refills with blood from the vena cava. This uptrend culminates in the V wave at the peak of right atrial filling. The pulmonic valve closes (indicated by S2) and the tricuspid valve opens (Choice E) to mark the beginning of passive right atrial emptying into the right ventricle (y descent).

A series of experiments is being conducted to determine the structure and function of different types of bacterial RNA. Cultures of Staphylococcus aureus are exposed to chemicals that lyse the bacterial cells, and the RNA molecules are then extracted. A specific RNA consisting of 90 nucleotides is purified for further analysis. It is found to contain high amounts of chemically modified bases such as dihydrouridine, pseudouridine, and ribothymidine, and its secondary structure arises from base pairing within the chain. Which of the following is the most likely composition of the 3'-end of this molecule?

CCA - Transfer RNA (tRNA) is a form of non-coding RNA composed of 74-93 nucleotides. Specific tRNAs transfer certain amino acid residues to the growing polypeptide during translation. tRNA functions by recognizing the 3 base codon on the mRNA being translated through its anticodon region, which contains complementary bases. The secondary structure of tRNA resembles a cloverleaf and contains the following regions: the acceptor stem, a 3' CCA tail, the D loop, the anticodon loop, and the T loop.

A 55-year-old man comes to the emergency department due to an hour of intense substernal chest burning accompanied by sweating. He has a history of obesity and diet-controlled type 2 diabetes mellitus. On initial evaluation, blood pressure is 110/70 mm Hg and pulse is 60/min and regular. Oxygen saturation is 98% on room air. The patient is comfortable lying flat and no heart murmurs are heard. Lungs are clear on auscultation. Immediate ECG shows normal sinus rhythm with a 2-mm ST-segment elevation in leads II, III, and aVF. Aspirin and morphine are administered. As the patient is about to be transported to the catheterization lab for urgent percutaneous coronary intervention, he develops dizziness and the telemetry monitor shows sinus bradycardia at 40/min. The patient appears pale and diaphoretic and peripheral pulses are faint. Blood pressure is 70/40 mm Hg. Which of the following is the best immediate step for managing this patient's condition?

Atropine - This patient with ST elevation in leads II, III, and aVF has an acute inferior wall myocardial infarction (MI), which most commonly occurs due to occlusion of the right coronary artery (RCA). The RCA usually supplies blood to the sinoatrial node, the atrioventricular node, and most of the bundle of His; therefore, conduction impairment (eg, sinus bradycardia, atrioventricular block) is common in patients with inferior wall MI. Sinus bradycardia is the most common arrhythmia in those with inferior wall MI (nearly 50% of patients); it occurs due to both nodal ischemia and enhanced vagal tone triggered by infarction of myocardial tissue. The bradycardia is usually transient and resolves with restoration of coronary blood flow; however, patients with hypotension or symptoms of bradycardia (eg, lightheadedness, syncope) should be managed with intravenous atropine. The anticholinergic effects of atropine counteract the enhanced vagal tone to increase heart rate and improve symptoms.

A 5-week-old boy is being evaluated for a week-long history of rapid breathing and tiring with feeds. The infant was born at home after an uneventful pregnancy. The mother declined all prenatal testing and ultrasound evaluations. His temperature is 36.7 C (98 F), blood pressure is 76/38 mm Hg, pulse is 124/min, and respirations are 66/min. The patient's cardiovascular examination is notable for a hyperdynamic precordium, a mid-diastolic rumble at the left sternal border, and a 3/6 holosystolic murmur in the apex that radiates to the left axilla. An echocardiogram shows defects in the lower part of the interatrial septum and the interventricular septum. This patient's condition is most likely associated with which of the following genetic conditions?

Autosomal trisomy - This patient's echocardiogram suggests a complete atrioventricular (AV) canal defect, the most common type of cardiac defect in patients with Down syndrome, an autosomal trisomy. Failure of endocardial cushion fusion results in an ostium primum atrial septal defect; a ventricular septal defect; and a single AV valve. Significant left-to-right shunting and AV valve regurgitation lead to excessive pulmonary blood flow and symptoms of heart failure (eg, tachypnea, poor feeding). Auscultatory findings of AV valve regurgitation (holosystolic, best heard at apex) and increased pulmonary venous return (mid-diastolic rumble) are characteristic.

Immunoglobulins of different classes have different structures. Some immunoglobulins have long hinge regions between the Fab and Fc regions, whereas others lack the hinge region. The hinge region provides movement flexibility to the Fab regions. This is most likely to affect which of the following immunoglobulin properties?

Avidity to antigen - IgG, IgA, and IgD also have a hinge region between the Fab and the Fc fragments. The hinge region is rich in cysteine and proline amino acids, which provide significant flexibility to the immunoglobulin and allow greater movement of the Fab fragment arms. Fab fragments with long hinge regions are better able to reach multiple epitopes on an antigen; this increases immunoglobulin avidity because avidity is directly proportional to the number of Fab-antigen interactions. However, long hinge regions are more susceptible to destruction by bacterial proteases (eg, IgA protease).

A 64-year-old man comes to the emergency department due to worsening left lower extremity pain, swelling, and redness over the past 3 days. The patient has had no trauma but reports feeling feverish. Medical history is significant for hypertension and obesity. Temperature is 38 C (100.4 F), blood pressure is 130/86 mm Hg, pulse is 92/min, and respirations are 18/min. BMI is 35 kg/m2. Physical examination shows diffuse erythema extending up to the left midcalf with indistinct border. There is increased warmth, tenderness, and edema of the left leg. No areas of fluctuation or purulent exudate are present, but the interdigital skin of the feet is macerated and fissured. Laboratory testing shows neutrophilic leukocytosis, and Doppler ultrasonography is negative for deep venous thrombosis. Infection with which of the following organisms is most likely responsible for this patient's current symptoms?

Beta-hemolytic streptococci - Cellulitis is an acute bacterial infection of the skin and subcutaneous tissues characterized by progressive skin erythema, warmth, and tenderness. Many patients also have associated fever, leukocytosis, and localized lymphadenopathy. Most cases are driven by disruptions to the skin barrier due to minor trauma, lymphedema, or dermatophytic infection, which allow a portal of entry for bacteria. Nonpurulent cellulitis is characterized by skin warmth, edema, and erythema with no fluctuant nodules. It is most often caused by beta-hemolytic streptococci (groups A, B, C, G, and F); group A streptococcus (Streptococcus pyogenes) accounts for the majority of cases. Purulent cellulitis is characterized by a painful, fluctuant nodule in the dermis or subcutaneous tissue with or without surrounding erythema. The most common cause is Staphylococcus aureus; strains that express the virulence factor Panton-Valentine leukocidin are particularly likely to cause skin abscess.

A 3-year-old girl is brought to the emergency department with abrupt-onset vomiting followed by frequent, large-volume, watery diarrhea for the last day. She has no prior medical conditions but has not received recommended vaccinations. Temperature is 37.8 C (100 F). Physical examination shows mild dehydration. The abdomen is soft and mildly tender to palpation throughout. Bowel sounds are increased. Polymerase chain reaction testing of the stool sample yields a virus with a segmented, double-stranded RNA genome. Which of the following pathologic findings is most likely to be present in this patient?

Blunting of the villi in the duodenum and proximal jejunum - Rotavirus is a segmented, nonenveloped, double-stranded RNA virus that is primarily transmitted via the fecal-oral route. It is a major cause of infectious diarrhea in children worldwide and typically presents in those age <5 with acute, self-limited fever and watery diarrhea that may lead to dehydration and electrolyte abnormalities. Rotavirus vaccination is protective and has resulted in a dramatic reduction in disease incidence. Rotavirus invades the villous epithelium of the duodenum and proximal jejunum. Infection causes diarrhea via multiple mechanisms including villous blunting (loss of absorptive capacity), proliferation of secretory crypt cells (secretory diarrhea), and reduced brush border enzymes (accumulation of unmetabolized disaccharides in the small intestine lumen). Rotavirus-induced diarrhea is noninflammatory; therefore, fecal leukocytes and erythrocytes are not typically seen. Repeat infections are often less severe than initial infection due to the generation of protective secretory IgA against rotaviral antigens.

A 36-year-old woman comes to the office due to dyspnea and weakness that are brought on while doing ordinary chores around the house. The patient has no other medical conditions and takes no medications. She does not use tobacco, alcohol, or illicit drugs. Her mother had similar symptoms and died at age 42. After an extensive workup, a lung biopsy is performed. Light microscopy of the tissue sample shows medial hypertrophy, intimal fibrosis, and decreased intraluminal diameter of the small branches of the pulmonary artery. Which of the following is the most appropriate pharmacotherapy for this patient's current condition?

Bosentan - Pulmonary hypertension causes specific morphologic findings in the branches of the pulmonary arteries, including increased arteriolar smooth muscle thickness (medial hypertrophy), intimal fibrosis, and significant luminal narrowing. In the setting of severe hypertension, lesions can progress to form interlacing tufts of small vascular channels called plexiform lesions. These changes can occur in both pulmonary hypertension due to underlying cardiac, lung, or thrombotic disease and in pulmonary arterial hypertension (PAH). PAH most commonly presents as dyspnea and exercise intolerance in women age 20-40. This patient's family history is suggestive of the familial form of PAH, which is most often caused by inactivating mutations involving the proapoptotic BMPR2 gene. The resulting increase in endothelial and smooth muscle cell proliferation leads to vascular remodeling, elevated pulmonary vascular resistance, and progressively elevated pulmonary arterial pressure. Although lung transplantation is the definitive treatment for PAH, medical therapy targeting the effects of endothelial dysfunction can help improve symptoms. Bosentan is an endothelin-receptor antagonist that blocks the effects of endothelin (a potent vasoconstrictor that also stimulates endothelial proliferation). Bosentan therapy decreases pulmonary arterial pressure and lessens the progression of vascular remodeling and right ventricular hypertrophy.

A 24-year-old nurse comes to the office for his annual wellness visit that includes tuberculosis screening. The patient has no chronic medical conditions and does not have recent fever, cough, or other health changes. He receives an intradermal injection of tuberculin on the inner surface of his forearm. Two days later, he has a distinct area of induration 20 mm across at the injection site. Which of the following interactions is essential to the development of this patient's skin reaction?

CD28 on T lymphocytes with CD80 on epidermal dendritic cells - Tuberculin skin testing (TST) introduces purified proteins from Mycobacterium tuberculosis into the dermis. Because M tuberculosis is primarily countered by the cell-mediated immune response (CMIR), previously infected patients have primed, antigen-specific CD4 T lymphocytes that rapidly replicate and mature in response to tuberculin antigen re-exposure. T-lymphocyte activation is a 2-step process: T lymphocytes with specific T cell receptors recognize antigens presented on class II major histocompatibility complexes on the surface of dendritic cells (eg, antigen-presenting cells [APCs]). A costimulatory interaction between CD28 on the T lymphocyte and CD80/86 on the dendritic cell then allows for activation. Activated CD4 T lymphocytes release inflammatory cytokines (eg, IL-1, IL-6, TNF alpha) that stimulate/recruit other immune cells and increase vascular permeability, forming an indurated wheal of inflammation following TST exposure. Tuberculin reactions appear 24-72 hours after antigen exposure due to delays between initial antigen processing by APCs, T-cell activation, and amplification of the cellular response.

A 32-year-old male with anorexia and fatigue is found to have persistently elevated serum alanine aminotransferase (ALT) and aspartate aminotransferase (AST) levels. He is also positive for HBsAg. Which of the following best describes the mechanism of liver injury in this patient?

CD8+ T lymphocyte response to viral antigens on the cell surface resulting in hepatocyte damage - Hepatitis B virus infection progresses through two phases: the proliferative phase and the integrative phase. In the proliferative phase, the entire virion and all related antigens of the episomal HBV DNA are present. On the hepatocyte cell surface, viral HBsAg and HBcAg are expressed in conjunction with the major histocompatibility complex (MHC) class I molecules. This expression serves to activate the cytotoxic CD8+ T lymphocytes, which respond by destroying the infected hepatocytes. Note that the virion itself does not have a cytopathic effect. In the integrative phase, the HBV DNA is incorporated into the host genome of those hepatocytes that survived the immune response. Infectivity ceases and liver damage tapers off when the antiviral antibodies appear and viral replication stops. The risk of hepatocellular carcinoma, however, remains elevated because of the HBV DNA that has been integrated into the host genome.

A 35-year-old man is brought to the emergency department after collapsing in his garage. He recently bought a classic 1960s convertible and was repairing it when he collapsed. The patient was inside the garage but had the door half-open. He lost consciousness after working on the car for 2 hours with the engine running. The patient has no known medical problems and takes no medications. He does not use tobacco, alcohol, or illicit drugs. Which of the following best reflects the combination of findings that would have been expected in an arterial blood sample taken when the patient lost consciousness?

Carboxyhemoglobin (%) [normal = 0-2]: 40 PaO2 (mmHg) [normal = 85-100]: 95 Methemoglobin (%) [normal = 0-1]: 1 Carbon monoxide (CO) is a colorless, odorless gas product of the combustion of carbon-containing compounds. CO poisoning can occur after exposure to automobile exhaust (especially in older cars without catalytic converters), fire smoke, or improperly vented natural gas appliances. CO has 2 major effects on oxygen delivery to tissues: CO binds to hemoglobin with an affinity approximately 250 times that of oxygen, reducing the number of heme binding sites available to oxygen. As a result, levels of CO-bound hemoglobin, represented by carboxyhemoglobin (HBCO), increase. By decreasing the fraction of hemoglobin available for oxygen binding, CO decreases the oxygen-carrying capacity and oxygen content of blood, but not the amount of oxygen dissolved in plasma, reflected by the partial pressure of oxygen (PaO2). CO poisoning causes a leftward shift of the hemoglobin-oxygen dissociation curve, reflecting a decreased tendency for oxygen to unload in the tissues. Methemoglobin results from drug exposures (ie, dapsone, nitrites) as well as enzyme deficiencies and hemoglobinopathies.

A 47-year-old man comes to the office for follow-up of his elevated blood pressure. The patient's medical history is significant for stable angina pectoris, for which he takes sublingual nitroglycerin as needed. He has no orthopnea, paroxysmal nocturnal dyspnea, or lower extremity swelling. His blood pressure is 154/107 mm Hg and pulse is 86/min. Physical examination shows no abnormalities. The patient is started on a low dose of atenolol. Which of the following cellular changes will most likely occur as a direct effect of this medication?

Cardiomyocyte [cAMP]: decreased Juxtaglomerular cell [cAMP]: decreased Vascular smooth muscle [cAMP]: no significant change At low doses, atenolol is a selective β1 adrenergic antagonist. β1 receptors are found in cardiac tissue and on renal juxtaglomerular cells, but not in vascular smooth muscle. The β1 receptor is a G protein-coupled receptor (GPCR) associated with Gs, which increases intracellular cAMP levels. Blockade of the β1 receptor leads to decreased cAMP levels in cardiac and renal tissue without significantly affecting cAMP levels in vascular smooth muscle.

A multinational research institute conducting experiments on human circulatory physiology enrolls a healthy 30-year-old male volunteer to assess the oxygen consumption rate of various organs. During the study, the blood oxygen content of the aorta and several other vessels is measured at rest. The greatest difference in these measurements will most likely be between the aorta and which of the following blood vessels?

Coronary sinus - Arterial blood supply to the cardiac muscle (myocardium) is provided by the right and left coronary arteries arising directly from the aortic root. Most cardiac venous blood drains into the right atrium via the coronary sinus, with the remainder draining directly into the other chambers of the heart. Three specific features distinguish cardiac circulation from blood flow to skeletal muscle and viscera: the left ventricle is perfused only during diastole; myocardial oxygen extraction is high; myocardial oxygen demand and coronary blood floor are tightly regulated. Deoxygenated systemic blood returning via the vena cava contains more oxygen than coronary sinus venous blood. Because the pulmonary artery contains an admixture of blood returning via coronary sinus and systemic venous circulation, its oxygen content is also higher than pure coronary venous return.

A 48-year-old man comes to the hospital after a day of high-grade fever, progressive headache, and double vision. The patient has been having purulent nasal drainage and frontal headache for the past several days. He has a history of type 2 diabetes mellitus. His temperature is 38.9 C (102 F), blood pressure is 110/70 mm Hg, and pulse is 94/min. Physical examination shows ptosis, mydriasis, mild proptosis, and loss of the corneal reflex of the right eye. Visual acuity is normal in both eyes, but the patient is unable to move the right eye in any direction. There is decreased sensation in the right upper face. Which of the following structures is most likely involved in this patient's presentation?

Cavernous sinus - Cavernous sinus thrombosis is most commonly due to contiguous spread of an infection from the medial third of the face, sinuses (ethmoidal or sphenoidal), or teeth. The infection can communicate in a retrograde fashion into the cavernous sinus through the valveless facial venous system (via the superior and inferior ophthalmic veins). Common pathogens include Staphylococcus aureus (most common) and streptococci, although fungal organisms (eg, Mucor, Rhizopus) are responsible in rare cases. Patients typically experience headache, fever, and diplopia. Ocular muscle paralysis occurs due to injury of cranial nerves (CNs) III, IV, and VI. CN III palsy may also result in ptosis and mydriasis. Involvement of the ophthalmic and maxillary branches of CN V can cause loss of both upper facial sensation and the afferent limb of the corneal reflex. Proptosis (eye protrusion) and chemosis (conjunctival swelling) may also be evident due to impaired venous drainage through the ophthalmic veins.

A 26-year-old man is being evaluated for recurrent boils and skin abscesses. Anterior nares swab culture is performed to determine colonization by the culprit bacteria, which yields gram-positive cocci in clusters. The bacteria are able to grow in a media containing oxacillin. PCR testing reveals the pathogen has acquired the mecA gene. These bacteria are most likely to be resistant to which of the following antibiotics?

Cefazolin - This patient with recurrent boils and skin abscesses is likely colonized with Staphylococcus aureus, a gram-positive cocci that grows in clusters. Methicillin-resistant strains are able to grow in the presence of oxacillin due to the acquisition of a mobile genetic element that contains the mecA gene, which encodes for penicillin-binding protein (PBP) 2a. PBPs catalyze peptidoglycan cross-linking during cell wall synthesis; they are the target of beta-lactam medications, which bind to and irreversibly destroy the enzyme. Unlike other PBPs, PBP 2a has a low affinity for beta-lactams and continues to cross-link peptidoglycan in the presence of oxacillin, methicillin, cephalosporins (eg, cefazolin), and other beta-lactam medications.

A 36-year-old man comes to the office due to skin lesions on his palms. The patient has yellowish skin nodules over the palmar creases that have been increasing in size and number over the past several years. He also has small clusters of yellow papules on his elbows, knees, and buttocks. His father died of a myocardial infarction at age 56. Biopsy of his lesions shows accumulation of lipid-laden macrophages. Immunoblot analysis suggests a lack of ApoE3 and ApoE4 in his circulating lipoproteins. Which of the following is most likely impaired in this patient?

Chylomicron remnant uptake by liver cells - Familial dysbetalipoproteinemia (type III hyperlipoproteinemia) is characterized by xanthomas and premature coronary and peripheral vascular disease. It is an autosomal recessive disorder that is clinically more severe in patients with other conditions affecting lipoprotein metabolism (eg, diabetes, hypothyroidism). The primary defects in familial dysbetalipoproteinemia are in ApoE3 and ApoE4, apolipoproteins found on the triglyceride-rich lipoproteins (chylomicrons and VLDLs) that are responsible for binding hepatic apolipoprotein receptors. Without ApoE3 and ApoE4, the liver cannot efficiently remove chylomicrons and VLDL remnants from the circulation, causing their accumulation in the blood and resultant elevations in cholesterol and triglyceride levels.

A 65-year-old man comes to the emergency department due to an episode of hemoptysis that occurred earlier this morning. The patient estimates that he coughed up about 100 mL of blood. Prior to this episode he was in his usual state of health and felt well, with no fevers, night sweats, or weight loss. He has had occasional episodes of blood-tinged sputum over the past few months. The patient's past medical history is significant for tuberculosis (TB) that was effectively treated several years ago. He had smoked a pack of cigarettes daily for 45 years but quit 4 years ago. A representative cut of the CT scan, shown in the image below, demonstrates changes consistent with prior TB infection, including an old left upper lobe cavity. Compared to prior scans, the main difference is that the cavity now appears to be filled with a round mass. Which of the following best describes this patient's condition?

Colonization - Aspergillus fumigatus is a mold that is widely present in organic matter. It forms septate hyphae that branch at 45-degree angles (V-shaped branching). The spores are inhaled with the air and are typically cleared by the mucus and ciliated epithelium of the respiratory tract. In individuals with suppressed immune defenses, Aspergillus causes a wide spectrum of diseases. This patient likely has hemoptysis due to an aspergilloma (mycetoma), which represents Aspergillus colonization. Aspergillomas develop in old lung cavities (from tuberculosis, emphysema, sarcoidosis). Aspergillus colonizes the cavity, forming a "fungus ball" (seen in the left upper lobe on this patient's CT) without lung tissue invasion. Classically, an aspergilloma will appear on chest x-ray as a radiopaque structure that shifts when the patient changes position. This condition may be asymptomatic or it may cause cough and hemoptysis.

A 53-year-old woman comes to the office due to progressive distension and tightness in her right lower extremity for the past year. She has had no shortness of breath, fever, or skin rash. The patient was diagnosed with melanoma of the right thigh approximately 2 years ago and underwent surgical resection. Two of her inguinal lymph nodes tested positive for metastasis, and she subsequently underwent inguinal lymphadenectomy and received adjuvant therapy. Recent imaging revealed no evidence of recurrent malignancy. The patient's medical history also includes hypertension and hyperlipidemia. Vital signs are within normal limits. Physical examination shows clear lungs and normal heart sounds without murmurs or gallops. No jugular venous distension is present. The right lower extremity is significantly larger in diameter than the left. Edema is present up to the thigh, with mild skin indentation after application of pressure. The skin overlying the involved area is thickened and dry. Laboratory testing, including complete blood count, serum metabolic panel, and urinalysis, is within normal limits. Treatment of this patient's symptoms should include which of the following strategies?

Compression to increase lymphatic flow - This patient with chronic swelling and thickened skin of the right leg most likely has chronic lymphedema. Lymphedema can be congenital, but it most commonly results from an acquired disruption of lymphatic drainage that allows for accumulation of lymphatic fluid in the interstitial tissue. Malignancy and its treatment (eg, radiation, lymphadenectomy) are frequent underlying causes; other causes include chronic inflammation (eg, recurrent cellulitis, connective tissue disease) and parasitic infection (ie, filariasis due to Wuchereria bancrofti). Obesity is often a strong contributing factor. Lymphedema usually presents with swelling, heaviness, and discomfort in one or more extremities. In early disease, the edema is pitting and the skin remains soft. However, over time there is progressive deposition of subcutaneous collagen and adipose tissue, which leads to firm, dry, and thickened skin and nonpitting edema. Treatment is difficult and typically involves conservative management to increase lymphatic drainage via compression bandages or physiotherapy (ie, manual lymphatic drainage).

A 46-year-old homeless man comes to the emergency department with fever and chest pain that worsens with swallowing. The patient has been hospitalized several times recently with Pneumocystis jirovecii pneumonia. He has a history of intravenous drug use. His temperature is 37.8 C (100 F). Oropharyngeal examination is remarkable only for poor dentition. Esophagogastroduodenoscopy is performed and reveals esophageal hyperemia and linear ulcerations. Which of the following is the most likely cause of this patient's condition?

Cytomegalovirus - Pneumocystis jirovecii affects almost exclusively immunocompromised individuals; therefore, it is likely that this patient has HIV. He has now developed the painful swallowing characteristic of esophagitis. The 3 main causes of HIV-associated esophagitis are Candida (most common), cytomegalovirus (CMV), and herpes simplex virus (HSV). All can manifest as dysphagia (difficulty swallowing) and/or odynophagia (pain on swallowing) and can be difficult to distinguish clinically; however, accurate diagnosis is essential for treatment. CMV esophagitis is typically characterized endoscopically by large, shallow linear ulcerations (most commonly in the distal esophagus) with intranuclear and cytoplasmic inclusions seen microscopically.

A 53-year-old man comes to the physician due to frequent headaches and dizziness. He has a history of hypertension and peptic ulcer disease. His medications include daily chlorthalidone and antacids as needed. The patient's temperature is 37 C (98.6 F), blood pressure is 146/92 mm Hg, pulse is 89/min, and respirations are 16/min. His BMI is 26 kg/m2. Physical examination shows facial plethora and moderate splenomegaly. Laboratory results are as follows: Complete blood count Hemoglobin 21.5 g/dL Hematocrit 64% Erythrocytes 7.6 million/mm3 Mean corpuscular volume 90 μm3 Mean corpuscular hemoglobin 31 pg/cell Mean corpuscular hemoglobin concentration 33% Hb/cell Red blood cell distribution width 14.0% (n = 10.3%-14.1%) Platelets 545,000/mm3 Leukocytes 15,500/mm3 This patient most likely has a mutation in which of the following types of proteins?

Cytoplasmic tyrosine kinase - This patient most likely has polycythemia vera, a myeloproliferative disorder characterized by uncontrolled erythrocyte production. Patients frequently present with nonspecific symptoms (eg, headache, weakness, diaphoresis), aquagenic pruritus, facial plethora (reddish complexion), and splenomegaly. Associated conditions include peptic ulcer disease (altered mucosal blood flow due to increased viscosity) and gouty arthritis (higher red cell turnover). Laboratory studies show increased erythrocyte mass, thrombocytosis, leukocytosis, and low erythropoietin levels. Erythrocyte indices are usually normal. Polycythemia vera is caused by abnormal transduction of erythropoietin growth signals. The erythropoietin receptor has no intrinsic kinase activity and must interact with Janus kinase 2 (JAK2), a cytoplasmic (non-receptor) tyrosine kinase, to initiate downstream signaling. Virtually all patients with polycythemia vera have a mutation in JAK2 that causes constitutive activation of its kinase domain, resulting in clonal proliferation of myeloid cells. JAK2 mutations have also been implicated in essential thrombocythemia, primary myelofibrosis, and other myeloproliferative disorders.

A 58-year-old woman comes to the emergency department due to abdominal discomfort and bloating. Physical examination reveals ascites. Abdominal ultrasonography shows a mass in the left ovary, and subsequent paracentesis reveals malignant cells. Surgical excision of the mass is performed. Histopathology shows high-grade serous carcinoma composed of atypical cells with pleomorphic nuclei and mitotic figures. Large numbers of T cells are present in the tumor stroma; further evaluation shows the tumor expresses PD-1 ligands that interact with the PD-1 receptor on the infiltrating T cells. Which of the following is most likely occurring in the tumor microenvironment as a result of this interaction?

Decreased IL-2 production - In order to survive and proliferate, cancer cells must develop adaptations to avoid detection by the innate and adaptive immune response. One common adaptation is overexpression of programmed cell death ligand 1 (PD-L1) on the tumor cell surface. PD-L1 binds to programmed cell death protein 1 (PD-1) on natural killer (NK) cells, T cells, and B cells and down-regulates their activity. When PD-L1 binds to PD-1 on an activated T-cell, it converts it to an exhausted T-cell. Exhausted T-cells primarily express immunoinhibitory molecules and are unable to effectively destroy cancer cells with perforins/granzymes or to secrete immunostimulatory cytokines (eg, IL-2). In order to counter the overexpression of PD-L1 by certain tumors, patients can receive immunotherapy medications that block PD-1 (eg, pembrolizumab) or PD-L1 (eg, atezolizumab), which restores anti-tumor T-cell activity.

A group of investigators is studying osteoprotegerin function in patients with postmenopausal osteoporosis. Using an engineered mouse strain, they develop a chimeric monoclonal antibody that blocks binding of receptor activator of nuclear factor kappa B (RANK) to RANK ligand (RANK-L). The investigators inject this monoclonal antibody into human subjects. Eight weeks later, a bone biopsy is obtained from all subjects. Which of the following is the most likely observed finding?

Decreased bone resorption - Interaction between the receptor for activated nuclear factor kappa B (RANK) and its ligand (RANK-L), is critical for the development of mature, multinucleated osteoclasts. This binding is blocked by osteoprotegerin (OPG), which acts as a decoy receptor for RANK-L (preventing it from interacting with RANK). By binding RANK-L, OPG reduces the differentiation and survival of osteoclasts, resulting in decreased bone resorption and increased bone density. Estrogen maintains bone mass in premenopausal women by inducing production of OPG by osteoblasts and stromal cells and decreasing expression of RANK on osteoclast precursors. The loss of OPG at menopause leads to increased osteoclast activity that predisposes to osteoporosis. Denosumab is a monoclonal antibody used in the treatment of postmenopausal osteoporosis. It works similar to OPG in that it binds RANK-L and prevents its interaction with RANK receptor. Denosumab therefore causes decreased osteoclast differentiation and activity as well as decreased bone resorption.

Researchers are testing various modalities to reduce glutamate-induced excitotoxic neuronal injury after ischemic stroke. The investigators record the postsynaptic potential of cortical neurons under various conditions. The neurons are exposed to glutamate, which causes them to depolarize; excessive depolarization triggers cell death. The experiment is repeated after altering the electrolyte composition of the extracellular fluid. When the extracellular magnesium concentration is increased, glutamate-induced neuron depolarization and cell death are decreased. Which of the following changes in ion conductance explains the improved neuron survival in response to magnesium?

Decreased influx of calcium and sodium - Glutamate is the primary excitatory neurotransmitter in the brain. The glutamate NMDA receptor is a potential-dependent and ligand-gated ion channel. In the resting hyperpolarized state, magnesium occupies the NMDA receptor ion pore, forming a constitutive magnesium plug to prevent the flow of other cations. The channel opens when there is displacement of the magnesium plug and binding of glutamate/. This leads to an influx of calcium and sodium. During acute ischemic stroke, a sudden, massive rise in extracellular glutamate occurs within the infarct zone. Sustained exposure to high glutamate levels causes intracellular calcium overload, ultimately accumulating in mitochondria, triggering apoptotic neuron death (excitotoxicity). These dying neurons release more glutamate into the extracellular space, creating a domino effect on neighboring neurons, further propagating damage beyond the original ischemic core. As a result, modulation of the NMDA receptor is an emerging frontier in neuroprotection.

A 75-year-old man is found unresponsive in his home. Emergency medical services are called by the neighbor, but the patient is pronounced dead at the scene. The patient had a history of hypertension, type 2 diabetes mellitus, and coronary artery disease, as well as a 25-pack-year smoking history. He worked at a coal mine in the remote past and retired as an auto mechanic 15 years ago. Autopsy is performed; a section of lung tissue stained with hematoxylin and eosin is shown. Which of the following is the most likely cause of this patient's abnormal histopathology findings?

Decreased left ventricular systolic function - This patient's autopsy demonstrates an abundance of alveolar macrophages filled with brown pigment in the lung parenchyma. Hemosiderin-laden macrophages located in the lungs are usually the result of chronic passive lung congestion in the setting of heart failure (eg, left ventricular systolic dysfunction). Elevated pulmonary venous pressure leads to transudation of fluid across the alveolar-capillary membrane (pulmonary edema) and can cause breaks in the endothelium with extravasation of red blood cells into the alveoli and lung parenchyma. As alveolar macrophages engulf and degrade the extravasated red blood cells, the released iron accumulates as intracellular hemosiderin. Severe left ventricular systolic dysfunction is associated with an increased risk of fatal ventricular arrhythmia (ie, ventricular tachycardia or fibrillation), which was the likely cause of death in this patient.

A 50-year-old man is seen in the office for new-onset constipation. Family history is unremarkable. Vital signs are within normal limits. Physical examination reveals normal findings. Serum laboratory results are as follows: Creatinine 1.1 mg/dL Calcium 11.3 mg/dL Phosphorus, inorganic 1.9 mg/dL Parathyroid hormone 98 pg/mL (normal: 10-65) Which of the following is the most likely mechanism causing this patient's low serum phosphate?

Decreased phosphate reabsorption in the proximal tubules - This patient has mild hypercalcemia associated with hypophosphatemia, suggesting an excess of parathyroid hormone (PTH). PTH is released in response to hypocalcemia and acts to raise plasma calcium and lower plasma phosphorus by the following mechanisms: In the bones, PTH indirectly activates osteoclasts to increase bone resorption, releasing calcium and phosphorus. In the kidneys, PTH decreases proximal tubular reabsorption of phosphate and increases calcium reabsorption in the distal convoluted tubule and collecting duct. PTH upregulates 1-alpha-hydroxylase in the kidney, which converts 25-hydroxyvitamin D to 1,25-dihydroxyvitamin D (the more active form), further increasing calcium absorption in the kidneys and small intestine.

A 23-year-old man is evaluated for prolonged paralysis following general anesthesia for an elective surgical procedure. Anesthesia was induced with propofol, succinylcholine was used for intubation, and anesthesia was maintained via propofol and fentanyl infusions. The patient has not had a return of neuromuscular function after several hours. His mother then discloses that she has also been "slow to wake up" from anesthesia. Which of the following is the most likely reason for this patient's prolonged paralysis?

Decreased plasma hydrolysis - This patient who developed prolonged muscle weakness (eg, failure of spontaneous respiration, voluntary muscle activity) after receiving succinylcholine most likely has pseudocholinesterase deficiency, an autosomal recessive disorder caused by a genetic polymorphism in the BCHE gene. Succinylcholine is a depolarizing neuromuscular blocking agent used to induce skeletal muscle relaxation during intubation and surgery. It acts as a competitive agonist of nicotinic acetylcholine receptors of the motor endplate, where it induces persistent depolarization, leading to desensitization and skeletal muscle paralysis. Succinylcholine is rapidly hydrolyzed by plasma pseudocholinesterase; only around 10% of the administered dose reaches the neuromuscular junction, where it typically has a duration of action of <10 minutes. Patients with pseudocholinesterase deficiency are unable to metabolize succinylcholine, which leads to a large amount of the drug reaching the neuromuscular junction. Heterozygotes may experience double the normal duration of paralysis; homozygotes can have persistent paralysis for several hours.

A 35-year-old woman comes to the office due to fever, headache, severe muscle aches, and sore throat for the last 4 days. Physical examination shows mild pharyngeal erythema and nasal congestion. A rapid influenza antigen test is positive. The patient's condition improves over the next several days despite receiving only symptomatic treatment. In response to the influenza virus, infected respiratory epithelial cells begin secreting increased quantities of interferons. The specific interferons secreted by these cells will most likely cause which of the following changes?

Decreased protein synthesis by infected cells - Type I interferons (α and β) are synthesized by most human cells in response to viral infections. Once secreted, α and β interferons bind to type I interferon receptors found on infected and neighboring cells (autocrine/paracrine signaling). This results in transcription of antiviral enzymes capable of halting protein synthesis, such as RNase L (endonuclease that degrades all RNA in the cell) and protein kinase R (inactivates eIF-2, inhibiting translation initiation). However, these enzymes become active only in the presence of double-stranded RNA, which forms in infected cells as a result of viral replication. As a result, normal metabolism and protein synthesis can continue in uninfected cells but is selectively inhibited in virally infected cells.

A 32-year-old man is started on infliximab for treatment of refractory Crohn disease. Ten days later, he develops joint pain and a pruritic skin rash. Skin biopsy shows scattered areas of fibrinoid necrosis and neutrophil infiltration involving his small blood vessels. Which of the following findings is most likely to accompany this patient's condition?

Decreased serum C3 level - This patient's symptoms and biopsy findings are suggestive of acute serum sickness, a condition caused by tissue deposition of circulating immune complexes (type III hypersensitivity). The most common manifestations include fever, pruritic skin rash, and arthralgias that begin 7-14 days after exposure to an antigen. Lymphadenopathy and proteinuria may also occur in some patients. Histologic examination of affected tissues typically shows small vessel vasculitis with fibrinoid necrosis and intense neutrophil infiltration. Deposition of IgG and/or IgM complement-fixing antibodies results in localized complement consumption and hypocomplementemia (decreased serum C3 levels).

A 66-year-old man comes to the office due to increasing shortness of breath over the past 3 weeks. He has had a nonproductive cough for several months, which he attributes to allergies, but also notes a 9-kg (20-lb), unintentional weight loss over this time. The patient has no prior medical conditions and takes no medications. He has a 50-pack-year smoking history. Temperature is 36.7 C (98 F), blood pressure is 120/80 mm Hg, pulse is 72/min, and respirations are 18/min. Chest x-ray is shown. Which of the following would be the expected physical examination finding over the right lower chest?

Decreased tactile fremitus - This patient's chest x-ray shows blunting of the right costophrenic angle and opacification of much of the right lung consistent with a large, right-sided pleural effusion. His unintentional weight loss and smoking history suggest a malignant etiology. Pleural effusions are collections of fluid that form between the visceral pleura that lines the lung and the parietal pleura that lines the thoracic cavity. On physical examination, the fluid acts to insulate vibrations and sounds originating within the airways of the lung. Therefore, both tactile fremitus, the vibration created by vocalized sound (eg, saying "ninety-nine"), and breath sounds are decreased over a pleural effusion. In addition, the relatively high density of pleural fluid compared with air creates dullness to percussion.

A 46-year-old woman with a history of hiatal hernia and severe gastroesophageal reflux undergoes antireflux surgery. The reflux symptoms were refractory to medical therapy, so an endoscopic fundoplication procedure is performed. During surgery, the hiatal defect is repaired, and the gastric fundus is mobilized and wrapped around the lower esophagus to reinforce the lower esophageal sphincter. No esophageal or gastric injuries occurred, but a neural structure traversing the esophageal hiatus of the diaphragm was inadvertently injured. Which of the following is the most likely potential effect of this operative injury?

Delayed gastric emptying - The anterior and posterior vagal trunks (branches of the vagus nerve) also pass through the esophageal hiatus and are vulnerable to injury during fundoplication. These nerves supply parasympathetic innervation to the gastrointestinal tract and stimulate gastric muscle contraction and motility. Injury to the vagal trunks can lead to delayed gastric emptying (ie, gastroparesis), manifesting with abdominal pain, early satiety, and postprandial emesis.

A 3-year-old boy is brought to the emergency department after falling off a jungle gym. The boy tried to catch himself and landed on his outstretched arms. He did not lose consciousness and has been crying inconsolably since he fell. Pulse is 140/min; all other vital signs are within normal limits. Physical examination reveals tenderness and swelling over the anterior portion of his right shoulder with crepitus over the swollen area. An x-ray of his right shoulder is shown. Tension from which of the following muscles is most likely responsible for displacement of the distal bone fragment?

Deltoid - Because of their attachments on the clavicle, numerous muscles can apply tension on the proximal or distal bone fragments of a clavicle fracture. The anterior portion of the deltoid muscle originates on the distal clavicle and inserts onto the humerus. Normally, this part of the muscle acts to abduct, flex, and medially rotate the humerus. However, in a distal clavicle fracture, deltoid muscle tone and the weight of the upper extremity act to pull the distal fragment in the inferolateral direction. The proximal segment of the bone is pulled in the superomedial direction by the sternocleidomastoid and trapezius muscles. The net effect is to displace the fracture, which increases the risk of nonunion.

A 52-year-old woman comes to the office due to a 2-month history of oral lesions that cause pain with chewing and swallowing food. The patient did not seek treatment because she thought the lesions would go away on their own, but they have persisted. On examination, there are erosions of the buccal and gingival mucosa, as shown. There are several flaccid bullae with erosions scattered over her trunk. The blisters spread laterally with pressure, and traction on seemingly uninvolved skin produces blistering. Autoantibodies directed against which of the following structures are most likely responsible for this process?

Desmosomes - This patient has pemphigus vulgaris (PV), an autoimmune bullous disease characterized by painful, flaccid bullae and erosions of the skin and mucosal membranes. Any mucosal site can be involved, but the oral mucosa is the most common. The bullae are structurally weak and often rupture prior to presentation, with only erosions remaining. The bullae spread laterally when pressure is applied on top (ie, Asboe-Hansen sign), and new blisters may form with gentle traction or rubbing (ie, Nikolsky sign). The underlying pathophysiology of PV involves autoantibodies directed against desmosomes, specifically desmogleins 1 and 3, which disrupt cohesion of keratinocytes. Biopsy of an active PV lesion will show intraepithelial cleavage with detached keratinocytes (acantholysis), retained keratinocytes along the basement membrane and an eosinophilic inflammatory infiltrate. Direct immunofluorescence of unaffected skin adjacent to PV lesions will show epidermal intercellular IgG and C3 deposits. Circulating autoantibodies against desmogleins are present in many patients and can aid diagnosis.

A 68-year-old man comes to the office due to severe fatigue for the last few months. He has also had 6.8-kg (15-lb) of unintentional weight loss over the same period. The patient has no significant medical history and has not seen a physician in many years. He takes no medications and does not use tobacco, alcohol, or illicit drugs. Physical examination shows mucosal pallor with no scleral icterus. The lungs are clear on auscultation, and heart sounds are normal. Abdominal examination shows mild hepatomegaly and a markedly enlarged spleen. Stool guaiac testing is negative. Laboratory results show pancytopenia, and peripheral blood smear is shown.Bone marrow aspiration is attempted but yields no marrow, and a bone marrow biopsy is subsequently performed. Which of the following findings is most likely to be observed in this patient's bone marrow?

Diffusely fibrotic marrow with clusters of megakaryocytes -This patient with pancytopenia, hepatosplenomegaly, and teardrop cells on peripheral blood smear likely has primary myelofibrosis, a hematopoietic stem cell malignancy associated with the clonal expansion of megakaryocytes. Neoplastic megakaryocytes secrete the cytokine transforming growth factor-beta, which stimulates bone marrow fibroblasts to fill the medullary space with collagen. Subsequent bone marrow fibrosis usually leads to extramedullary hematopoiesis, cytopenias, and darcrocytes. The diagnosis of primary myelofibrosis requires bone marrow examination. Bone marrow aspiration frequently results in a dry tap (no marrow) due to significant bone marrow fibrosis. Therefore, bone marrow biopsy is usually required; the presence of a diffusely fibrotic marrow with occasional clusters of atypical megakaryocytes confirms the diagnosis.

A 28-year-old woman comes to the office due to vaginal spotting. The patient normally has regular monthly menses with 3-4 days of moderate bleeding. However, she has had spotting for the past 2 days, and her last menstrual period was 6 weeks ago. Vital signs are normal. Serum quantitative β-hCG level is 800 IU/mL (normal at 6 weeks gestation: 1,080-56,500), and a repeat level performed 2 days later shows an abnormal rise to 900 IU/mL. Pelvic ultrasound reveals no gestational sac in the uterus. If uterine curettage is performed, which of the following microscopic findings would most likely be found in this patient?

Dilated, coiled endometrial glands and edematous stroma - This patient with vaginal bleeding, positive β-hCG, and no visible intrauterine gestational sac has a pregnancy of unknown location, which is concerning for an ectopic pregnancy (ie, embryo implantation in an extrauterine location such as the fallopian tube). Early ectopic pregnancy may not be visible on ultrasound; therefore, patients require serial β-hCG measurements for diagnosis. This patient has an abnormally low β-hCG level for gestational age (eg, 800 IU/mL at 6 weeks gestation) plus an inappropriate rise in β-hCG level. In normal pregnancies, these levels double approximately every 48 hours; in cases of spontaneous abortion, levels typically decrease. Therefore, the most likely cause of this patient's low β-hCG level is ectopic pregnancy. Risk factors include pelvic inflammatory disease and prior pelvic surgery. Despite its extrauterine location, an ectopic pregnancy changes the uterine endometrium due to the secretion of β-hCG, which signals the ovarian corpus luteum to continue progesterone production. Progesterone promotes endometrial remodeling and decidualization, which normally optimizes the intrauterine environment for pregnancy. Therefore, if uterine curettage is performed on this patient, it would likely show dilated, coiled endometrial glands and vascularized, edematous stroma (ie, decidualization) but no products of conception (eg, villi).

A 44-year-old man reports exertional shortness of breath and palpitations. On examination, he has a systolic murmur at the left sternal border and cardiac apex, which gets louder when he stands up. He is diagnosed with obstructive hypertrophic cardiomyopathy and paroxysmal atrial fibrillation, and is started on a medication. Changes in the action potential of ventricular muscle cells before and after administration of medication are shown. The patient is most likely being treated with which of the following medications?

Disopyramide - The illustration shows predominant slowing of phase 0 depolarization and prolongation of phase 3 repolarization (red curve) of ventricular action potential after administration of the drug. This action is usually seen with the use of class IA antiarrhythmics drugs (quinidine, procainamide, and disopyramide). Class I antiarrhythmic drugs block sodium channels and inhibit initial phase 0 depolarization of action potential, thereby slowing conduction. Class IA agents also have moderate potassium channel-blocking activity, leading to a slow rate of repolarization and prolongation of action potential duration in the cardiac myocytes

The antiviral activity of a guanosine derivative is tested by examining cells infected with different viruses for characteristic abnormal changes in cell morphology due to the viruses. These changes are known as the cytopathic effect (CPE). In the presence of significant antiviral activity, CPE will be inhibited. In the experiment CPE is not present in HSV1, HSV2, and Varicella zoster, but is still present in Epstein-Barr and CMV. A difference in which of the following best explains the variation in susceptibility of the various viruses to this specific antiviral agent?

Drug phosphorylation rate - The drug described in this study demonstrates significant antiviral activity (cytopathic effect) against herpes simplex virus type 1 (HSV 1), HSV 2, and varicella zoster virus (VZV) but has weak activity against Epstein-Barr virus (EBV) and cytomegalovirus (CMV). This agent is most likely acyclovir, a guanosine analog. Once acyclovir enters the HSV-infected host cell, it is phosphorylated to acyclovir monophosphate, principally via a virally-encoded thymidine kinase (TK). This is the rate-limiting step in acyclovir activation. Acyclovir monophosphate is then phosphorylated by cellular enzymes into the active triphosphate form, which impairs viral DNA polymerase-mediated replication of HSV. EBV and CMV do not produce the same TK as HSV and VZV. As a result, EBV- or CMV-infected cells cannot easily convert acyclovir into its pharmacologically active form. At very high intracellular concentrations of acyclovir, some of the drug appears to be activated by unidentified cellular phosphorylating enzymes, which may explain the partial susceptibility of EBV and CMV to higher acyclovir doses. The partial activity of acyclovir against CMV may also be mediated by inhibited synthesis of a virus-specific polypeptide.

A 30-year-old nulliparous woman comes to the office for evaluation of infertility. The patient has been attempting to conceive for the past 2 years. Menarche was at age 11, and her menstrual cycles occur 2-3 times per year and last 7-10 days. The patient does not use tobacco, alcohol, or illicit drugs. Blood pressure is 122/84 mm Hg and pulse is 84/min. BMI is 35 kg/m2. Physical examination shows mild acne and hair growth on the upper lip and chin. This patient is at greatest risk for which of the following complications?

Endometrial carcinoma - This patient with oligomenorrhea, signs of hyperandrogenism (eg, acne, hirsutism), and infertility likely has polycystic ovary syndrome (PCOS). PCOS is characterized by increased ovarian androgen production, which acts on the hypothalamic-pituitary-ovarian (HPO) axis to cause an abnormally elevated LH/FSH ratio that inhibits ovulation. Therefore, a common presentation is irregular menses with anovulatory infertility. Obesity, which is common with PCOS, can also contribute to the anovulatory state by inducing hyperinsulinism, which acts synergistically in the ovary with LH to upregulate androgen production. In addition to increased androgen levels, PCOS causes increased estrogen levels due to androgen-to-estrogen conversion by the enzyme aromatase within the ovary (eg, estradiol) and adipose tissue (eg, estrone). Estrogen normally stimulates endometrial proliferation to allow for implantation. Progesterone, which is released after ovulation, opposes endometrial proliferation by inducing cellular differentiation, protecting against endometrial hyperplasia. Women with PCOS have disproportionately high estrogen production but low progesterone release (ie, unopposed estrogen exposure) due to chronic anovulation, resulting in unregulated, increasingly disordered endometrial proliferation and increased risk for endometrial hyperplasia and carcinoma.

A 52-year-old woman with a history of cirrhosis due to chronic hepatitis C experiences worsening dyspnea. She has no other medical conditions. Vital signs are within normal limits. Physical examination shows normal jugular venous pressure, right-sided dullness to percussion and decreased breath sounds, normal heart sounds, and moderate-sized ascites. Chest x-ray reveals a large right-sided pleural effusion with no parenchymal lesions. Thoracentesis yields transudative fluid with normal cell counts. Which of the following is the most likely underlying mechanism of this patient's pulmonary findings?

Fluid movement across the diaphragm - This patient with cirrhosis complicated by ascites who presents with a right-sided, transudative pleural effusion most likely has a hepatic hydrothorax. As ascites fluid accumulates in the abdomen, increased intraabdominal pressure can force fluid into the chest cavity through small fenestrations in the diaphragm; fluid movement across the diaphragm occurs most often on the right side. Because the development of ascites in patients with cirrhosis is driven by changes in hydrostatic and oncotic pressure (ie, increased hydrostatic pressure due to portal venous hypertension and decreased oncotic pressure due to hypoalbuminemia), the pleural fluid in hepatic hydrothorax is transudative by Light criteria.

A 43-year-old woman comes to the office due to occasional chest discomfort over the past year. She describes the pain as a midline pressure or squeezing sensation that lasts 10-15 minutes and is sometimes accompanied by sweating. The patient has no history of hypertension or diabetes mellitus but is a smoker. Ambulatory ECG monitoring shows transient ST-segment elevations in the anteroseptal leads during an episode of chest pain at night. Coronary angiography reveals no atherosclerotic stenosis, but the administration of acetylcholine elicits similar chest pain and ECG changes. Which of the following best explains the coronary intervention findings in this patient?

Endothelial dysfunction - This patient with episodes of recurrent chest discomfort that spontaneously resolve within 10-15 minutes most likely has vasospastic angina (formerly called Prinzmetal angina). It results from coronary endothelial dysfunction and autonomic imbalance that lead to intermittent coronary vasospasm and myocardial ischemia. Affected patients are usually young (eg, age <50) and lack typical risk factors for coronary artery disease (CAD) (eg, hypertension, diabetes mellitus); however, smoking is a strong risk factor, likely because it contributes to endothelial dysfunction. Symptoms of vasospastic angina are thought to be triggered by excess vagal tone, and they occur most commonly at night when vagal tone is at a peak. The diagnosis is made by ambulatory ECG demonstrating ST-segment elevation during an episode of chest discomfort and coronary angiography revealing an absence of obstructive atherosclerotic CAD. Because of the pathophysiology of the disorder, both acetylcholine and ergot alkaloids (eg, dihydroergotamine) provoke symptoms.

A 52-year-old woman comes to the emergency department due to increasing dyspnea and chest tightness over the past 2 weeks. She has a history of stage 4 chronic kidney disease due to chronic glomerulonephritis. Temperature is 36.6 C (97.8 F), blood pressure is 106/82 mm Hg, and pulse is 98/min. Systolic blood pressure decreases by 12 mm Hg during inspiration. Physical examination shows distant heart sounds and clear lungs. Echocardiography in this patient is most likely to reveal which of the following findings?

Engorged inferior vena cava - This patient with progressive dyspnea and chest tightness over a period of 2 weeks and a drop in systolic blood pressure >10 mm Hg with inspiration (ie, pulsus paradoxus) likely has subacute cardiac tamponade; uremia in the setting of advanced renal failure can lead to pericarditis with associated pericardial effusion and sometimes tamponade. Tamponade classically presents with Beck's triad of hypotension (often less prominent with subacute compared to acute tamponade), distant heart sounds, and jugular venous distension. Normally, a pressure gradient exists between the vena cava and the right side of the heart, allowing blood to fill the right atrium during diastole. With inspiration, a reduction in intrathoracic pressure pulls additional blood into the right side of the heart and the pulmonary circulation, causing normal inspiratory collapse of the inferior vena cava (IVC). With cardiac tamponade, increasing pericardial pressure begins to compress the relatively low-pressure, right-sided heart chambers and restrict diastolic filling. The increased right-sided pressures are transmitted backward to the vena cava, causing engorgement of the IVC with decreased inspiratory collapse that can be seen on echocardiography. In addition, because the right ventricle is compressed and unable to expand during diastole, the increase in right-sided blood flow during inspiration causes the ventricular septum to bow into the left ventricle; this reduces left ventricular stroke volume during inspiration and is responsible for the observed >10 mm Hg drop in systolic blood pressure. As tamponade progresses, impaired right-sided filling causes the left ventricle to receive less blood, resulting in decreased cardiac output and obstructive shock. Because the obstruction to blood flow is primarily right sided, there is no pulmonary edema and the lungs remain clear to auscultation.

A 47-year-old man comes to the office due to dysphagia. The patient says that it feels like food has been getting stuck in his throat over the last month. He has changed his diet to accommodate his symptoms and now consumes only liquid foods. The patient has a long history of postprandial heartburn that sometimes awakens him from sleep. He has treated his heartburn with over-the-counter antacids but usually only has partial relief. A thoracic CT image near the level of the aortic arch is shown below. This patient's symptoms are most likely related to pathology involving which of the following structures?

Esophagus - This patient's prolonged history of heartburn is suggestive of gastroesophageal reflux disease (GERD), a condition that affects the esophagus. Dysphagia (ie, difficulty swallowing) is common in patients with longstanding GERD and can be caused by impaired peristalsis, esophageal inflammation/stricture, or malignancy. In the thorax, the esophagus courses between the trachea and vertebral bodies and is typically collapsed with no visible lumen. The trachea serves as a good landmark because the radiolucency of the air in this structure allows one to differentiate it from the esophagus posteriorly and the great vessels anteriorly

A 6-month-old boy is brought to the urology clinic for follow-up of an undescended right testicle discovered during evaluation in the newborn nursery. Physical examination shows absence of a palpable right testis in the scrotal sac. However, a round mass is palpated superior to the scrotum in the inguinal canal. Orchiopexy, the placement and fixation of the testis in the scrotum, is recommended to the family. During this patient's procedure, the malpositioned testis will most likely be pulled through a physiologic opening in which of the following structures?

External oblique muscle aponeurosis - Testicles develop in the fetal abdomen during organogenesis. Between 8 weeks and full term, each testis descends from the abdomen into the inguinal canal through the deep inguinal ring, which is an opening in the transversalis fascia bounded laterally by the transversus abdominis muscle and medially by the inferior epigastric vessels. Each testis then passes anteromedially through the canal and enters the scrotum via the superficial inguinal ring, which is a physiologic opening in the external oblique muscle aponeurosis above the pubic tubercle. Cryptorchidism is the failure of one or both testes to descend to the scrotum before birth, which occurs more commonly in preterm neonates. It is associated with a significantly increased risk of testicular cancer and infertility. A testis that is palpable in the inguinal canal typically descends spontaneously by age 6 months. Testes that have not descended by this point are unlikely to do so and require surgical intervention (ie, orchiopexy). In this case, the patient's undescended testicle is lodged within the inguinal canal and must be mobilized through the superficial inguinal ring and stitched into place in the scrotum.

A 68-year-old man comes to the emergency department due to cough, breathlessness, and chest pain. The patient has a medical history of hypertension and type 2 diabetes mellitus. He is a former smoker with a 20-pack-year history. Temperature is 38 C (100.4 F), blood pressure is 130/80 mm Hg, respirations are 22/min, and oxygen saturation is 95% while breathing ambient air. The trachea is central in position. There is dullness to percussion over the lower right lung area compared to the resonancy on the left. Auscultation of the right lower lobe reveals breath sounds that are loud, hollow, and high-pitched. When the patient recites the word "ninety-nine" in a normal voice, the sound has higher clarity and intensity over the right base compared to other lung areas. Which of the following is the most likely cause of these findings?

Exudation of neutrophil-rich fluid in the lung parenchyma - This patient's presentation is consistent with bacterial pneumonia. The infection causes the alveoli to fill with inflammatory, neutrophil-rich exudate (pus); alveolar filling with any kind of fluid (eg, pus, edema, blood) creates alveolar consolidation. Alveolar consolidation leads to a number of characteristic physical examination findings. Sound travels faster and more efficiently through liquids than gases; therefore, fluid-filled alveoli transmit higher intensity sound than those filled with air. As a result, breath sounds and tactile fremitus (vibration) are more prominent over areas of alveolar consolidation. Bronchophony, a phenomenon in which a patient's spoken words (eg, "ninety-nine," "toy boat") are heard more loudly and clearly when auscultated over an area of consolidation compared to normal lung, is also present. The higher density of fluid-filled alveoli also creates relative dullness to percussion compared to the resonant sound created on percussion of air-filled alveoli.

A 23-year-old woman is evaluated due to 10 days of nonproductive cough, low-grade fever, headache, and malaise. The patient has no other medical problems and takes no medications. Lung examination reveals scattered rales. Chest x-ray reveals bilateral patchy areas of consolidation. She has mild anemia and an elevated serum lactate dehydrogenase level. The patient is treated for presumed Mycoplasma pneumonia with azithromycin. Two months later, her symptoms and the anemia have resolved. Which of the following best explains the resolution of this patient's anemia?

Fading of immune response against the bacteria - Mycoplasma pneumoniae is a common cause of tracheobronchitis and walking pneumonia. It attacks the respiratory epithelium by binding to an oligosaccharide (I-antigen) that is also present on the surface of erythrocytes. This leads to the formation of cross-reacting IgM antibodies that attach to red blood cells, activate the complement system, and cause erythrocyte lysis. The cross-reacting antibodies are called cold agglutinins because they bind to erythrocytes in areas where the blood temperature is below core body temperature (eg, distal extremities, nose). Most patients with cold agglutinins are asymptomatic, but some develop manifestations of intravascular hemolytic anemia such as elevated reticulocyte count and lactate dehydrogenase level. IgM titers usually begin to fall approximately 4 weeks after initial infection leading to a resolution of the hemolytic anemia (within 8 weeks).

A 32-year-old man is evaluated in the emergency department due to fever, night sweats, and chills over the last several days. The patient has been using intravenous drugs recently as he is "stressed out." He has otherwise been in good health with no medical problems. Temperature is 38.3 C (101 F), blood pressure is 120/80 mm Hg, and pulse is 105/min and regular. Further evaluation reveals aortic valve endocarditis with an intracardiac abscess and small fistula formation between the aortic root and right ventricle. Doppler ultrasound interrogation of the fistula will most likely reveal which of the following blood flow patterns?

Flow from the aortic root to the right ventricle continuously - Echocardiography with Doppler analysis can be used to detect and quantify shunts in patients with intracardiac fistulas. During the normal cardiac cycle, central aortic pressure (eg, 120/80 mm Hg) is higher than right ventricular pressure (eg, 25/5 mm Hg) during systole and diastole. Consequently, in patients with aortocavitary fistula, Doppler interrogation will most likely demonstrate continuous blood flow from the higher-pressure aortic root to the lower-pressure right ventricle (left-to-right cardiac shunt). This can lead to a continuous murmur heard on cardiac auscultation.

A 62-year-old man comes to the office for follow-up. The patient has a 10-year history of type 2 diabetes mellitus. Over the past 2 years, he has had burning pain and pins and needle sensations in his feet, which is worse at nighttime. Nortriptyline was prescribed, and the dosage was gradually increased. Currently, the patient takes the maximal recommended dose but states the symptoms are not well controlled. He reports no drug adverse effects. Other medical history includes hypertension and hyperlipidemia. Blood pressure is 124/70 mm Hg and pulse is 68/min. Physical examination shows decreased light touch and pain sensation in the feet. Ankle reflexes are decreased bilaterally. His most recent hemoglobin A1c level is 7.0%. Appropriate foot care is advised, and additional pharmacotherapy for the patient's symptoms is planned. Which of the following medications would provide additional benefit through a different mechanism with no drug-drug interaction?

Gabapentin - First-line therapy for painful diabetic neuropathy includes the following three classes of medications: Tricyclic antidepressants (TCAs) (eg, nortriptyline, amitriptyline) modulate pain transmission by inhibiting voltage-gated sodium channels in sensory nerves and increasing norepinephrine signaling in the central nervous system (CNS). Serotonin-norepinephrine reuptake inhibitors (SNRIs) (eg, duloxetine, venlafaxine) alter central transmission of pain by increasing norepinephrine in the central synapses. Anticonvulsants (eg, gabapentin, pregabalin) inhibit the release of excitatory neurotransmitters by binding to voltage-gated calcium modulators on nerve terminals. This patient is being treated with nortriptyline, a TCA drug. His symptoms are uncontrolled; therefore, an additional medication with a different mechanism of action should be added to avoid additive or potentially even synergistic adverse effects. Gabapentin does not inhibit serotonin and norepinephrine in the CNS but rather inhibits the release of excitatory neurotransmitters, so it is less likely to compound the adverse effects of a TCA.

A 34-year-old man comes to the office due to upper abdominal pain. The patient has vague discomfort that happens mostly in the afternoon and at night and is partially relieved by food. He sometimes feels nauseated. He has had no vomiting, black or bloody stools, or weight loss. The patient takes no prescription or over-the-counter medications. Family history is negative for cancer. Upper gastrointestinal endoscopy reveals a small ulcer with a clean base in the duodenal bulb. Biopsy of which of the following sites is most likely to demonstrate the infectious agent responsible for this patient's current condition?

Gastric antrum - Peptic (eg, duodenal, gastric, gastroesophageal) ulcers are most commonly due to Helicobacter pylori infection or use of nonsteroidal anti-inflammatory drugs. Because this patient takes no medications, his ulcer is likely due to H pylori infection. The site of ulcer formation is dependent on relative bacterial colonization density within the stomach. H pylori preferentially colonizes the gastric antrum, particularly early in the disease, although it may occur in other areas of the stomach or spread with time. Colonization of H pylori in the gastric antrum is associated with decreased somatostatin formation and increased gastrin secretion, which stimulate the parietal cells to produce excess acid. This results in an increased acid load emptying into the proximal duodenum, leading to duodenal ulcer (DU) formation. Biopsy of the antrum can confirm H pylori infection in patients with a DU, which characteristically presents with upper abdominal pain that improves with eating. In contrast, gastric ulcers (GUs) are associated with colonization in the gastric corpus (body). The etiology of GUs is not acid related, as these patients typically have normal or reduced acid levels, but is thought to be due to direct mucosal damage and chronic inflammation. Unlike DU pain, GU pain often worsens with eating.

A group of researchers investigates why individuals who have undergone gastrectomy lose more weight than can be explained by dietary changes alone. They recruit several subjects who have had distal gastrectomy for benign gastric-outlet obstruction for a clinical study. Control subjects with matching baseline weight are also enrolled and are instructed to follow a diet similar to that of the postgastrectomy subjects during the study period. Plasma levels of several hormones are serially measured over 24 hours in both groups. Levels of one of the hormones are shown with peaks around meal times. This profile most likely represents which of the following hormones?

Ghrelin - The 24-hour hormone levels in the control subjects show increasing secretion leading up to meals, followed by a sharp postprandial drop. However, the subjects who have undergone gastrectomy show lower baseline levels with attenuation of meal-related fluctuations. This pattern suggests that the hormone is likely secreted in the stomach and suppressed by food intake, which is consistent with the secretory pattern of ghrelin. Ghrelin is produced primarily in the stomach in response to fasting; levels surge leading up to meals and fall after eating. Ghrelin stimulates appetite and promotes weight gain. Caloric restriction and falling fat stores lead to increased ghrelin levels (along with decreased leptin and insulin levels), which limits weight loss from dietary modification alone. However, patients who have undergone bariatric procedures that remove a portion of the stomach (eg, roux-en-Y gastric bypass, sleeve gastrectomy) can lose a significant number of ghrelin-secreting cells. This leads to lower ghrelin levels and less stimulation of appetite in response to fasting, promoting weight loss.

A 48-year-old woman is evaluated for postcoital bleeding and vaginal discharge. Pelvic examination shows a friable mass at the cervix that bleeds easily on touch. Cervical biopsy confirms invasive squamous cell cancer confined to the cervix and uterus. Lymph node metastases are not seen. A radical hysterectomy is performed during which the right ureter is accidentally injured but then repaired. Imaging studies performed after the surgery show a partial obstruction of the right ureter with mild dilation of the proximal collecting system. Which of the following changes are most likely to be seen in the right kidney?

Glomerular filtration rate ↓ Filtration fraction ↓ The glomerular filtration rate (GFR) depends on the interplay of hydrostatic and oncotic pressures in the glomerular capillaries and Bowman's space. The GFR increases with higher glomerular hydrostatic pressure and decreases with increasing Bowman's capsule hydrostatic pressure or higher glomerular capillary oncotic pressure. Acute ureteral obstruction increases hydrostatic pressure proximal to the constriction. This pressure rise is transmitted back to the Bowman's space, resulting in decreased GFR. The filtration fraction (FF) is the portion of the renal plasma flow (RPF) that is filtered from the glomerular capillaries into Bowman's space (ie, the GFR:RPF ratio). With acute ureteral obstruction (first 12 hours), the RPF may transiently increase; however, with time, efferent arteriolar constriction (in response to reduced GFR) will decrease RPF. Even at later stages, though, the GFR remains depressed to a greater extent than the RPF, resulting in reduced FF.

A 27-year-old man is brought to the emergency department due to confusion. His roommate says that he has been binge drinking for the last 5 days and probably has had very little to eat. The patient's medical history is significant for alcohol-related seizures 1 year ago. He had been sober until 2 weeks ago, when he started drinking again. The patient's past medical history is otherwise unremarkable. On examination, he responds to voice but does not follow commands. Fingerstick glucose is 35 mg/dL and urine is strongly positive for ketones. Suppression of which of the following is the primary cause of this patient's hypoglycemia?

Gluconeogenesis - The metabolism of ethanol by alcohol dehydrogenase and aldehyde dehydrogenase reduces NAD+ to NADH and increases the NADH/NAD+ ratio. This inhibits all other pathways requiring NAD+, including reactions required for gluconeogenesis. In particular, lactate cannot be converted to pyruvate, and instead the reaction is driven from pyruvate toward lactate. In addition, excess NADH inhibits the conversion of malate to oxaloacetate. Pyruvate and oxaloacetate are intermediates in gluconeogenesis; therefore, conversion of these molecules to lactate and malate inhibits gluconeogenesis. Alcohol does not inhibit glycogenolysis, and so in the initial phase of binge drinking (heavy alcohol intake with reduced nutritional carbohydrate intake), hepatic glycogenolysis is able to maintain euglycemia. However, after a prolonged binge, hepatic glycogen is eventually depleted and blood glucose levels drop.

A 23-year-old female is being evaluated for infertility. Hormone studies reveal a normal rise in the blood estradiol level during the first phase of the menstrual cycle. Androgens synthesized in this patient's ovaries are converted to estradiol in which of the following cell types?

Granulosa - Estradiol, the predominant estrogen in the human body, is primarily derived from androgens. Androgens and progesterone are synthesized from cholesterol in the theca interna cells under the influence of luteinizing hormone (LH). Androgens released from the theca interna cells migrate into the nearby granulosa cells, which contain the enzyme aromatase. Aromatase converts the androgens into estradiol. This reaction is stimulated by follicle stimulating hormone (FSH).

Pharmacology researchers develop a novel monoclonal antibody medication to treat the wet form of age-related macular degeneration. The antibody binds vascular endothelial growth factor, decreasing abnormal blood vessel formation in the subretinal space. In a clinical trial, the medication is found to improve visual function. During the next phase of the study, researchers use only the antigen binding fragment (Fab) of the antibody instead of the whole immunoglobulin. Which of the following is most likely to be observed with use of the antibody fragments compared to the intact immunoglobulin?

Greater tissue penetration of the drug - Immunotherapy is monoclonal because all the immunoglobulin components in the medication have the same hypervariable region (produced from the same B-cell clone). However, most immunotherapy regimens use a fragment of the immunoglobulin with 1 valence (binding) site rather than the full immunoglobulin with 2 valence sites, because fragments are significantly smaller than the full immunoglobulin, which improves tissue/tumor penetration and medication pharmacokinetics.

A 56-year-old man comes to the emergency department with acute onset of severe right foot pain. The pain is associated with local redness and swelling, but he has had no fever or recent trauma. The patient has never had similar symptoms. Medical history is significant for type 2 diabetes mellitus, mixed hyperlipidemia, hypertension, and coronary artery disease, for which he takes several medications. Physical examination shows a swollen, tender first metatarsophalangeal joint. Aspiration reveals a high leukocyte count, negative Gram stain, and numerous needle-shaped negatively birefringent crystals. Which of the following drugs is most likely to have precipitated this patient's condition?

Hydrochlorothiazide - This patient has an acute inflammatory arthritis of the first metatarsophalangeal joint with needle-shaped, negatively birefringent crystals noted on joint fluid microscopy. This presentation is consistent with an acute gout flare, which occurs due to the deposition of monosodium urate crystals within joint spaces (most commonly in the lower extremities). Gout occurs in the setting of hyperuricemia, which may occur due to increased uric acid production (eg, dietary sources of purine) or reduced uric acid clearance. Most diuretics (eg, hydrochlorothiazide, furosemide) can cause hyperuricemia and trigger gout flares by causing relative volume depletion, which decreases the fractional excretion of uric acid. Other medications that can cause hyperuricemia include certain immunosuppressants (eg, cyclosporine, tacrolimus) and cytotoxic chemotherapeutic agents. In addition, medications that rapidly lower uric acid levels (eg, allopurinol) can paradoxically trigger an acute attack of gout due to mobilization of tissue urate stores (ie, dissolving crystals become fragile and more easily shed into the joint space).

A 1-month-old boy is brought to the office on a hot summer day due to vomiting and excessive sleepiness. He had been exclusively breastfeeding with normal feeding patterns until this afternoon, when he had 2 episodes of emesis and appeared much sleepier than usual. The infant was born at 41 weeks gestation after an unremarkable pregnancy. Newborn screening was positive for a homozygous ΔF508 mutation affecting an epithelial transmembrane protein; further confirmatory testing is pending. Physical examination shows a somnolent boy with a sunken fontanelle and dry mucous membranes. Which of the following electrolyte disturbances is most likely responsible for this patient's current symptoms?

Hyponatremia - The ΔF508 mutation in the cystic fibrosis transmembrane conductance regulator (CFTR) protein is the most common mutation in patients with cystic fibrosis (CF). In unaffected individuals, the CFTR protein serves as a chloride channel that regulates the flow of sodium, chloride, and water across the epithelial membranes of the airways, biliary tree, intestines, sweat ducts, and pancreatic ducts. The most likely cause of this patient's symptoms (lethargy, vomiting) is hyponatremia due to excessive salt wasting from his sweat. Risk factors for hyponatremia in patients with CF include exclusive breast or formula feeding prior to the introduction of sodium-rich, solid foods; exposure to high-temperature environments; and exercise. Therefore, salt supplementation is recommended for patients with CF.

A 34-year-old man develops excessive thirst and frequent urination while being hospitalized for multiple injuries following a motorcycle collision. His urinary osmolality after 4 hours of water restriction is 250 mOsm/L and increases to 700 mOsm/L after administration of antidiuretic hormone. The patient is started on an appropriate medication, and his symptoms promptly resolve. Over the next 2 years, multiple attempts are made to wean the patient off the medication, but his symptoms would always return, necessitating the resumption of treatment. The permanence of this patient's condition is best explained by traumatic damage to which of the following structures?

Hypothalamic nuclei - This patient has diabetes insipidus (DI), a condition that results in the production of large volumes of very dilute urine due to impaired antidiuretic hormone (ADH) activity. DI can be due to deficient ADH production in the brain (central) or resistance to ADH in the kidneys (nephrogenic). Injection of exogenous ADH can help distinguish between central and nephrogenic DI: ADH administration substantially increases urine osmolality in patients with central DI, whereas patients with nephrogenic DI typically have a more limited increase in urine osmolality (<300 mOsm/L increase). ADH is synthesized by magnocellular neurosecretory cells in the paraventricular and supraoptic nuclei of the hypothalamus. These neurons project axons to the posterior pituitary where axon terminals store large quantities of ADH for later release into the circulation. Injury to the hypothalamus or posterior pituitary (eg, head trauma, transsphenoidal neurosurgery, suprasellar masses) can result in central DI. Damage to the more distal portions of the hypothalamic-hypophyseal tract (ie, below the infundibulum) typically causes transient DI because the cell bodies of the magnocellular neurons remain intact. However, hypothalamic injury results in death of the magnocellular neurons, causing permanent central DI.

A gastroenterologist notices one of her patients at the hospital cafeteria while waiting for her friend, a hospitalist, to meet her for lunch. The patient greets the gastroenterologist and has a brief conversation before leaving. The hospitalist arrives to the cafeteria and says, "I saw you talking to my old neighbor. He's a close family friend, and I heard that he has colon cancer. I've been really worried about him, especially since his wife had a stroke recently. Are you treating him?" Which of the following is the most appropriate response by the gastroenterologist?

I cannot say whether or not he is my patient - Confidentiality is a basic tenet of medical ethics and a prerequisite for a trusting physician-patient relationship. Physicians are ethically obligated to protect patient privacy and maintain confidentiality in most situations, including during interactions with other physicians who are not directly involved in the patient's medical care. The hospitalist has asked the gastroenterologist whether a certain man, a former neighbor and close family friend, is being treated by the gastroenterologist. Because the inquiry is based on personal and not professional interest or medical necessity, the most appropriate response by the gastroenterologist is to avoid divulging the man's protected health information, such as his diagnosis, treatment, and whether he is, in fact, the gastroenterologist's patient (Choice A). Identifying the man as a patient and admitting to treating him could indirectly confirm a diagnosis of colon cancer, which is a breach of patient privacy because sharing this information is not medically necessary. Therefore, the most appropriate response is for the physician to state that they cannot say whether or not the patient is under their care.

A 24-year-old, previously healthy woman is evaluated for skin rash, joint pains, and renal failure. She is found to have decreased C3 and C4 levels and a normal factor B level. Which of the following most likely triggered the complement system activation in this patient?

IgG-antigen complex formation - The complement cascade is an ancient proteolytic defense mechanism that plays a major role in both the innate and adaptive immune responses. It is activated by 3 major inciting events, all of which terminate in the generation of C3 convertase as follows, antibody-antigen binding, lectin pattern recognition receptor binding, and C3b binding. The most common cause of complement deficiency is autoantibodies, which activate the classical complement system after binding host antigens. Classical complement pathway activation is marked by low C4 and C3 levels and normal factor B levels; CH50, a measure of functional activity of the entire classical pathway (eg, sufficient C1-C9), will also be low. This pattern is common in systemic lupus erythematosus, particularly in the setting of active renal, skin, and joint disease.

A 45-year-old woman comes to the emergency department due to sudden-onset, severe headache. On arrival, the patient is alert and oriented and has no focal neurological deficits. CT scan of the head reveals subarachnoid hemorrhage, and she is admitted to the critical care unit. Over the next several hours, the patient gradually becomes somnolent and less responsive. She opens her eyes to painful stimuli only and does not follow commands. Repeat imaging of the brain shows no new hemorrhage, but there is enlargement of the entire ventricular system compared to the previous CT scan. Which of the following is the most likely cause of this patient's neurologic deterioration in the hospital?

Impaired absorption of cerebrospinal fluid - This patient with a worsening neurologic status has enlargement of the ventricles on CT scan, consistent with hydrocephalus, a common complication of subarachnoid hemorrhage (SAH). Blood in the subarachnoid space can acutely obstruct and impair absorption of cerebral spinal fluid (CSF) by the arachnoid granulations. Later, blood-induced inflammation can cause fibrosis of the arachnoid granulations that perpetuates the absorption impairment and leads to chronic hydrocephalus. Impaired CSF absorption creates a communicating hydrocephalus. Because the blockage to flow is distal to the entire ventricular system, enlargement of all 4 ventricles occurs. The ventricular enlargement stretches adjacent nerve fibers and, in the acute setting, leads to deteriorating mental status (eg, somnolence, unresponsiveness). Treatment involves the placement of an external ventricular drain to relieve pressure in the ventricular system and subarachnoid space.

A 63-year-old man comes to the emergency department due to increasing dyspnea. He was unable to mow his lawn the previous day due to shortness of breath. The patient has a history of hypertension and a 20-pack-year smoking history. He is retired and traveled to Hawaii 2 months ago. Blood pressure is 160/86 mm Hg, pulse is 94/min, and respirations are 24/min; oxygen saturation is 94% on room air. Physical examination reveals bilateral lung crackles and occasional wheezes. Laboratory testing reveals elevated brain natriuretic peptide levels. Which of the following is the primary trigger for the release of this substance in this patient?

Increase in ventricular wall stress - In addition to their contractile and electrical conduction properties, cardiomyocytes possess endocrine capabilities to help regulate blood volume. Volume overload leads to stretching and increased wall stress of the ventricular walls. In response, the ventricular myocardium releases brain natriuretic peptide (BNP); similarly, the atrial myocardium releases atrial natriuretic peptide (ANP) in response to atrial stretching. These natriuretic peptides stimulate both venous and arterial vasodilation to decrease cardiac preload and afterload and reduce strain on the myocardium. In addition, ANP and BNP stimulate salt and water excretion by the kidneys to facilitate diuresis. Clinically, BNP has diagnostic utility in patients with shortness of breath of unknown cause. For patients in whom the serum BNP level is not elevated, shortness of breath is unlikely to be due to heart failure.

A 67-year-old man is evaluated for persistent lower back pain and fatigue. The patient has no other medical conditions and takes no medications. Laboratory testing shows anemia and renal dysfunction. Serum total protein levels are elevated, and electrophoresis of the serum proteins reveals a narrow spike in the gamma globulin region. A subsequent bone marrow biopsy shows abnormal marrow cells. A medication that blocks cellular proteasome action is administered. This treatment is most likely to cause which of the following effects on the abnormal cells?

Increased apoptosis - This patient's manifestations (anemia, back pain, renal insufficiency), monoclonal protein spike on electrophoresis, and bone marrow findings indicate likely multiple myeloma (MM), a plasma cell malignancy associated with the overproduction of monoclonal immunoglobulins or IgG fragments (eg, IgG light chains). Treatment regimens for MM often incorporate medications that block proteasome function (eg, bortezomib). The proteasome is a barrel-shaped structure with a proteolytic core that degrades ubiquitin-tagged proteins. Proteasome inhibition results in the accumulation of damaged and misfolded proteins that have a cytotoxic effect. MM cells are particularly susceptible to proteasome inhibition because they generate extremely high quantities of secretory proteins (eg, monoclonal immunoglobulins), some of which misfold and aggregate within the endoplasmic reticulum. Increased stress on the endoplasmic reticulum triggers an unfolded protein response that eventually leads to activation of caspases and cellular apoptosis.

A 52-year-old man comes to the office for a follow-up appointment. Four years ago, the patient experienced a small intracerebral hemorrhage due to uncontrolled hypertension; he had multiple seizures after the stroke, and treatment with phenytoin was initiated. The seizures are well controlled with the medication, but attempts at tapering therapy have led to recurrent seizures. The patient has mild residual weakness of the left side but can perform daily activities without difficulty. He is adherent with antiepileptic and antihypertensive treatments. Vital signs are within normal limits and physical examination is unremarkable except mild left hemiparesis. Laboratory studies reveal a decreased serum level of 25-hydroxyvitamin D. Which of the following effects of the antiseizure medication on vitamin D metabolism best explains this patient's laboratory findings?

Increased catabolism by hepatic microsomal enzymes - Several antiepileptic medications (eg, phenytoin, carbamazepine, phenobarbital) are strong inducers of cytochrome P-450 (CYP) enzymes. This effect is responsible for notable drug interactions due to increased clearance of certain pharmaceutical agents (eg, warfarin). In addition, these medications, especially phenytoin, induce CYP24 (ie, 25-hydroxyvitamin D3-24-hydroxylase), which converts 25-hydroxyvitamin D to 24,25-dihydroxyvitamin D (inactive form). As a result, there is less 25-hydroxyvitamin D available for conversion to 1,25-dihydroxyvitamin D (active form). Deficiency of 1,25-dihydroxyvitamin D leads to decreased intestinal absorption of calcium. The resulting compensatory increase in parathyroid hormone secretion causes increased release of calcium from bones and progressive loss of bone mineral density.

A 33-year-old woman who recently immigrated to the United States is brought to the emergency department with severe shortness of breath and hemoptysis. Physical examination reveals a diastolic murmur. Chest x-ray shows severe pulmonary vascular congestion and edema. She is admitted to the hospital, treated with diuretics, and begins to feel better. However, during her hospitalization, she develops right-sided hemiparesis. Which of the following additional findings in this patient would be most suggestive of combined disease involving the mitral and aortic valves rather than exclusive mitral involvement?

Increased left ventricular diastolic pressure - This patient's physical examination findings are suggestive of mitral stenosis (MS). In isolated MS, cardiac and pulmonary pressures proximal to the stenotic mitral valve are markedly elevated. However, diastolic pressure in the left ventricle (LV) is usually near normal or even decreased with severe stenosis. Increased LV diastolic pressure in a patient with MS suggests concomitant aortic valve dysfunction. Rheumatic heart disease almost always affects the mitral valve, but both the mitral and aortic valves are affected in about 25% of cases. Aortic valve involvement typically involves combined aortic stenosis and regurgitation, both of which can increase LV diastolic pressure. Rheumatic deformation of the aortic valve can also predispose to infective endocarditis even when the valve is functionally normal. This patient's right-sided hemiparesis is most likely due to embolism, which could have originated from an atrial mural thrombus secondary to atrial dilation from MS or an endocarditis-related valvular vegetation (less likely).

A 46-year-old man comes to the emergency department due to fever. He returned from a vacation in Southeast Asia a week ago and began having symptoms toward the end of his trip. The patient describes continuous fever that has risen slowly to reach a high plateau and is relieved only by antipyretics. He has also had headaches and abdominal discomfort; he initially had constipation but has had loose stools for the past several days. Temperature is 40 C (104 F), blood pressure is 114/68 mm Hg, and pulse is 62/min. No oropharyngeal lesions, cervical lymphadenopathy, or neck rigidity is present. Lungs are clear on auscultation, and heart sounds are normal. Abdominal examination shows mild distension and generalized tenderness with hepatosplenomegaly. There is a faint, erythematous, macular skin rash over the trunk and abdomen. Laboratory studies reveal mild, normocytic, normochromic anemia and leukopenia with left shift. Peripheral blood smear is normal. Which of the following is the most likely route of transmission of this patient's infection?

Ingestion of contaminated foods - Typhoid fever is a life-threatening infection caused by Salmonella Typhi or Paratyphi. These gram-negative, enteric pathogens reside only in the gastrointestinal tract of humans (Choice B). Transmission occurs via the fecal-oral route, primarily due to ingestion of food or water contaminated by human feces. Therefore, most cases arise in the developing world where sanitation is poor. Travelers who have not been vaccinated (with typhoid vaccine) are at risk and may return clinically ill. Typhoidal Salmonella species invade enterocytes, travel from the apical to the basolateral end of the cell in vacuoles, and emerge in the intestinal submucosa. Here, the organism is phagocytosed by macrophages but is able to evade destruction due to a specialized capsular antigen (Vi) that impairs the macrophage-mediated oxidative burst. The pathogen subsequently replicates in an unchecked fashion within macrophages and spreads through the lymphatic and reticuloendothelial system, leading to widespread, systemic illness. Manifestations typically develop over weeks in a step-wise fashion. Patients usually develop a progressive fever, often with relative bradycardia (pulse-temperature dissociation), followed by abdominal pain, salmon-colored macules on the trunk ("rose spots"), and constipation or diarrhea. Dissemination through the reticuloendothelial system often causes hepatosplenomegaly, anemia, and leukopenia. Later, ulceration of Peyer patches (lymphoid aggregations in the small intestine submucosa) can result in gastrointestinal bleeding or intestinal perforation. Antibiotic treatment is required to prevent complications and death.

A 47-year-old man comes to the office due to worsening shortness of breath for several months. The patient has no fever, weight loss, or cough. He has no other medical conditions and takes no medications. The patient does not use tobacco, alcohol, or illicit drugs. Vital signs are normal. Pulmonary examination demonstrates wheezing and poor air movement. Laboratory results are as follows: Platelets 160,000/mm3 Leukocytes 6,500/mm3 Creatinine 0.8 mg/dL Total bilirubin 1.3 mg/dL Aspartate aminotransferase (SGOT) 112 U/L Alanine aminotransferase (SGPT) 124 U/L A liver biopsy is subsequently performed. Light microscopy of the tissue specimen after treatment with diastase and periodic acid-Schiff staining is shown in the exhibit. Given these findings, which of the following is the most likely cause of this patient's shortness of breath?

Interalveolar septa destruction - This patient has alpha-1 antitrypsin (AAT) deficiency, an autosomal codominant disorder that can affect the lungs and liver. Produced primarily in the liver, AAT is a serum protein that inhibits several different proteolytic enzymes (in particular, neutrophil elastase), thereby reducing tissue damage caused by inflammation. Most individuals with AAT deficiency are homozygous for the Z allele, which causes decreased secretion of AAT secondary to abnormal protein folding. The diagnosis of AAT deficiency is established by measurement of the serum AAT level, followed by confirmatory genetic testing. Most individuals with AAT deficiency eventually develop severe panacinar emphysema due to unchecked destruction of the interalveolar septa (which contain large amounts of elastin). Smoking plays a synergistic role in the disease process by inducing lung inflammation and permanently inactivating AAT through oxidation of a crucial methionine residue. Therefore, smokers with AAT deficiency tend to develop dyspnea at a median age of 36 versus a median age of 51 in nonsmokers. Some patients with AAT deficiency develop liver disease due to intrahepatocyte accumulation of polymerized AAT molecules; the liver disease can progress to cirrhosis (the second most common cause of death behind emphysema in this population) and hepatocellular carcinoma. Histologically, intracellular granules of unsecreted AAT are seen within the periportal hepatocytes. These globules resist digestion by diastase (an enzyme that breaks down glycogen) and appear reddish-pink with the periodic acid-Schiff stain.

A 3-month-old girl is brought to the emergency department due to fever, irritability, and vomiting for the past 2 days. On examination, she is ill-looking, lethargic, and febrile. Blood cultures grow Mycobacterium tuberculosis. One of her brothers died from disseminated mycobacterial infection during infancy. Impairment of which of the following protective mechanisms is most likely contributing to this patient's infection?

Interferon signaling - Host defense against mycobacterial infections depends on the ability of macrophages to sequester mycobacteria in granulomas and directly kill the bacteria in phagosomes. Infected macrophages produce IL-12, which stimulates T cells and natural killer cells to produce interferon-γ (IFN-γ). Binding of IFN-γ to its receptor on a macrophage causes transcription of IFN-γ-regulated genes through activation of the JAK-STAT signaling pathway. The result is upregulation of IL-12 production, enhanced mycobacterial intracellular killing in phagolysosomes, and production of tumor necrosis factor-α (TNF-α) which promotes formation of granulomas. Autosomal recessive deficiencies of the IFN-γ receptor (or other elements of this pathway) result in disseminated mycobacterial disease (or BCG infection if the vaccine is administered) in infancy or early childhood. These patients require lifelong treatment with continuous antimycobacterial antibiotics.

A 26-year-old man is brought to the emergency department after a high-speed motorcycle collision. The patient was thrown several feet after his motorcycle collided with a car. On arrival, he is profoundly comatose. A CT scan of the head performed within an hour of the event shows no abnormalities. The patient dies several hours later, and autopsy examination is performed. Brain histopathology shows widespread axonal swelling, predominantly at the gray-white junction. Immunohistochemical staining of these axons reveals accumulation of alpha-synuclein and amyloid precursor proteins. Which of the following is the most likely cause of these observed findings?

Interruption of the white matter tracts - This patient has histopathologic findings of diffuse axonal injury (DAI), a type of severe traumatic brain injury that can occur from direct blunt force injury (eg, head hitting windshield) or abrupt changes in acceleration-deceleration (eg, restrained passenger coming to sudden stop after a collision). Transfer of force can result in immediate shearing of the white matter tracts or induce secondary biochemical changes leading to degradation of the axonal cytoskeleton with subsequent axon breakage. Normal axonal transport is inhibited, leading to accumulation of axonally transported proteins (eg, amyloid precursor, alpha-synuclein) within axonal swellings at the point of injury (eg, axonal bulb formation). Presentation varies based on the extent of injury; most patients are comatose (Glasgow Coma Score <8); however, those with very mild DAI may have only concussive symptoms (eg, headache, amnesia). CT scan has low sensitivity for diagnosis and is often normal, but MRI may demonstrate lesions at the gray-white junction. Microscopically, DAI is visible as widespread axonal swelling, most pronounced at the gray-white matter junction. Prognosis is poor, with up to 25% of cases resulting in death.

A 45-year-old man is brought to the emergency department due to new-onset seizure. His family witnessed an episode of blank staring followed by a generalized tonic-clonic convulsion with tongue biting and urinary incontinence. The patient has a past medical history of seasonal asthma and takes no prescription medications. He does not use alcohol or illicit drugs. Vital signs and neurologic examination are normal. Noncontrast head CT and brain MRI are consistent with a diagnosis of cavernous hemangioma. This patient is at greatest risk of developing which of the following complications?

Intracerebral hemorrhage - Cavernous hemangiomas are vascular malformations (sporadic or familial) that most commonly occur within the brain parenchyma above the cerebellar tentorium. They consist of clusters of dilated, thin-walled capillaries with little or no intervening nervous tissue. Patients with cavernous hemangiomas often have seizures and may develop progressive neurologic deficits due to mass effect with interruption of the cerebral microcirculation. These lesions also have a tendency to bleed, which can contribute to neurologic symptoms (eg, due to subcortical/cortical tissue irritation) and appear as an irregular hyperdense/bright mass on head CT.

A phase II trial is being conducted at an academic medical center to assess the safety and efficacy of a novel antifungal medication that has yielded promising results in preclinical studies. The medication is known to inhibit fungal colony growth by changing the composition of the fungal cell membrane. Phase 0 and phase I trials on healthy volunteers have been uneventful. The current phase II trial includes 175 subjects. In 168 subjects, the new antifungal agent is found to inhibit liver cytochrome P450-dependent metabolism of several drugs. Based on these data, the new antifungal agent is most similar to which of the following?

Itraconazole - Azoles are antifungal medications that inhibit the demethylation of lanosterol into ergosterol in fungal cells. They exert their antifungal effects by suppressing synthesis of ergosterol, an essential component of the fungal cell membrane. However, azoles also inhibit the activity of the human P450 cytochrome oxidase system. This property leads to multiple drug-drug interactions. All azoles (eg, ketoconazole, fluconazole, itraconazole, voriconazole) increase the serum concentrations of drugs metabolized by liver P450 enzymes. This means that, when used concomitantly with azoles, drugs metabolized by the liver P450 system may have increased toxicity. For this reason, medications such as warfarin, cyclosporine, tacrolimus, and oral hypoglycemics should be avoided (or have their use carefully monitored) in patients who are receiving azole therapy. Cytochrome oxidase inducers (eg, rifampin, phenytoin, carbamazepine, phenobarbital) increase azole metabolism, thereby lowering azole concentration in serum.

A 54-year-old man comes to the emergency department due to severe fatigue and dyspnea. He has a long history of progressively worsening heart failure that has been resistant to treatment with medications, including diuretics. He was treated with chest radiation 10 years ago for non-Hodgkin lymphoma and has been in remission since then. The patient is admitted to the hospital, but his condition continues to deteriorate despite aggressive therapy. He dies 3 days later, and an autopsy is performed. Gross inspection of the heart shows dense, thick, fibrous tissue in the pericardial space between the visceral and parietal pericardium. Which of the following signs would most likely have been detected during a physical examination of this patient just prior to his death?

Kussmaul sign - The autopsy finding of thick, fibrous tissue in the pericardial space is consistent with constrictive pericarditis, a potential complication of chest radiation therapy for non-Hodgkin lymphoma. This dense, rigid pericardial tissue encases the heart and restricts ventricular filling, causing low cardiac output (manifesting with fatigue and dyspnea on exertion) and progressive right-sided heart failure (manifesting with hepatomegaly and peripheral edema). Physical examination in constrictive pericarditis typically shows elevated jugular venous pressure (JVP) with prominent x and y descents and a pericardial knock (early diastolic sound that occurs before S3) and may also demonstrate pulsus paradoxus (>10 mm Hg drop in systolic blood pressure during inspiration). In addition, Kussmaul sign may be present. Under normal circumstances, the decrease in intrathoracic pressure during inspiration increases venous return to the right side of the heart and lowers JVP. However, in constrictive pericarditis, the rigid pericardium prevents the right side of the heart from accommodating increased venous return, which leads to a paradoxical rise in JVP during inspiration, referred to as Kussmaul sign.

A 35-year-old woman is brought to the emergency department due to severe right leg pain. She is a concert pianist with no known medical issues. During rehearsal, she began experiencing cramping of the right foot that did not improve with stretching or rest. The pain quickly spread to her calf, and the patient now rates the pain as 9 out of 10 in intensity. On examination, the right foot and calf appear pale and cool compared with the left, and sensation is diminished. Right-sided dorsalis pedis and posterior tibial pulses are absent, and the popliteal pulse is barely palpable. Emergency embolectomy is performed and a gelatinous mass causing arterial occlusion is successfully removed. Histopathologic section of the mass is shown in the exhibit. Which of the following is the most likely origin of this patient's embolus?

Left atrium - This patient presented with acute limb ischemia, as evidenced by the 6 Ps: pain, pallor, poikilothermia (coolness to touch), paresthesia, paralysis, and reduced or absent pulses. The ischemia resulted from systemic embolization of a gelatinous mass, with histopathology demonstrating an amorphous extracellular matrix with scattered stellate myxoma cells in mucopolysaccharide ground substance; this is consistent with tumor fragments from an atrial myxoma. Myxomas are the most common primary cardiac neoplasm. Approximately 80% of these benign tumors originate in the left atrium. Patients can initially present with nonspecific constitutional symptoms (eg, fatigue, low-grade fever, weight loss) or systemic embolization (eg, stroke, mesenteric ischemia, acute limb ischemia), as in this patient. In addition, the tumor can cause position-dependent obstruction of the mitral valve, leading to signs and symptoms that mimic mitral valve stenosis (eg, dyspnea, cough, mid-diastolic murmur).

A 57-year-old man is brought to the emergency department following a generalized tonic-clonic seizure. His wife reports that he has no history of seizures. However, she says that he has been complaining of intermittent headaches, memory loss, and problems with his vision for the past 2 weeks. Brain imaging shows a solitary mass within the right temporal lobe. Which of the following visual field defects is most likely present in this patient?

Left homonymous superior quadrantanopia (pie in the sky) - Damage to the visual pathway produces distinct types of visual field defects depending on the location of the lesion. Visual perception begins with light from the nasal visual fields striking the temporal side of each retina and light from the temporal visual fields striking the nasal side of each retina. Information from the retina is then transmitted by the optic nerves to the optic chiasm. At the optic chiasm, optic nerve fibers from the nasal half of each retina cross and project into the contralateral optic tract. In contrast, nerve fibers from the temporal parts pass into the ipsilateral optic tract. The optic tract thus contains nerve fibers from the temporal part of the ipsilateral retina and the nasal part of the contralateral retina. Optic tract fibers project mainly to the lateral geniculate nucleus (LGN), but also project to superior colliculus (reflex gaze), pretectal area (light reflex), and the suprachiasmatic nucleus (circadian rhythms). Axons from the LGN that project to the striate (primary visual) cortex are known as the optic radiation (or geniculocalcarine tract). The lower fibers of the optic radiation carry information from the lower retina (upper contralateral visual field) and take a circuitous route anteriorly into the temporal lobe (Meyer's loop) before reaching the lingual gyrus of the striate cortex. The upper fibers of the optic radiation carry information from the upper retina (lower contralateral visual field) and pass more directly from the LGN through the parietal lobe to reach the cuneus gyrus of the striate cortex. Lesions in the temporal lobe can disrupt Meyer's loop and produce a contralateral superior quadrantanopia. Temporal lobe lesions can also produce other neurologic manifestations, including aphasia (dominant hemisphere lesions), memory deficits, seizures (complex partial and tonic-clonic), and hallucinations (auditory, olfactory, and visual).

A 65-year-old man is brought to the emergency department due to acute-onset, right-sided weakness and slurred speech. He also has a severe headache and nausea. Medical history is significant for poorly controlled hypertension and chronic tobacco use. Blood pressure is 240/120 mm Hg and pulse is 104/min. On physical examination, the patient is lethargic with right hemiparesis and lower facial weakness, right hemisensory loss, and dysarthria. Noncontrast CT scan of the head is shown. Which of the following cerebral blood vessels is most likely affected in this patient?

Lenticulostriate arteries - This patient's CT findings (eg, hyperdense mass) are consistent with an acute putaminal hemorrhage (ie, affecting the basal ganglia). Because of their location, putaminal hemorrhages almost always affect the adjacent internal capsule, leading to dysarthria, contralateral hemiparesis, and contralateral hemisensory loss due to disruption of the corticobulbar, corticospinal, and somatosensory fibers. As the hemorrhage expands, it leads to increase intracranial pressure (headache, nausea/vomiting, and altered mental status), midline shift from mass effect, and possible cerebral herniation. Hypertensive vasculopathy involving the small, penetrating branches of the major cerebral arteries is the most common cause of spontaneous deep intracerebral hemorrhage. Chronic hypertension leads to the formation of Charcot-Bouchard aneurysms, which may ultimately rupture and bleed within the deep brain structures. The most frequently affected locations include the basal ganglia (putamen), cerebellar nuclei, and pons. The basal ganglia are supplied by the lenticulostriate arteries, which are deep, small vessel branches off the middle cerebral arteries.

A 3-month-old boy is brought to the office for follow-up due to a unilateral cleft lip and palate. The patient has been doing well feeding with specialized bottle nipples and is gaining weight. As part of his multidisciplinary care, the parents are told that he will need to be monitored for ear infections and hearing loss. This is because the patient most likely has dysfunction of which of the following muscles?

Levator veli palatini - This patient has a cleft lip and palate, which can lead to problems with feeding, speech, and orthognathic (ie, jaw) development. Children with cleft palate often have recurrent otitis media due to eustachian tube dysfunction. The muscles primarily responsible for opening the eustachian tube include the tensor veli palatini and the levator veli palatini. The levator veli palatini arises from the petrous portion of the temporal bone and medial part of the eustachian tube. Its fibers insert into the superior portion of the palatine aponeurosis (soft palate) and interdigitate in the midline with the contralateral muscle fibers. It elevates the soft palate during swallowing to prevent food from refluxing into the nasopharynx; contraction also opens the eustachian tube. With cleft palate, levator veli palatini insertion is no longer stable, so contraction does not lead to force on the eustachian tube.

A 32-year-old woman, gravida 2 para 1, at 30 weeks gestation comes to the office for a routine prenatal visit. The patient's pregnancy has been uncomplicated, and she has had no vaginal bleeding or contractions. She has continued to jog daily throughout the pregnancy but is becoming more fatigued by the end of her workouts. The patient has no chronic medical conditions, and her only medication is a daily prenatal vitamin. She does not use tobacco, alcohol, or illicit drugs. Blood pressure is 110/70 mm Hg, pulse is 76/min, and respirations are 20/min. Fetal heart tones are normal. Physical examination is unremarkable. Compared to a nonpregnant state, this patient most likely has which of the following hematologic changes?

Plasma volume ↑ Red blood cell mass ↑ Hemoglobin concentration ↓ During normal pregnancy, the maternal hematologic system undergoes several adaptations to accomodate the developing fetus and placenta. These adaptations include increased plasma volume, increased maternal red blood cell mass, and mild reduction in hemoglobin concentration.

A 47-year-old woman comes to the office due to an itchy rash. For the last 3 months, she has had a gradual onset of multiple pruritic lesions at her wrists, legs, and ankles. The patient tried treating the lesions with emollients and topical over-the-counter hydrocortisone but had no relief. Examination shows scattered, scaly, pink papules and plaques as shown in the exhibit. Which of the following is the most likely diagnosis?

Lichen planus - This patient has lichen planus (LP), an immune-mediated condition that typically presents with pruritic, pink papules and plaques. LP lesions often have lacy, scaly, white markings (Wickham striae) and can form at sites of minor trauma (Köbner phenomenon). The skin lesions typically occur symmetrically on the flexural surfaces of the wrists and ankles but can also involve the nails, oral mucous membranes, and genitalia. The pathologic process in LP is characterized by a (CD8+) T cell-mediated response against cells along the dermal-epidermal junction. Although the etiology is unknown, the risk may be increased in patients with hepatitis C and those taking certain medications (eg, ACE inhibitors, thiazide diuretics). LP typically follows a chronic course with gradual, spontaneous remission within 2 years.

A 49-year-old woman is brought to the emergency department with squeezing chest pain and profuse sweating for the last 2 hours. Medical history includes diet-controlled type 2 diabetes mellitus. ECG reveals ST-segment elevation in leads I, aVL, and V1-V4. The patient is immediately taken to the cardiac catheterization laboratory, where she is found to have complete occlusion of the left anterior descending coronary artery. The blockage is opened with percutaneous coronary intervention with stenting, but afterward she experiences recurrent and sustained episodes of ventricular arrhythmia. She is treated with an antiarrhythmic agent that preferentially binds to rapidly depolarizing and ischemic ventricular myocardial fibers and has minimal effect on normal ventricular myocardium. Which of the following agents was most likely used in this patient?

Lidocaine - This patient sustained an anterolateral ST elevation myocardial infarction (MI) with successful reperfusion following percutaneous coronary intervention. Ventricular arrhythmias (premature beats, ventricular tachycardia/fibrillation) are common in the first 24-48 hours after MI and may be treated with antiarrhythmic medications (eg, amiodarone, lidocaine). Class IB drugs like lidocaine are the weakest sodium channel blockers (dissociate the fastest) compared to other class I antiarrhythmics. They predominantly bind to sodium channels in the inactivated state, and dissociation from the channels occurs so rapidly that they have a negligible effect on QRS duration in normal cardiac tissues. Ischemic myocardium has higher than normal (less negative) resting membrane potential, which delays voltage-dependent recovery of sodium channels from the inactivated to the resting state. This allows increased binding of class IB agents; therefore, class IB antiarrhythmics are highly efficacious in inhibiting ischemia-induced ventricular arrhythmias.

A 62-year-old woman comes to the emergency department due to difficulty walking that started 3 hours ago. The patient also cannot feel anything on the right side of her body. She has a history of hypertension and diabetes mellitus, and has smoked a pack of cigarettes daily for the past 30 years. The patient's father had a myocardial infarction at age 60. Physical examination shows loss of touch, temperature, and vibratory sensation affecting the entire right side of the body. Sensation is also diminished over the right side of the face. Muscle strength is 5/5 throughout. Speech, vision, and hearing tests are normal. The patient was diagnosed with a pure sensory stroke and received appropriate treatment. Her symptoms improved, and she returned home to live with her daughter after a few weeks of physical rehabilitation. Five years later, the patient dies of a large myocardial infarction. On autopsy, there are two 5- to 6-mm cavities in the deep structures of her brain filled with clear fluid. Which of the following processes is most likely responsible for the patient's brain findings?

Lipohyalinosis with small-vessel occlusion - Lipohyalinosis and microatheromas are believed to be the primary causes of lacunar infarcts. Lipohyalinosis occurs secondary to leakage of plasma proteins through damaged endothelium and is characterized by hyaline thickening of the vascular wall, collagenous sclerosis, and accumulation of mural foamy macrophages. Microatheromas result from atherosclerotic accumulation of lipid-laden macrophages within the intimal layer of a penetrating artery near its origin off the parent vessel. These changes predispose to small-vessel occlusion and infarction of CNS tissue with liquefactive necrosis and the formation of a fluid-filled cavity.

A 24-year-old man comes to the office for treatment of opioid use disorder. The patient has a 6-year history of heroin use, which he says has cost him most of his savings. His boss has spoken to him about his erratic moods and inconsistent work performance, and he is now in danger of losing his job. The patient asks for help, saying, "I have tried so hard to do this on my own. My parents have spent a lot of money on counseling. It helps for a while, but then I go back to using. They are fed up and refuse to see me anymore. I don't want to use, but the craving is so strong." The physician refers the patient to an opioid treatment program where he is started on maintenance therapy with methadone. During the next followup appointment, he reports a marked reduction in his cravings. Which of the following properties of the administered drug is most likely responsible for the improvement in this patient's condition?

Long half life - Treatment of addiction to opioids includes the use of alternative opioid agonists with fewer euphoric effects and less potential for acute withdrawal and craving, thereby allowing patients to function more productively on a daily basis. In the United States, the most commonly used agonists are methadone and buprenorphine. Methadone is a full mu-opioid receptor agonist used for withdrawal and maintenance treatment for opioid use disorder. It has a long half-life, which allows it to effectively suppress cravings and withdrawal symptoms; it also blocks the euphoric effects of other opioids by maintaining high tolerance levels. Adverse effects of methadone include QT interval prolongation and respiratory depression, accounting for its lethality in overdose.

A 56-year-old woman is brought to the emergency department due to a 2-day history of high fever, headache, mild confusion, and dry cough. She also has mild abdominal discomfort and watery diarrhea. The patient recently returned from a cruise to Hawaii. Her other medical problems include hypertension and hyperlipidemia. She has smoked 1 pack of cigarettes daily for over 20 years. Her temperature is 40 C (104 F), blood pressure is 104/63 mm Hg, pulse is 85/min, and respirations are 24/min. Lung examination reveals lower lobe crackles with no wheezing. Her abdomen is soft, non-distended, and non-tender. Chest x-ray shows bilateral lower lobe interstitial infiltrates. Which of the following additional findings is most likely to be present in this patient?

Low serum sodium - Legionellosis can be divided into Pontiac fever (an acute, flu-like, self-limited disease) and the more common Legionnaires' disease. Legionnaires' disease should be suspected in patients with recent exposure to contaminated water (sporadic cases or common-source outbreaks in cruise ships, spas, hospitals, or air-conditioned hotels), radiographic evidence of pneumonia (typically patchy infiltrates that may progress to consolidation), high fever (>39 C [102.2 F]) sometimes associated with relative bradycardia, neurologic symptoms (eg, confusion, headache), and gastrointestinal (GI) symptoms (eg, diarrhea). Risk groups include the elderly or immunocompromised, smokers, and patients with chronic obstructive pulmonary disease. Because Legionella pneumophila is a faintly staining gram-negative bacillus that is facultatively intracellular, Gram stain typically shows many neutrophils but few or no organisms. Legionella is often diagnosed by urinary antigen testing; it grows on selective medium (buffered charcoal yeast extract [BCYE]). The most common laboratory abnormality is hyponatremia, frequently associated with Legionella but not other causes of pneumonia. The hyponatremia may be related to inappropriate antidiuretic hormone secretion and/or renal tubulointerstitial disease impairing sodium reabsorption (possibly due to either direct effect of Legionella, cytokines, or natriuretic peptides). Elevated transaminases are also common.

A researcher is conducting an experiment to determine factors affecting vascular resistance. Experiment animals are anesthetized, intubated, and attached to a controlled ventilation system. A catheter is placed into a small artery in various organs in each animal. The catheter is connected to a manometer for recording pressure and to an oximeter for continuous recording of oxygen saturation. The fraction of inspired oxygen is gradually changed, and arteriolar resistance is calculated from the manometer readings at regular intervals. The results shown below were most likely obtained from which of the following organs?

Lungs - This graph depicts a vascular bed in which arterial/arteriolar resistance increases as the tissue O2 content decreases. This response is unique to the pulmonary vasculature, in which hypoxic vasoconstriction occurs to divert blood flow away from underventilated lung regions and toward well-ventilated lung areas. This phenomenon improves ventilation-perfusion mismatch by decreasing physiologic shunting in poorly ventilated alveoli, leading to overall more efficient gas exchange. The relationship between hypoxia and vascular resistance is reversed throughout the rest of the body, ensuring that hypoxic organs and tissues receive increased blood flow.

A 36-year-old man comes to the office for follow-up regarding hepatitis C. The patient was last seen 3 years ago for a 2-week history of fatigue, nausea, and decreased appetite; at that time, he was diagnosed with hepatitis C infection due to injection drug use. The patient was offered treatment but declined. He is currently healthy and feels well. There is no family history of liver disease. The patient currently does not use illicit drugs or alcohol. Vital signs are normal. BMI is 26 kg/m2. Examination is normal. Laboratory results are as follows: Albumin 3.6 g/dL Total bilirubin 1.7 mg/dL Aspartate aminotransferase (SGOT) 54 U/L Alanine aminotransferase (SGPT) 62 U/L Hepatitis B surface antigen negative Hepatitis B surface antibody positive Anti-hepatitis C virus antibody positive Hepatitis C RNA positive HIV antibody negative Liver ultrasound is negative for cirrhosis or masses. If a liver biopsy is performed, which of the following histologic findings would most likely be present in this patient?

Lymphocytic infiltration of portal tracts - Most cases of Hepatitis C (~80%) progress to chronic HCV, which is predominantly characterized by portal tract-based lymphocytic infiltration, including lymphoid follicles, and fibrosis. Chronic inflammation causes persistent hepatocyte injury and can involve the hepatocytes adjacent to the portal tracts (ie, interface activity). After several decades, chronic HCV can progress to cirrhosis characterized by abnormal hepatic architecture with extensive fibrosis and the formation of bridging fibrous septa that surround regenerative parenchymal nodules.

The acidification of lysosomes within antigen presenting cells is prevented in an experimental setting. Affected cells show impaired interaction with T lymphocytes upon antigen exposure. This impaired interaction is most likely the result of decreased cell surface expression of which of the following molecules?

MHC class II - Antigen presenting cells (eg, dendritic cells, macrophages, B lymphocytes) present extracellular antigens (eg, bacterial, viral) on MHC class II molecules. After phagocytosis or endocytosis, the protein is degraded in acidified lysosomes and the antigen is loaded onto an MHC class II molecule. The MHC class II-antigen complexes are then displayed on the surface of antigen presenting cells where they can bind to T cell receptors (TCR) to initiate a T cell response to the antigen. Without lysosomal acidification, exogenous antigen processing and association with MHC class II molecules would not occur, resulting in impaired interaction between antigen presenting cells and T cells.

A research scientist studying the metabolic pathways that contribute to obesity feeds experimental animals a high-carbohydrate, high-protein diet for a prolonged period. A sample of liver tissue is then obtained from the animals, and the activity of various enzymes involved in fatty acid metabolism is measured and recorded. It is determined that beta-oxidation of fatty acids is inhibited within these cells as a result of the diet. An increase in which of the following substances is most likely responsible for the observed effect?

Malonyl-CoA - In the well-fed state, the abundance of ATP in hepatocytes inhibits isocitrate dehydrogenase, leading to high levels of citrate in the mitochondria. Citrate is transferred to the cytosol via the citrate shuttle and cleaved by ATP citrate lyase to form acetyl-CoA. High citrate levels (in addition to elevated insulin caused by high carbohydrate intake) causes upregulation of Acetyl-CoA carboxylase. This cytosolic enzyme catalyzes the conversion of acetyl-CoA to malonyl-CoA in the rate-limiting step of de novo fatty acid synthesis. Fatty acid synthase then catalyzes the condensation of malonyl-CoA with acetyl-CoA to create a 4-carbon molecule that will undergo subsequent condensation reactions to form a 16-carbon fatty acid. Beta-oxidation of fatty acids takes place primarily within the mitochondrial matrix. Mitochondrial membranes are impermeable to fatty acids due to their negative charge, so a specialized membrane carrier (carnitine) must be used to shuttle them into the matrix. Malonyl-CoA inhibits carnitine acyltransferase, preventing the transfer of acyl groups into the mitochondria. This inhibitory action functions to prevent the breakdown of newly synthesized fatty acids.

A 3-year-old boy is rushed to the emergency department after he is found underwater in the bathtub. His parents say the patient was left alone for 5 minutes. On physical examination, he does not respond to painful stimuli and is pulseless and not breathing. Cardiac rhythm is asystole and spontaneously returns after several minutes of resuscitation. While performing endotracheal intubation, stimulation of the posterior pharyngeal wall with laryngoscope elicits no soft palate movements or gagging. This patient's findings during intubation most likely indicate dysfunction of which of the following neural structures?

Medulla oblongata - This patient with prolonged oxygen deprivation has an absent gag reflex. The gag reflex is an airway protective mechanism that acts to expel foreign or toxic material from the aerodigestive tract. It is mediated by the following: Sensory signals from the back of the tongue or posterior pharyngeal wall are sent via the glossopharyngeal nerve (CN IX); Somatic sensory fibers of CN IX enter the brainstem at the medulla, descend along the spinal trigeminal tract, and synapse in the caudal spinal nucleus; Information is then sent to the bilateral nuclei ambiguus, which are also located in the medulla; Bilateral motor (efferent) signals are sent via the vagus nerves (CN X), causing elevation of the soft palate and contraction of the pharynx. Therefore, an absent gag reflex is suspicious for dysfunction of the medulla oblongata. In combination with a prolonged anoxic period, this is concerning for brain death, which is the irreversible absence of cerebral and brainstem reflexes.

A 10-year-old girl is brought to the office for evaluation of short stature. She was an average-sized infant, but over the past few years, her height growth velocity has plateaued. The patient has not menstruated and has no symptoms. She takes no medications, has no allergies, and has received all recommended immunizations. Menarche occurred in her mother at age 14 and both of her parents are tall. The patient's height is at the <5th percentile and weight is at the 50th percentile for age and sex. She has no breast buds and no axillary or pubic hair. She has a low hairline, a short and wide neck, a broad chest, and widely spaced nipples. Which of the following is the most likely underlying mechanism for this patient's condition?

Meiotic nondisjunction - Turner syndrome is a genetically heterogeneous condition that is most commonly due to meiotic nondisjunction during gametogenesis. The loss of one X chromosome in the sperm or egg results in a missing X chromosome in all of the patient's cells (45,X). In other patients, the nondisjunction event occurs during mitosis in early embryogenesis; these patients are missing the X chromosome in only some of their cells (mosaic Turner syndrome [45,X/46,XX]). A minority of patients have both X chromosomes, but one is structurally abnormal and missing some genetic material (eg, X fragments, isochromosomes). The loss of all or part of the X chromosome in Turner syndrome results in a missing SHOX gene, which normally promotes long bone growth. Therefore, patients with this syndrome typically have short stature. Meiotic nondisjunction is also responsible for Klinefelter syndrome and trisomies 13, 18, and 21.

A 33-year-old man comes to the office due to episodic headaches for several months accompanied by sweating and feelings of anxiety that spontaneously resolve after 15-30 minutes. He has no other medical conditions and takes no medications. His brother recently had surgery to treat hyperparathyroidism. Blood pressure is 180/110 mm Hg, and pulse is 102/min. Laboratory results show normal serum electrolytes and renal function. CT scan of the abdomen reveals a mass in the right adrenal gland. Which of the following cells have the same embryological origin as the tissue responsible for this patient's current condition?

Melanin-producing cells - This patient with severe hypertension, headaches, and an adrenal mass has typical features of pheochromocytoma, a catecholamine-secreting tumor of the chromaffin cells of the adrenal medulla. Pheochromocytoma can be sporadic or associated with several familial disorders, especially multiple endocrine neoplasia (MEN) type 2 (particularly likely in this patient given his family history of hyperparathyroidism). MEN type 2 is associated with germ-line mutations in the RET proto-oncogene. Neural crests are embryological structures composed of parallel strips of cells arising from the ectoderm at the margin of the neural tube. The chromaffin cells of the adrenal medulla are derived from neural crest tissue (the adrenal cortex is derived from the mesoderm). Structures arising from neural crest cells can be remembered with the mnemonic "SOME SALT" (Schwann cells, odontoblasts, melanocytes, enterochromaffin cells, spinal membranes [pia and arachnoid], adrenal medulla/ganglia, laryngeal cartilage, tracheal cartilage).

A 45-year-old woman is brought to the emergency department after an episode during which she abruptly lost consciousness and had rhythmic jerking movements of the limbs followed by a period of confusion. The patient has had mild daily headaches associated with nausea that have progressed over the past 2 years. The headaches are worse at night and occasionally awaken her. The patient reports no head trauma, visual changes, weakness, numbness, or difficulty swallowing or speaking. She does not use alcohol or illicit drugs. There is no family history of seizures. An intracranial mass is discovered on neuroimaging and a biopsy of the mass is shown. Which of the following is the most likely diagnosis?

Meningioma - This patient's biopsy findings are consistent with meningioma, a slow-growing benign tumor derived from meningothelial cells of the arachnoid. Note the syncytial nests and whorled pattern of cellular growth, which may calcify and appear as round, laminar structures called psammoma bodies. Other examples of tumors that form psammoma bodies include papillary thyroid carcinoma, mesothelioma, and serous carcinoma of the ovary and endometrium. Meningiomas form well-circumscribed, round masses attached to the dura and are commonly found at the falcine, parasagittal, or lateral convexity regions of the brain. Seizure can occur due to compression of the adjacent cerebral cortex. Headaches, nausea, and vomiting may result from increased intracranial pressure, which worsens during recumbency and with sleep (due to increased intracranial blood volume). Individuals may also develop focal neurologic deficits depending on tumor size/location.

A 56-year-old woman comes to the emergency department due to 3 days of frequent urination, suprapubic pain, dysuria, and progressive hematuria. She has had no fevers or chills. The patient has a history of lymph-node-positive breast cancer that was diagnosed following a routine mammogram. A month ago, she began treatment with systemic chemotherapy. Temperature is 37.1 C (98.8 F). Suprapubic tenderness is present on abdominal examination. Hemoglobin is 9.8 g/dL. Urinalysis shows numerous red blood cells but no leukocyte esterase or bacteria. Which of the following could have prevented this patient's current condition?

Mesna - This patient who recently received systemic chemotherapy for breast cancer and now has progressive hematuria and suprapubic tenderness most likely has hemorrhagic cystitis caused by a nitrogen mustard-based chemotherapeutic agent, such as cyclophosphamide or one of its analogs (eg, ifosfamide). These agents are metabolized by the kidneys into acrolein, which is then excreted in the urine. Acrolein is toxic to uroepithelial cells and can cause cell death and necrosis if allowed to be in contact with these cells for a prolonged period. Hemorrhagic cystitis associated with nitrogen mustard-based chemotherapy can be prevented by aggressive hydration and the coadministration of mesna (2-mercaptoethanesulfonate), a sulfhydryl compound that binds and inactivates the toxic metabolites of the chemotherapeutic agents in the urine.

A 28-year-old woman comes to the emergency department with acute-onset abdominal pain, nausea, and confusion. She has no significant past medical history and does not use tobacco or alcohol as they have made her feel sick in the past. Serum lipase and liver function tests are within normal limits. CT scan of the abdomen shows no abnormalities. A sample of her urine is reddish in color and darkens on standing for 24 hours. Intravenous dextrose is administered and her symptoms improve significantly. Dextrose infusion most likely improved this patient's condition by affecting which of the following pathways?

Porphyrin synthesis - This patient most likely has acute intermittent porphyria (AIP), an autosomal dominant disorder of the heme synthesis pathway caused by porphobilinogen (PBG) deaminase deficiency. In general, enzyme deficiencies in the early steps of porphyrin synthesis cause neurovisceral symptoms (acute porphyrias); deficiencies in the latter steps (after condensation of PBG to HMB) result in photosensitivity (cutaneous porphyrias). AIP is an acute porphyria that causes nervous system dysfunction due to the accumulation of early heme pathway intermediates (PBG and delta-aminolevulinic acid [ALA]). It presents acutely with variable gastrointestinal and neurologic symptoms, most commonly abdominal pain, vomiting, peripheral neuropathy, and neuropsychiatric derangements. A key feature of acute porphyrias is reddish urine that darkens on exposure to light and air due to the oxidation of excess PBG. Treatment and prevention of acute porphyria attacks is directed at inhibiting ALA synthase (the rate-limiting enzyme of heme synthesis) to reduce formation of the toxic intermediate metabolites. ALA synthase is upregulated by CYP450 inducers (eg, most antiepileptics, griseofulvin, rifampin) and downregulated by heme and glucose. As such, avoidance of alcohol, smoking, and other CYP450-inducing drugs is important for preventing acute attacks. Likewise, intravenous heme administration and carbohydrate loading (such as with dextrose infusion) are useful for ameliorating acute symptoms.

A 24-year-old man comes to emergency department complaining of abdominal pain, vomiting, and severe watery diarrhea. He recently returned from a camping trip and admits to eating wild mushrooms that he collected in the woods. His past medical history is insignificant and he takes no medications. He does not use illicit drugs. On physical examination, he is ill-appearing and jaundiced. His liver edge is soft, tender, and palpable 4 cm below the right costal margin. Laboratory tests are significant for elevated levels of alanine aminotransferase, aspartate aminotransferase, and bilirubin. Synthesis of which of the following is most likely to be directly inhibited by the responsible toxin?

Messenger RNA - Amatoxins are found in a variety of poisonous mushrooms (eg, Amanita phalloides, known as death cap) and are responsible for the majority of mushroom poisoning fatalities worldwide. Ingestion of 1 or more amatoxin-containing mushrooms is a life-threatening emergency. After absorption by the gastrointestinal tract, amatoxins are transported to the liver via the portal circulation where active transport by organic anion transporting polypeptide (OATP) and sodium taurocholate co-transporter (NTCP) concentrates the toxin within the liver cells. There, amatoxins bind to DNA-dependent RNA polymerase type II and halt mRNA synthesis, ultimately resulting in apoptosis. Other organ systems with rapid cellular turnover can also be affected in amatoxin poisoning, including the gastrointestinal tract and proximal convoluted renal tubules. Symptoms typically start 6-24 hours after ingestion and include abdominal pain, vomiting, and severe, cholera-like diarrhea that may contain blood and mucus. Severe poisoning can lead to acute hepatic and renal failure. Urine testing for α-amanitin can confirm suspected amatoxin poisoning.

A 68-year-old man comes to the emergency department due to a 1-week history of increasing leg and abdominal swelling. The patient has a history of pulmonary hypertension and cor pulmonale from advanced chronic obstructive pulmonary disease. Other medical conditions include hypertension and gout. Physical examination shows scattered rhonchi, prolonged expiratory phase of expiration, mild ascites, and extensive edema of the abdominal wall and lower extremities. The patient is hospitalized and intravenous loop diuretic therapy is begun. Two days later, acetazolamide is added to his treatment regimen. Which of the following most likely prompted the additional therapy in this patient?

Metabolic alkalosis -Loop diuretics (eg, furosemide) inhibit the Na+-K+-2Cl− transporter in the ascending loop of Henle to stimulate potent excretion of Na+ and water and reduce total body fluid volume. Electrolyte abnormalities are common with the use of loop diuretics; metabolic alkalosis occurs due to renal excretion of H+ and K+ as well as retention of HCO3-. This metabolic alkalosis can have important implications because it stimulates compensatory hypoventilation that may hinder weaning from mechanical ventilation in critically ill patients. Carbonic anhydrase inhibitors (eg, acetazolamide) help offset the metabolic alkalosis; these drugs inhibit the reabsorption of sodium bicarbonate (NaHCO3) in the proximal tubule, leading to increased HCO3− excretion. The metabolic acidosis that is generated reduces blood alkalinity to help normalize pH.

A 3-day-old boy is brought to the emergency department due to poor feeding, emesis, and lethargy over the last 24 hours. The patient was born via uncomplicated spontaneous vaginal delivery to a 30-year-old woman who had a normal pregnancy. The boy was discharged from the newborn nursery yesterday and was breastfeeding exclusively until the onset of symptoms. Stool and urine output were normal while he was in the newborn nursery. The patient is afebrile and normotensive but tachycardic and tachypneic. He appears dehydrated, and the abdomen is distended. The patient vomits during the examination, and the vomitus is shown in the exhibit. On laparotomy, fibrous bands are seen extending from the cecum and right colon to the retroperitoneum, causing extrinsic compression of the duodenum. Which of the following embryologic processes most likely failed in this patient?

Midgut rotation around the superior mesenteric artery - Around 6 weeks gestation, the midgut (supplied by the superior mesenteric artery) herniates through the umbilical ring in order to grow rapidly. During this process, the midgut rotates 90 degrees counterclockwise. Following additional growth, the midgut returns to the abdominal cavity at 8-10 weeks gestation and turns an additional 180 degrees counterclockwise (270 degrees total). Subsequently, the gut is fixed to the posterior abdomen on a wide-based mesentery. Incomplete counterclockwise rotation (eg, 180 degrees) will result in midgut malrotation. The cecum will rest in the right upper quadrant instead of the right lower quadrant (RLQ). Additionally, Ladd's (fibrous) bands connect the retroperitoneum in the RLQ to the right colon/cecum by passing over the second part of the duodenum, causing intestinal obstruction in the process. Obstruction manifests as bilious emesis during the first days of life. In addition, because the mesenteric base is abnormally narrowed, the mesentery is vulnerable to twisting around the superior mesenteric artery. The twisting, referred to as midgut volvulus, compromises intestinal perfusion and may lead to life-threatening bowel necrosis.

A 69-year-old woman comes to the office due to memory concerns. The patient first began having difficulties with her memory a year ago, and her symptoms have steadily worsened. She often forgets the content of conversations she has with her son and frequently walks to another room, only to forget what she went there to do. The patient lives alone and has no difficulty cooking, cleaning, or otherwise caring for herself. She manages her own finances. Medical history is significant for hypertension, hyperlipidemia, and type 2 diabetes mellitus. Temperature is 36.9 C (98.4 F), blood pressure is 132/83 mm Hg, pulse is 72/min, and respirations are 14/min. The patient is alert and oriented. Neurological examination reveals no focal deficits. She scores 22/30 on the Montreal Cognitive Assessment. Which of the following is the most likely diagnosis?

Mild cognitive impairment - This patient's mild memory impairment, abnormal Montreal Cognitive Assessment score (<26/30) and ability to function independently are consistent with mild cognitive impairment, also known as mild neurocognitive disorder. MCI is diagnosed when there is objective evidence of cognitive deficits that do not interfere with activities of daily living (ADLs) (eg, bathing, preparing meals, managing finances). In contrast, a diagnosis of dementia, also known as major neurocognitive disorder, requires the presence of progressive cognitive deficits that impair ADLs. MCI is common in older age, with >10% of patients age >70 meeting criteria. Patients with subjective memory concerns should be evaluated objectively through cognitive testing because certain memory issues (eg, difficulty remembering names, decreased processing speed) may represent normative age-related cognitive changes. When evaluating for MCI, it is important to rule out reversible causes of cognitive impairment such as depression-related cognitive impairment, hypothyroidism, vitamin B12 deficiency, impaired sleep (eg, obstructive sleep apnea), and adverse effects of pharmacotherapy (eg, benzodiazepines). This patient does not meet criteria for dementia (ie, progressive cognitive impairment that impairs ADLs). Alzheimer disease presents with early and prominent memory impairment. Frontotemporal dementia presents with personality and behavioral changes. Vascular dementia is characterized by focal neurological deficits and evidence of cerebrovascular disease on imaging.

A 15-year-old girl is brought to the clinic for evaluation of swelling around her eyes that developed over the past week. The patient is a cheerleader, and she had been taking ibuprofen daily for the last 3 months for various sprains and aches after practice. She has no chronic medical conditions. Vital signs are normal. On physical examination, there is moderate periorbital edema with bilateral lower extremity pitting edema. Serum creatinine is 0.5 mg/dL and serum albumin is 2.1 g/dL. Urinalysis shows 4+ protein and negative blood. Multiple regions of the kidney are biopsied, and a representative image is shown. Which of the following is the most likely diagnosis?

Minimal change disease - This patient with edema, proteinuria, and hypoalbuminemia has nephrotic syndrome. The normal glomeruli on light microscopy (LM) suggest a diagnosis of minimal change disease (MCD), the most common cause of nephrotic syndrome in children. It is often idiopathic but may be triggered by drugs (eg, nonsteroidal anti-inflammatory drugs [NSAIDs], as in this patient), immunizations, or malignancy (eg, Hodgkin lymphoma). Pathogenesis involves T cell dysfunction with production of a glomerular permeability factor (possibly IL-13), which damages podocytes and decreases the anionic charge of the glomerular basement membrane (GBM), allowing for selective loss of albumin in the urine. Clinical features include acute weight gain, diffuse edema, and frothy urine due to heavy proteinuria. Renal biopsy demonstrates normal glomeruli on LM, with no immunoglobulin or complement deposits visible on immunofluorescent microscopy. However, electron microscopy shows diffuse effacement and fusion of podocyte foot process.

A 54-year-old man with a history of chronic kidney disease due to hypertension develops anemia. Evaluation reveals that the cause of anemia is erythropoietin deficiency. Erythropoietin increases the numbers of erythroid precursor cells in the bone marrow and induces heme production in erythrocyte precursors. In this patient, mature erythrocytes are found that are unable to synthesize heme even though they contain detectable levels of cytoplasmic enzymes involved in heme synthesis. Lack of which of the following cellular organelles best explains this phenomenon?

Mitochondria - Heme synthesis occurs partly in the mitochondria and partly in the cytoplasm of erythrocytes. Mitochondria are necessary for the first and the final 3 steps. Erythrocyte precursors divide a number of times before finally losing their nuclei and mitochondria and forming mature red blood cells that survive for about 120 days (4 months). When erythrocytes lose their mitochondria, they lose the ability to generate heme and therefore hemoglobin. Heme is synthesized in virtually every organ, but the principal sites of synthesis are erythrocyte precursor cells (located in the bone marrow) and hepatocytes (use heme in microsomal cytochrome P450 system).

A 44-year-old man comes to the office due to low mood, impaired concentration, increased sleep and appetite, feelings of heaviness in his arms and legs, and loss of energy. He is having difficulty at work as he is overly sensitive to criticism. Although his boss has told him not to worry, the patient is concerned that his job is in jeopardy due to poor performance. His symptoms started 8 months ago without any clear-cut precipitating event. The patient received adequate trials of 3 different antidepressants without improvement and has been off of medication for the past several weeks. His physician is now considering electroconvulsive therapy (ECT). The patient declines ECT and asks to try another medication. The physician then considers phenelzine. The presence of which of the following additional symptoms would make the patient more likely to respond to this medication?

Mood reactivity - Monoamine oxidase inhibitors (MAOIs) (eg, phenelzine, tranylcypromine) are antidepressants that work by inhibiting oxidative deamination, thereby increasing the presynaptic availability of serotonin, norepinephrine, and dopamine. Due to their risk of severe adverse effects (eg, hypertensive crisis, serotonin syndrome), MAOIs are not considered first- or second-line antidepressant treatments. However, MAOIs are still useful in certain situations when other antidepressants fail. In addition to their role in treatment-resistant major depressive disorder (MDD), there is some evidence to suggest that they are superior to other classes of antidepressants in the treatment of MDD with atypical features. MDD with atypical features is characterized by mood reactivity (ie, mood improves in response to positive events), leaden paralysis (ie, patient's arms and legs feel extremely heavy), rejection sensitivity (ie, overly sensitive to slight criticism), and the reversed vegetative signs of increased sleep and appetite. This patient is both treatment-resistant (failure of multiple antidepressant agents) and has atypical features, making him a good candidate for a trial of an MAOI.

A 4-year-old boy is evaluated due to 4 days of fever and leg pain. Medical history includes recurrent skin abscesses and lymphadenitis. A maternal uncle died of recurrent infections during childhood. The patient is at the 10th percentile for height and weight. Temperature is 38.3 C (100.9 F). Examination shows an ill-appearing patient with point tenderness over the right lower tibia. There are healed incisions over the thighs and neck from prior drainage sites. Imaging reveals evidence of tibial osteomyelitis. Blood and bone cultures grow Serratia marcescens. Evaluation of the patient's neutrophils by dihydrorhodamine flow cytometry shows an absence of fluorescence. This patient's condition is most likely due to impaired activity of which of the following enzymes?

NADPH oxidase - This patient's abnormal flow cytometry findings are characteristic of chronic granulomatous disease (CGD), an X-linked condition caused by NADPH oxidase deficiency. NADPH oxidase is a membrane-bound complex within phagolysosomes that catalyzes the reduction of oxygen to superoxide during the respiratory burst. The production of reactive oxygen species facilitates the intracellular killing of organisms ingested by phagocytes. Patients with CGD have impaired intracellular killing, resulting in recurrent bacterial and fungal infections (eg, abscesses, lymphadenitis, osteomyelitis) caused by certain catalase-positive organisms (eg, Serratia).

A 48-year-old woman is evaluated for a painless left breast mass. Physical examination shows a 4-cm mass with dimpling of the overlying skin; left axillary lymphadenopathy also is noted. Breast imaging reveals a spiculated soft tissue mass with microcalcifications. Biopsy of the mass demonstrates invasive ductal carcinoma. Immunohistochemistry of the tissue sample shows the tumor cells are negative for estrogen and progesterone receptors but overexpress the HER2 epidermal growth factor receptor. The patient is treated with systemic chemotherapy and a monoclonal antibody drug that binds the extracellular domain of the growth factor receptor. Repeat evaluation 6 weeks later shows a decrease in the size of the breast mass. Which of the following host factors are primarily responsible for mediating the response to the monoclonal antibody therapy?

Natural killer cells and granzymes - HER2 is an epidermal growth factor receptor overexpressed by aggressive breast cancers. Treatment with a monoclonal antibody (eg, trastuzumab) that targets the HER2 surface receptor often results in tumor regression. Although antibody binding can trigger cancer cell death through a variety of mechanisms (eg, receptor downregulation, complement deposition), much of the response is due to antibody-dependent cellular cytotoxicity (ADCC). This classically involves: A naturally occurring or monoclonal IgG binds to a complementary surface antigen on a host or foreign cell; A patrolling natural killer (NK) cell binds the Fc portion of the attached IgG using CD16; Binding triggers the NK cell to release its granules, which contain perforin and granzymes (proteases) that induce apoptosis/osmotic lysis of the antibody-bound cell.

A 31-year-old woman comes to the office due to a 4-month history of left hip and groin pain. She has a constant, dull, achy pain at rest that is exacerbated by movement of the hip or weight bearing. The patient has had no recent falls or other trauma. Medical history is significant for sickle cell disease, pneumococcal pneumonia, and acute chest syndrome. There is no redness, warmth, or tenderness over the hip joint, but decreased active and passive internal and external rotation, extension, and abduction are noted. Imaging of the hip is shown in the exhibit. Which of the following is the most likely cause of this patient's pain?

Necrosis of bone and marrow cells - This patient has chronic hip pain that is worse with weight bearing and decreased range of motion in multiple axes. X-ray reveals flattening and patchy sclerosis of the femoral head consistent with osteonecrosis (avascular necrosis). Biopsy is not generally required for diagnosis, but microscopic analysis of the affected bone may show dead bony trabeculae (empty lacunae) and fat necrosis. Osteonecrosis occurs due to disruption of the macrovasculature or microvasculature of bone. The femoral head is the most common location, although other bones may be affected. Conditions associated with osteonecrosis include sickle cell disease, embolic disorders, high-dose systemic corticosteroids, excessive alcohol intake, vasculitis, and femoral neck fracture.

A 14-year-old girl comes to the emergency department due to rapidly progressive exertional dyspnea and generalized weakness, which were preceded by a mild febrile illness several days ago. She has a history of sickle cell disease and takes a daily folic acid supplement. Her temperature is 37.2 C (99.1 F), blood pressure is 115/70 mm Hg, and pulse is 112/min and regular. On examination, the patient has conjunctival pallor but no icterus. A cardiac flow murmur is present. The abdomen is soft and nontender with no organomegaly. Laboratory studies show a hematocrit of 16% with reticulocyte count of 0.1% (normal range, 0.5%-1.5%). Leukocyte and platelet counts are normal. Which of the following best describes the virus most likely responsible for this patient's current condition?

Nonenveloped single-stranded DNA virus - This patient has developed severe anemia (eg, pallor, low hematocrit, flow murmur) following a minor febrile illness. If her bone marrow were able to respond appropriately to the degree of anemia, the reticulocyte count would be elevated (normal reticulocyte count is 0.5%-1.5% of red cells). Instead, the patient's reticulocyte count is markedly reduced. This scenario describes an aplastic crisis, which in sickle cell patients is usually due to parvovirus B19 infection of erythroid precursor cells in the bone marrow. Parvovirus is a nonenveloped single-stranded DNA virus. Destruction of the erythroid precursor cells by this virus diminishes the number of reticulocytes available to replace the deformed and/or removed erythrocytes.

A 25-year-old primigravida has a stillbirth at 18 weeks gestation. Her only symptom during pregnancy was pain in both knees and feet, which she attributed to pregnancy-related weight gain and being "on my feet all day" as an elementary school teacher. The pain lasted approximately a week and resolved without medication. The patient was taking prenatal vitamins daily, and her prenatal care was appropriate. Fetal autopsy shows pleural effusion, pulmonary hypoplasia, and ascites. Infection with which of the following is the most likely etiology of the stillbirth?

Nonenveloped, single-stranded DNA virus - Parvovirus is a nonenveloped single-stranded (ss) DNA virus with an increased incidence in children and teachers, as most individuals are infected during school outbreaks. Infection in adults can present with an acute, symmetric arthralgia/arthritis involving hands, wrists, knees, and/or feet, with or without rash. Fetal infection with parvovirus can lead to interruption of erythropoiesis, causing profound anemia and congestive heart failure. Fetal congestive heart failure can cause pleural effusions, pericardial effusions, and ascites. The findings on fetal autopsy (pleural effusion with secondary pulmonary hypoplasia, and ascites) represent fetal hydrops. Parvovirus is also responsible for erythema infectiosum ("fifth disease"), which presents with a "slapped cheek" appearance in children, and aplastic crisis in patients with sickle cell anemia.

Autopsy of a 78-year-old man demonstrates decreased left ventricular cavity size and a sigmoid-shaped ventricular septum. Light microscopy shows increased collagen content within the ventricular wall. Some myocardial cells also have brownish perinuclear cytoplasmic inclusions. The changes described are most consistent with which of the following conditions?

Normal aging - The morphologic changes described are consistent with normal aging of the heart. Aging is associated with decreased left ventricular chamber size, particularly in the apex-to-base dimension. This decrease in chamber length causes the ventricular septum to acquire a sigmoid shape, with the basilar portion bulging into the left ventricular outflow tract. Atrophy of the myocardium results in increased interstitial connective tissue, often with concomitant extracellular amyloid deposition. Within the cardiomyocytes, there is a progressive accumulation of cytoplasmic granules containing brownish lipofuscin pigment (the result of indigestible byproducts of subcellular membrane lipid oxidation).

A 55-year-old woman comes to the emergency department with nausea, fever, fatigue, and anorexia. She returned a week ago from a trip to Mexico, where she underwent an emergency cholecystectomy without complications. Her temperature is 38.3 C (101 F), blood pressure is 100/60 mm Hg, pulse is 90/min, and respirations are 12/min. The patient is alert and answers questions but appears extremely weak and slightly icteric. She has no other medical problems and takes no medications. She does not use tobacco, alcohol, or illicit drugs. The patient is admitted to the hospital, but her condition worsens and she dies 2 days later. Postmortem viral serologies are negative. Gross examination of the liver on autopsy is shown. Which of the following additional findings would have most likely been found in this patient?

Prolonged thrombin time - This patient presents after a recent surgery with signs and symptoms consistent with drug-induced liver injury, most likely due to inhaled anesthetic use. Inhaled anesthetic hepatotoxicity is most frequently associated with halothane, which remains one of the most commonly used inhaled anesthetics worldwide. In the United States, halothane has been largely replaced with other halogenated anesthetics, such as enflurane, isoflurane, desflurane, and sevoflurane. Although these agents are safer than halothane, there have been reports of associated hepatotoxicity. Halogenated inhaled anesthetics predominantly cause a hepatocellular pattern of liver injury. This can range from mild asymptomatic aminotransferase elevation to fulminant hepatitis with a 50% fatality rate. In severe cases, extensive hepatocellular damage causes the liver to rapidly atrophy and appear shrunken on autopsy. Histologically, widespread centrilobular necrosis and inflammation of the portal tracts and parenchyma are observed, making the condition indistinguishable from fulminant viral hepatitis. The underlying mechanism is thought to be a hypersensitivity reaction to the drug that causes an immune-mediated attack against hepatocytes. Patients suffering from inhaled-anesthetic hepatotoxicity typically have fever, anorexia, nausea, myalgias, arthralgias, and rash. Tender hepatomegaly (reflecting widespread liver inflammation) and jaundice can be present on physical examination. Laboratory findings typically include markedly elevated serum aminotransferase levels, prolonged prothrombin time, leukocytosis, and eosinophilia. The prolonged prothrombin time is due to failure of hepatic synthetic function and deficiency of factor VII (which has the shortest half-life of all the procoagulant factors).

A 12-year-old girl comes to the office with constant swelling and pain of her elbows for the past week that have prevented her from participating in basketball practice. She also had knee pain during the preceding week that was attributed to a fall during practice. Her parents say that she is healthy and has had only minor illnesses that children typically experience during the winter. The patient's temperature is 38.9 C (102 F), blood pressure is 110/70 mm Hg, and pulse is 110/min. Her elbows are swollen and tender with limited range of movement. Her knees appear normal. A new holosystolic murmur is heard on cardiac auscultation. Antistreptolysin O titers are 400 Todd units/mL (normal: <300 Todd units/mL). The patient is admitted to the hospital. During her hospitalization, this patient is at greatest risk of dying from which of the following complications?

Pancarditis - Acute rheumatic fever (ARF) is the most likely diagnosis in this patient with migratory arthritis, new-onset murmur, fever, and a positive anti-streptolysin O titer. ARF is a multisystem complication that develops 2-4 weeks after untreated group A streptococcal pharyngitis. Most organs are often only mildly and transiently affected in ARF, with the exception of the heart. Acute morbidity is most likely due to pancarditis (inflammation of the endocardium, myocardium, and epicardium). Inflammation of the mitral valve can lead to mitral regurgitation, which is the likely cause of the new holosystolic murmur in this patient. Severe regurgitation and/or myocarditis can lead to cardiac dilation, heart failure, and death in a small percentage of patients.

A 56-year-old man with chronic renal insufficiency due to polycystic kidney disease is evaluated for placement of an arteriovenous fistula for dialysis access. Blood pressure is 140/90 mm Hg and pulse is 80/min. Examination shows 2+ bilateral edema of the lower extremities. Estimated glomerular filtration rate is 15 mL/min/1.73 m2. Which of the following sets of laboratory findings is most likely in this patient?

Parathyroid hormone ↑ Serum calcium ↓ Serum phosphorus ↑ 25-hydroxyvitamin D Normal 1,25-dihydroxyvitamin D ↓ This patient has advanced chronic kidney disease (CKD). CKD can cause hyperphosphatemia due to the impaired ability of the kidneys to excrete phosphorus (particularly when GFR is <20 mL/min/1.73 m2). Elevated blood phosphate triggers the release of fibroblast growth factor 23 from bone, which lowers calcitriol (1,25-dihydroxyvitamin D) production and intestinal calcium absorption. In addition, patients with advanced CKD typically have decreased renal conversion of 25-hydroxyvitamin D to 1,25-dihydroxyvitamin D (the more active form) because of inadequate function of renal tissue. The resulting hypocalcemia, along with hyperphosphatemia, stimulates the secretion of parathyroid hormone (PTH) and leads to secondary hyperparathyroidism.

A 45-year-old man is evaluated for persistent cough and progressive dyspnea on exertion. The patient has no medical issues but has been exposed to large quantities of coal dust while working as a miner for the past 15 years. Chest imaging reveals small, nodular opacities in the upper lobes. Lung biopsy shows fine carbon particles within the patient's respiratory bronchioles and alveolar ducts. Which of the following mechanisms is most directly responsible for clearing the foreign particles from this patient's respiratory tract?

Phagocytosis - Inorganic dust is constantly being inhaled and must be cleared by the respiratory tract to prevent disease. The clearance mechanisms used by the lungs vary depending on the size of the particles. Larger particles become trapped by mucus secretions in the trachea, bronchi, and proximal bronchioles; these trapped particles are swept upward toward the pharynx by the collective beating of ciliated cells. The finest particles (<2 µm) can travel past the highly ciliated airways into the respiratory bronchioles and alveoli, where they are phagocytized by alveolar macrophages. Engulfment of inorganic dust causes macrophage activation and the release of a number of cytokines that induce pulmonary inflammation. Growth factors, including platelet-derived growth factor and insulin-like growth factor, are also released and stimulate fibroblasts to proliferate and produce collagen. This production results in progressive interstitial lung fibrosis that characterizes the pneumoconioses.

A 32-year-old man undergoes evaluation for episodes of paroxysmal supraventricular tachycardia. An electrophysiology study reveals focal atrial tachycardia that originates in the crista terminalis of the right atrium. Radiofrequency ablation is planned to destroy the ectopic focus and treat the arrhythmia. Which of the following nerves is in close proximity to the treatment site and is at risk for injury during the ablation?

Phrenic nerve - Paroxysmal supraventricular tachycardias (PSVTs) are arrhythmias that occur intermittently and have abrupt onset and offset. They arise from regions of abnormal electrical activity that occur in the atrioventricular node, an accessory pathway, or the atria. Radiofrequency ablation can be used to destroy the inciting cardiomyocytes and cure the arrhythmia. The crista terminalis, located in the right atrium, is a common site of origination for atrial PSVT. However, the right phrenic nerve, which innervates to the right hemidiaphragm, courses along the pericardium that overlies the right atrium and is at risk of injury during procedures that target structures in or near the right atrium (eg, crista terminalis, cavotricuspid isthmus, right-sided pulmonary veins). Such injury is typically recognized by elevation of the right hemidiaphragm on chest x-ray.

Researchers are developing an enzyme-linked immunosorbent assay test for diagnosing rheumatoid arthritis. The test is designed to detect the presence of serum antibodies against citrullinated proteins. Two test populations with a differing prevalence of rheumatoid arthritis are selected. The researchers plan to assess the test's performance in the 2 populations by comparing a number of diagnostic test parameters. Which of the following performance measures is most likely to be different between the 2 test populations?

Positive predictive value - Predictive values are performance measures of diagnostic tests that are dependent on the prevalence of disease in a population of interest. The positive predictive value (PPV) is the probability that someone who tests positive actually has the disease. It is calculated by dividing the number of true-positive results by the total number of positive results (ie, TP / [TP + FP]). The number of true-positive results depends on the sensitivity, and the number of false-positive results depends on the specificity; the relative proportion of each is determined by the prevalence of disease in the population. Populations with a lower disease prevalence have fewer true-positive results and higher numbers of false positives, so the PPV decreases. As disease prevalence increases, the number of true positives also increases, while the number of false positives decreases, resulting in a higher PPV. Similarly, the negative predictive value increases as the disease prevalence decreases.

A 6-year-old boy is evaluated in the office for difficulty hearing. The patient has no ear pain, discharge, or upper respiratory symptoms. Initial testing suggests that he has bilateral sensorineural hearing loss. He has no motor deficits or cerebellar signs. His paternal uncle died suddenly at age 12. Examination of the ears, nose, and throat is normal. ECG shows normal sinus rhythm with a prolonged QT interval (520 msec). Echocardiogram shows normal left and right ventricular function with no significant valvular disease. A genetic defect affecting which of the following is most likely present in this patient?

Potassium channels -This patient's clinical presentation is suggestive of Jervell and Lange-Nielsen syndrome, an autosomal recessive disorder characterized by profound bilateral sensorineural hearing loss and congenital long QT syndrome, which predisposes individuals to syncope and sudden cardiac death. This syndrome occurs secondary to mutations in genes (eg, KCNQ1, KCNE1) that encode the alpha and beta subunits of voltage-gated potassium channels. These subunits contribute to the slow-acting component of the outward-rectifying potassium current, which is responsible for ventricular repolarization during phase 3 of the cardiac action potential. Mutations in the potassium channel lead to a decrease in potassium current with prolongation of action potential duration and the QT interval. QT interval prolongation predisposes to the development of life-threatening ventricular arrhythmias, such as torsades de pointes and ventricular fibrillation.

A 40-year-old woman is brought to the emergency department due to difficulty breathing and muscle weakness. She was one of several people who developed symptoms in a movie theater. Temperature is 36.7 C (98.1 F), blood pressure is 112/62 mm Hg, pulse is 51/min, and respirations are 24/min. On physical examination, the patient is diaphoretic. The pupils are pinpoint and unreactive, and significant tearing is noted. Diffuse rhonchi and wheezing are present in the lungs bilaterally. Muscle strength is diminished throughout, and fasciculations are noted in the extremities. First-line therapy is administered, but the patient remains weak. Treatment with which of the following is most likely to improve this patient's current condition?

Pralidoxime - This patient with bradycardia, miosis, diaphoresis, excessive secretions (eg, bronchorrhea, tearing), and weakness with fasciculations has signs of cholinergic toxicity. Most cases of cholinergic toxicity are due to organophosphate pesticides. However, the occurrence in multiple patients in a city setting suggests intentional organophosphate exposure, possibly due to a chemical weapon (eg, sarin, soman). Organophosphates inhibit acetylcholinesterase in the muscarinic and nicotinic cholinergic synapses, leading to decreased acetylcholine degradation and overstimulation of the corresponding receptors. In addition to widespread increased visceral smooth muscle tone and glandular secretions due to muscarinic hyperactivity (mnemonic: DUMBELS), nicotinic hyperactivity causes muscle weakness and paralysis that can lead to rapid respiratory depression and death. Initial management of organophosphate toxicity includes atropine, a competitive inhibitor of acetylcholine at the muscarinic receptor, which relieves muscarinic hyperstimulation. However, atropine does not have activity at the nicotinic receptors and cannot treat neuromuscular dysfunction. Therefore, pralidoxime, a cholinesterase-reactivating agent that works at both nicotinic and muscarinic sites, should be administered to any patient with neuromuscular dysfunction (eg, weakness, fasciculations). It should be given only after atropine because pralidoxime can cause transient acetylcholinesterase inhibition, which can momentarily worsen symptoms.

A 29-year-old woman, gravida 2 para 1, at 32 weeks gestation comes to the emergency department due to heavy vaginal bleeding. The bleeding started an hour ago. It was light initially and associated with only mild abdominal pain, but both the bleeding and the pain have increased, and now the pain is constant and severe. The patient has no chronic medical conditions. Her prior pregnancy was a term vaginal delivery complicated by preeclampsia with severe features. Blood pressure is 156/98 mm Hg, and pulse is 112/min. The uterus is firm and tender. Pelvic examination reveals heavy bleeding from the cervical os. Which of the following is the most likely cause of this patient's current presentation?

Premature separation of the placenta and myometrium - This patient in the third trimester with painful vaginal bleeding likely has abruptio placentae, the premature separation of the placenta from the myometrium prior to fetal delivery. Placental abruption occurs when maternal vessels rupture at the uteroplacental interface (ie, decidua basalis); the resultant bleeding causes placental separation from the uterine myometrium. Blood can also accumulate between the placenta and uterine wall and cause an acute increase in intrauterine pressure, resulting in severe abdominal pain, uterine irritability (ie, high-frequency contractions), and a tender, firm uterus. In this patient, abruptio placentae is likely a complication of preeclampsia, a hypertensive disorder of pregnancy that causes widespread endothelial cell damage. Endothelial cell dysfunction results in dysregulated vascular tone (eg, hypertension) and increased vessel fragility. Preeclampsia also impairs early spiral artery development needed to supply blood to the fetus and placenta. As a result, patients with preeclampsia have abnormally high-resistance spiral arteries that produce low placental perfusion, ischemia, and possible placental infarction, all of which increase the risk of abruptio placentae.

A 4-day-old boy born to a 23-year-old woman is brought to the office for evaluation of poor feeding and vomiting. The pregnancy was uneventful and the mother had a normal delivery. Family history is noncontributory. The patient's temperature is 37.2 C (99 F), blood pressure is 60/30 mm Hg, pulse is 110/min, and respirations are 56/min. Physical examination reveals a lethargic newborn with exaggerated deep tendon reflexes and clonus. Further investigation reveals that the patient has an inherited condition that results in impaired transport of ornithine from the cytosol to the mitochondria. Nutritional restriction of which of the following substances can improve this patient's condition?

Proteins - Impaired transport of ornithine into the mitochondria can be caused by ornithine translocase deficiency, which results in a defect in the hepatic urea cycle. The urea cycle converts ammonia, which is generated from the catabolism of amino acids, into urea for excretion in urine. Urea cycle defects cause ammonia to accumulate in the blood, resulting in progressive lethargy, vomiting, seizures, and cerebral edema (may cause hyperreflexia and abnormal posturing when severe) in infancy and early childhood; milder defects caused by partial enzyme deficiencies may not manifest until adulthood. Effective treatment of urea cycle disorders requires balancing dietary protein intake and metabolic requirements. Protein restriction is the main treatment for urea cycle disorders, such that the body receives the essential amino acids needed for growth and development but not in excess such that excessive ammonia is formed.

A 50-year-old man comes to the office for evaluation of abdominal fullness and mild right flank pain. He also reports a weight loss of 4.5 kg (10 lb) over the past 2 months. The patient has no other medical issues and works in a local industrial chemical manufacturing facility. Examination shows a soft abdomen. Ultrasound reveals a mass in the right kidney. A subsequent abdominal CT scan confirms the presence of a large right renal mass with evidence of necrosis. The patient undergoes a right total nephrectomy. The specimen is shown. The patient's lesion most likely originated from which of the following portions of the kidney?

Proximal renal tubules - This patient with a history of chemical exposure and a necrotic kidney mass likely has renal cell carcinoma (RCC), the most common renal malignancy. RCC originates in the renal cortex and occurs most commonly in patients age 60-70. Risk factors include smoking, obesity, hypertension and toxin exposure (eg, heavy metal, petroleum by-products, asbestos). RCC is classified into subtypes based on cellular origin; clear cell carcinoma (CCC) is the most common type and accounts for up to 85% of RCCs. CCC originates from the epithelium of the proximal renal tubules. Gross pathology typically demonstrates a sphere-like mass composed of golden-yellow tissue (due to high lipid content) with areas of focal necrosis and hemorrhage. It often invades the renal vein and may extend into the inferior vena cava. On microscopy, CCC appears as cuboidal or polygonal cells with abundant clear cytoplasm.

A 6-year-old girl is brought to the emergency department after falling from a chair onto her outstretched right arm. Physical examination shows swelling of the right elbow and tenderness to palpation over the distal humerus. Radial pulses are full and symmetric bilaterally. The patient is unable to cooperate with neurologic examination of the right hand due to significant pain. A peripheral intravenous line is placed, and analgesics are administered for pain control. X-rays show a supracondylar humeral fracture with anterolateral displacement of the proximal fracture fragment. Which of the following structures is most likely to be injured in this patient?

Radial nerve - Supracondylar humeral fractures are common pediatric elbow fractures that characteristically occur after hyperextension injuries (eg, falling onto an outstretched arm). These fractures carry a significant risk of neurovascular compromise, especially if the fragments are displaced. The brachial artery, median nerve, and radial nerve all run anterior to the elbow, with the brachial artery and median nerve over the medial epicondyle and the radial nerve over the lateral epicondyle. The median nerve and brachial artery are frequently injured in supracondylar fractures due to anteromedial displacement of the proximal fracture fragment (Choice C). However, this patient has intact radial pulses bilaterally (ie, sparing of the brachial artery) and anterolateral displacement of the proximal fracture fragment, making radial nerve injury more likely. Injury to the radial nerve at the elbow can result in wrist drop due to denervation of hand/finger extensor muscles, and sensory loss over the posterior forearm/dorsolateral hand.

A 67-year-old woman comes to the office due to difficulty concentrating. She reports repeatedly reading the same page of material due to an inability to focus. She also describes memory problems, stating that it "takes a while to remember another person's name." The patient's medical history is significant for hypertension and type 2 diabetes mellitus. These are well controlled through diet, exercise, and oral medications. She lives alone and describes her mood as "good." Blood pressure is 127/67 mm Hg and pulse is 65/min. The physician decides to administer a brief cognitive test to assess her difficulty in concentrating. Which of the following elements of cognitive testing would best assess for impaired attention and concentration in this patient?

Reciting months of the year backwards - There are several quick clinical tests to assess attention and concentration in situations where they may be impaired (eg, dementia, delirium, head injury, depression). These include counting down from 100 by intervals of 3 or 7, reciting the months of the year in reverse order, or spelling "world" backwards. Asking a patient to recite the months of the year backwards is often helpful if the patient has a low educational level or is hesitant to perform a task involving math or spelling.

A 40-year-old man comes to the office due to recurrent epigastric abdominal pain, nausea, and watery diarrhea for the past several months. The patient has had similar burning pain previously and was treated for peptic ulcer disease. He has been taking over-the-counter antacids but with no significant symptom relief. The patient does not use tobacco, alcohol, or illicit drugs. Vital signs are within normal limits. Abdominal examination shows mild epigastric tenderness and increased bowel sounds. Testing for stool occult blood is positive, and laboratory studies show microcytic hypochromic anemia. Serum gastrin level is markedly elevated. Upper gastrointestinal endoscopy reveals prominent gastric folds and multiple ulcers in the duodenum and proximal jejunum. Given these findings, this patient should be assessed for a personal or family history of which of the following conditions?

Recurrent nephrolithiasis - This patient's recurrent peptic ulcer disease, associated with chronic blood loss (iron deficiency) anemia and an elevated gastrin level, is likely due to a gastrin-secreting pancreatic tumor (Zollinger-Ellison syndrome). Although most gastrinomas are sporadic, 20%-30% occur in association with multiple endocrine neoplasia type 1 (MEN1). This autosomal dominant disorder is most often due to mutations of the MEN1 gene (menin protein) and is characterized by parathyroid adenomas, pituitary tumors, and pancreatic endocrine tumors (the 3 Ps). Patients with Zollinger-Ellison syndrome or other pancreatic neuroendocrine tumors should be assessed for symptoms or family history of other MEN1 manifestations. Potential associated conditions include primary hyperparathyroidism (eg, hypercalcemia, constipation, kidney stones) and pituitary adenomas (eg, headaches, visual field defects).

A 25-year-old man comes to the office due to shortness of breath. He states that even moderate exertion forces him to stop to catch his breath. The patient has no family history of sudden cardiac death. He does not use tobacco or illicit drugs. On examination, the patient has a faint systolic murmur at the left sternal border in the supine position that increases to 3/6 in intensity with Valsalva maneuver. Echocardiography shows interventricular septal hypertrophy and increased left ventricular mass. The patient is started on high-dose beta blocker therapy. On follow-up 2 weeks later, he reports significant improvement in his shortness of breath. Physical examination reveals no murmurs supine or with Valsalva. Which of the following best accounts for improvement of this patient's symptoms?

Reduction in left ventricular contractility - This patient's echocardiogram shows increased left ventricular (LV) mass with predominant interventricular septal hypertrophy, which are classic findings for hypertrophic cardiomyopathy (HCM). Many patients with HCM have LV outflow tract obstruction that can lead to symptoms of poor cardiac output (eg, dyspnea, syncope). The outflow tract obstruction worsens with decreased LV blood volume (as can occur during exercise due to decreased LV filling time and increased LV contractility); on physical examination, this effect is evidenced by a systolic murmur that increases in intensity with maneuvers that decrease LV blood volume (eg, Valsalva strain phase, abrupt standing). In patients with HCM and symptomatic LV outflow tract obstruction, beta blockers (eg, metoprolol, nadolol) improve symptoms mostly via an increase in LV volume that reduces LV outflow tract obstruction. This occurs via the following 2 mechanisms: Beta blockers reduce heart rate, which lengthens diastolic filling time to increase LV end-diastolic volume; beta blockers reduce LV contractility, which decreases the amount of blood ejected during ventricular systole. Nondihydropyridine calcium channel blockers (eg, verapamil) have similar effects and can be used in patients with a contraindication to beta blockers or as additional therapy for persistent symptoms.

Public health officials involved in developing nutritional guidelines commission a study to determine how dietary eating patterns influence total body iron stores in children age 5-17. As part of the study, researchers want to assess how 2 independent variables, red meat consumption and egg consumption (both reported in units of ounces/week), affect serum ferritin concentrations while adjusting for age and gender. Which of the following statistical techniques is most helpful for determining the association between the study variables?

Regression analysis -Variables are broadly classified as qualitative (ie, categorical) or quantitative (eg, continuous) based on their scale of measurement. Qualitative variables (eg, type of treatment, blood type) represent categories or groups whereas quantitative variables (eg, temperature, glucose levels) represent numerical values. The scale of measurement of the dependent (eg, outcome) and independent (eg, exposures, risk factors) variables in a study determines the correct statistical test. Regression analysis is used to describe the association between 1 or more independent variables (eg, exposures, risk factors), which may be quantitative or qualitative, and 1 quantitative dependent variable (ie, a quantitative outcome [eg, laboratory values]) while adjusting for other variables of interest.

A 42-year-old man comes to the office due to a 2-week history of reddish-brown urine, fatigue, joint pain, and a lower extremity rash. The patient has no medical history, and family history is unremarkable. He has had multiple sexual partners and only uses protection intermittently. Temperature is 37.7 C (99.9 F), blood pressure is 146/94 mm Hg, and pulse is 75/min. Palpable purpura and pitting edema are present in bilateral lower extremities. Laboratory results are as follows: Serum chemistry Creatinine 3.2 mg/dL Liver function studies Aspartate aminotransferase (SGOT) 78 U/L Alanine aminotransferase (SGPT) 96 U/L Immunologic studies C3 low C4 low Urinalysis Protein 3+ White blood cells 2-3/hpf Red blood cells many/hpf Casts several RBC casts Which of the following immunologic mechanisms is most likely responsible for this patient's renal injury?

Renal deposition of circulating immune complexes formed by exogenous antigens - This patient's acute kidney injury (eg, elevated creatinine), proteinuria, hypertension, and glomerular hematuria (eg, red cell casts) are indicative of glomerulonephritis (GN). GN is commonly a manifestation of small vessel vasculitis (SVV), which usually has associated systemic symptoms (eg, fever, fatigue). GN due to SVV is immune mediated by 3 possible mechanisms: Antibodies targeting antigens in the glomerulus (intrinsic or deposited) with immune complex (IC) formation in the kidney Preformed circulating ICs targeting endogenous or exogenous antigens with subsequent kidney deposition Antibodies targeting neutrophil antigens causing neutrophil activation without significant IC deposition. Several features of this patient's presentation suggest GN/SVV due to hepatitis C virus (HCV). Evidence for HCV infection includes multiple sexual partners and elevated aminotransferases. HCV-infected patients often produce cryoglobulins against exogenous HCV antigens that form circulating ICs, which activate complement, causing hypocomplementemia. These circulating ICs can deposit in small vessels ("cryoglobulinemic vasculitis") affecting the kidney (eg, membranoproliferative GN), skin (eg, palpable purpura), and joints (eg, arthralgias).

A 12-year-old boy is brought to the office with fever, chills, and a rash that appeared this morning. Two days earlier, the child started complaining of a sore throat. His temperature is 38.3 C (101 F). Examination shows a diffuse erythematous rash on his chest and abdomen that blanches with pressure, along with numerous 1- to 2-mm papules. The throat is erythematous with gray-white tonsillar exudates and the tongue is bright red. Which of the following complications is most likely to develop in this patient?

Rheumatic fever -This patient most likely has scarlet fever, which is caused by strains of Group A streptococcus that produce pyrogenic exotoxins. Scarlet fever is most often associated with streptococcal pharyngitis, which begins acutely after an incubation period of 1-5 days. Initial symptoms include fever, malaise, abdominal pain, and sore throat. The pharynx is typically erythematous, swollen, and possibly covered with gray-white exudates. In addition, the tongue can have inflamed red papillae with an appearance similar to that of a red strawberry. After 1-2 days, a rash appears on the neck, armpits, and groin that subsequently generalizes to the rest of the body. The rash begins as scarlet spots or blotches, giving a boiled lobster appearance. As the rash progresses and becomes more widespread, it begins to resemble a sunburn with goose pimples ("sandpaper-like" rash). The cheeks commonly appear flushed, giving the area around the mouth a pale appearance in comparison (circumoral pallor). Toward the end of the first week, desquamation begins and is most pronounced in the armpits, groin, and tips of the fingers and toes. As with any streptococcal upper respiratory infection, scarlet fever can predispose to acute rheumatic fever and glomerulonephritis. Treatment with penicillin V can prevent the development of rheumatic fever, although its role in preventing glomerulonephritis is uncertain.

A 32-year-old man comes to the hospital due to 1 week of progressive shortness of breath and cough. He also reports sharp right-sided chest pain that is worsened by deep inspiration. Examination reveals decreased tactile fremitus, dullness on percussion, and decreased breath sounds over the right lower lung. Chest x-ray shows consolidation of the right lower lobe and a right-sided effusion. A thoracentesis is performed, during which the needle is inserted along the upper border of the 10th rib at the right midaxillary line. Which of the following structures is most at risk of being injured as a result of the intervention?

Right hepatic lobe - The visceral pleura covers the surface of the lung, whereas the parietal pleura lines the inner surface of the chest wall and diaphragm. The parietal pleura generally extends approximately 2 ribs below the inferior margin of the lungs. The potential space at the reflection of the costal pleura onto diaphragmatic pleura is called the costodiaphragmatic recess, where pleural fluid accumulates when the body is erect. Thoracentesis is typically performed between the 6th and 8th ribs along the midclavicular line, the 8th and 10th ribs along the midaxillary line, and the 10th and 12th ribs along the paravertebral line. This allows pleural fluid to be drained without risking lung injury (which can occur at higher insertion sites). However, insertion of a needle below the 9th rib still risks penetrating abdominal structures. In this case, thoracentesis is being performed along the upper border of the 10th rib at the right midaxillary line, putting the patient at risk of liver injury if the needle is inserted too deep.

A 28-year-old woman at 20 weeks gestation comes to the office for her first prenatal appointment. An ultrasound is performed, during which the fetus is found to have a congenital defect and no cardiac activity. Due to the pregnancy loss, the patient undergoes an induction of labor. After delivery, gross pathology of the fetus is shown in the exhibit. These findings are most likely due to an impairment in which of the following embryologic processes?

Rostral neuropore closure - The gross pathology of this fetus is consistent with anencephaly, an open neural tube defect (NTD). Open NTDs are caused by a defect in primary neurulation. This process normally begins with formation of the neural plate, the lateral ends of which elevate to become neural folds during the third week of fetal development. These folds form the neural tube by fusing in the midline, starting in the cervical region and extending rostrally and caudally to the anterior and posterior neuropores, respectively. Failure of either neuropore to close results in an open NTD, and failure of anterior (rostral) neuropore closure specifically leads to anencephaly, a fatal condition in which the skull and meninges fail to form. The fetal brain is then exposed to amniotic fluid, resulting in degeneration of the developing forebrain, as seen here. Maternal serum alpha fetoprotein is markedly elevated due to direct leakage from the open neural tube into the amniotic fluid. Prenatal ultrasound shows absence of the brain and skull superior to the orbits.

A 36-year-old woman comes to the office due to arthralgias and nodules on her legs. She has no prior medical conditions and takes no medications. On physical examination, there is moderate hepatomegaly. A representative skin rash is shown. The lesions are tender and present predominantly on the anterior surface of the lower extremities. Chest x-ray demonstrates enlarged hilar lymph nodes, and laboratory testing reveals an elevated ACE level. Which of the following would most likely be found on liver biopsy in this patient?

Scattered granulomas - This patient has tender erythematous nodules on the anterior lower extremities, consistent with erythema nodosum, a nonspecific, delayed hypersensitivity reaction that can be seen in a variety of infections or inflammatory disorders. However, in combination with hilar adenopathy and elevated ACE levels (formed by activated macrophages), this presentation is highly suggestive of sarcoidosis. Sarcoidosis is a systemic inflammatory disorder of unknown etiology characterized pathologically by noncaseating granulomas. Granulomas can occur in any organ but commonly involve the lymph nodes (eg, bilateral hilar adenopathy), lungs (interstitial lung disease), joints (eg, arthralgias), eyes (uveitis), or skin. Most patients with sarcoidosis also develop liver involvement, typically in the form of asymptomatic hepatomegaly with mild liver function test abnormalities (alkaline phosphatase > aminotransferases). Biopsy will show scattered noncaseating granulomas, predominantly around the portal veins.

A 20-year-old man is brought to the emergency department due to fever, headache, and neck pain for the last day. He has no significant medical history. Temperature is 38.7 C (101.7 F), blood pressure is 120/72 mm Hg, pulse is 112/min, and respirations are 26/min. There is neck stiffness and a petechial rash on the trunk. Cerebrospinal fluid (CSF) analysis reveals the following: Glucose 30 mg/dL Protein 180 mg/dL Leukocytes 1,500/mm3 Neutrophils70% CSF gram stain shows gram-negative diplococci. In the emergency department, the patient's hemodynamic status deteriorates rapidly. Blood pressure drops to 80/50 mm Hg, and the venous access sites are oozing blood. Which of the following findings is most likely to be seen on this patient's peripheral blood smear?

Schistocytes - This patient's clinical findings (fever, headache, neck stiffness, rash) and cerebrospinal fluid analysis (elevated protein, low glucose, leukocytosis, gram-negative diplococci) are indicative of meningococcal meningitis; he subsequently developed septic shock (severe hypotension) while in the emergency department. His bleeding from venous puncture sites is highly suggestive of disseminated intravascular coagulation (DIC), a consumptive coagulopathy associated with sepsis, malignancy, trauma, and obstetric complications. In DIC due to gram-negative sepsis, the coagulation cascade is activated by bacterial endotoxins, leading to widespread fibrin deposition and the consumption of coagulation factors and platelets, with eventual bleeding. Deposition of fibrin strands in small vessels can cause shearing of circulating erythrocytes, resulting in schistocytes (fragmented erythrocytes) on peripheral smear. Laboratory values in DIC typically show decreased platelet count and fibrinogen level and prolonged PT and PTT (indicating a consumption of coagulation factors).

A 29-year-old woman comes to the office due to increased hair loss. She delivered a healthy baby boy 8 months ago and has not felt the same since. The patient has fatigue and decreased libido, and her menstrual cycles have not returned. She has no prior medical problems and no drug allergies. The patient takes a multivitamin and does not use tobacco, alcohol, or illicit drugs. There is no significant family history. Blood pressure is 110/70 mm Hg and pulse is 58/min. Physical examination shows diffuse alopecia and thickened, brittle nails. The thyroid is nontender. Laboratory results show decreased serum TSH and free thyroxine (T4) levels. Which of the following is the most likely diagnosis in this patient?

Secondary hypothyroidism - This patient has hypothyroidism manifesting as fatigue, hair loss, and brittle nails with a low serum thyroxine (T4) level. The low TSH confirms that this is due to hypothalamic-pituitary dysfunction (ie, central/secondary hypothyroidism) rather than a primary thyroid disorder, which would cause a compensatory rise in TSH. Central hypothyroidism is much less common than primary hypothyroidism. However, when it occurs, other hypothalamic-pituitary systems may be affected and patients may develop deficiencies of other pituitary hormones (eg, low gonadotropin levels leading to amenorrhea). The causes of central hypothyroidism are the same as those of hypopituitarism in general. Sheehan syndrome is ischemic necrosis of the pituitary gland and is typically caused by systemic hypotension during delivery. During pregnancy, the pituitary gland enlarges due to estrogen-induced hyperplasia of lactotrophs, but the blood supply does not increase proportionally. Subsequent peripartum hemorrhage with hypotension can cause underperfusion of the pituitary gland with subsequent ischemic injury.

A 30-year-old woman is evaluated for a 3-month history of progressive fatigue, decreased appetite, and 10-lb (4.5-kg) weight loss. The patient has type 1 diabetes mellitus and has noticed decreased insulin requirements over this time. She has no other medical conditions and does not use tobacco, alcohol, or illicit drugs. Physical examination shows a generalized increase in pigmentation of the skin, especially involving the palmar creases. Measurement of serum cortisol before and after administration of exogenous adrenocorticotropic hormone (ACTH) shows no difference in the levels. Which of the following changes in serum laboratory values is most likely present in this patient?

Sodum: ↓ Potassium ↑ Chloride ↑ Bicarbonate ↓ This patient's fatigue and weight loss are likely due to primary adrenal insufficiency (Addison disease). Type 1 diabetes mellitus is caused by autoimmune destruction of pancreatic beta cells and is associated with an increased risk of other autoimmune endocrinopathies. Autoimmune adrenalitis is the most common cause of primary adrenal insufficiency and results from autoantibodies against all 3 zones of the adrenal cortex. Because the main defect is in the adrenal gland, exogenous ACTH administration does not increase cortisol levels. Decreased negative feedback of cortisol on the pituitary gland increases release of ACTH and melanocyte-stimulating hormone, resulting in hyperpigmentation. Patients with diabetes often have reduced insulin requirements due to decreased appetite, weight loss, and increased insulin sensitivity (glucocorticoids decrease insulin sensitivity). Hypoaldosteronism in primary adrenal insufficiency leads to decreased Na+ reabsorption and decreased urinary excretion of K+ and H+. Decreased H+ excretion causes a nonanion gap metabolic acidosis with low plasma HCO3-. This leads to compensatory Cl- retention to maintain electrical neutrality of the extracellular fluid. The hypotension induced by lack of aldosterone stimulates vasopressin release, increasing free water reabsorption and further exacerbating hyponatremia. As a result, patients with primary adrenal insufficiency present with hyponatremia, hyperkalemia, hyperchloremia, and nonanion gap metabolic acidosis.

A 50-year-old man has recurrent episodes of paroxysmal atrial fibrillation accompanied by uncomfortable palpitations and chest pressure. Echocardiogram reveals normal biventricular function and no significant valvular disease. Coronary angiography reveals no obstructive coronary artery disease. The patient is started on medication to reduce his symptoms. Two weeks later, he is seen in the emergency department for lightheadedness, weakness, and presyncope. ECG reveals sinus bradycardia at a rate of 53/min with QTc prolongation (508 msec). Telemetry monitoring reveals a short episode of self-resolved torsades de pointes. Which of the following medications was most likely used to treat this patient's palpitations?

Sotalol - Paroxysmal atrial fibrillation is usually treated with rate or rhythm control strategies. Rate control uses AV nodal-blocking drugs (beta blockers, calcium channel blockers) to prevent rapid ventricular response, whereas rhythm control attempts to maintain sinus rhythm with antiarrhythmic drugs (eg, sotalol, flecainide, amiodarone). The QT interval represents both ventricular depolarization and repolarization (total duration of ventricular action potential). However, ventricular depolarization normally contributes only a small amount of time to the interval, and in practice, the QT interval is primarily used as a measure of ventricular repolarization. Class III antiarrhythmics (eg, sotalol, amiodarone, dofetilide) predominantly block potassium channels and inhibit outward repolarizing currents during phase 3 of the cardiomyocyte action potential. This leads to increased action potential duration and QT interval prolongation, which predisposes to the development of polymorphic ventricular tachycardia (torsades de pointes [TdP]). Amiodarone has a lesser proarrhythmic effect than other class III drugs; therefore, TdP is less frequently observed with amiodarone than with sotalol.

A 38-year-old man comes to the office due to a 2-month history of an enlarging, localized swelling on the left foot. Tying his shoelaces too tight leads to localized pain and numbness in the third and fourth toes. Examination shows a firm, nontender, well-circumscribed lesion on the dorsum of the foot as shown in the exhibit. The lesion transilluminates when a penlight is applied to it. Without treatment, which of the following is most likely to occur in this patient?

Spontaneous resolution - This patient has a ganglion cyst, an outpouching of connective tissue arising from tendon sheaths, joint structures (eg, joint capsule), or bursae. Clear mucinous or gelatinous fluid accumulates in the sac, giving rise to a round, well-circumscribed, firm cyst that transilluminates on penlight examination. Ganglion cysts are most common on the dorsal and volar wrists, but dorsal foot lesions are also common. Most ganglion cysts resolve spontaneously and do not require specific treatment. Cysts that raise cosmetic concerns or cause pain or paresthesia (due to nerve compression) can be treated with needle aspiration if desired. However, recurrence is common, and patients may eventually require surgical excision.

A 23-year-old woman comes to the hospital due to fever, chills, dyspnea, and cough for the past several days. She also has sharp right-sided chest pain exacerbated by breathing. Her temperature is 39 C (102 F), blood pressure is 115/70 mm Hg, and pulse is 108/min. On examination, the patient has several needle track marks on both arms. There is a 3/6 holosystolic murmur heard best at the lower sternal border that increases on inspiration. Chest imaging reveals scattered bilateral peripheral lung opacities. HIV testing is negative. This patient's blood cultures are most likely to grow which of the following organisms?

Staphylococcus aureus - Staphylococcus aureus can cause right-sided endocarditis in intravenous drug users (IVDUs). In patients other than IVDUs, S aureus disseminates from a primary disease process (eg, abscess, infected central line) and moves by hematogenous spread to the endocardium. S aureus settles on the valve leaflets due to blood flow turbulence at these sites. S aureus can cause perforations in the heart valves, rupture the chordae tendineae, and send septic emboli to the lung (with right heart endocarditis) or to the brain and systemic circulation (with left heart endocarditis). This patient with fever is an IVDU with a holosystolic murmur that increases with inspiration (likely tricuspid regurgitation) and multiple lung opacities (likely septic emboli). She likely has S aureus endocarditis.

A 57-year-old man comes to the office accompanied by his wife for follow-up of Parkinson disease. The patient reports no change in motor symptoms, and the disease has been well controlled with anti-Parkinson medication. However, his wife is very concerned about a change in behavior, reporting that he recently started gambling and has lost a significant amount of their savings. The patient drives several hours multiple times a week to gamble at a casino in a neighboring state. His only other medical condition is major depressive disorder for which he takes sertraline. The patient does not use tobacco, alcohol, or illicit drugs. Vital signs are within normal limits. Physical examination shows a mild resting hand tremor and minimal rigidity. Which of the following is most likely responsible for this patient's recent change in behavior?

Stimulation of central dopamine receptors - Parkinson disease (PD) is caused by loss of dopaminergic neurons in the basal ganglia, resulting in symptoms of bradykinesia, tremor, and rigidity. Dopaminergic medications used to treat PD have been associated with the abrupt onset of impulse control disorders, including pathological gambling, compulsive buying, and compulsive sexual behavior. Dopamine agonists (eg, pramipexole) as a class are most frequently associated with this adverse effect. These drugs act by binding to central dopamine receptors and stimulating dopamine activity in the ventral striatum (includes the nucleus accumbens, which is involved in motivation and reward pathways) and substantia nigra. Patients on dopamine agonist therapy should be monitored for dysfunctional impulse control. Gradual taper of the offending agent results in improvement in most cases.

A 30-year-old man comes to the emergency department due to severe abdominal pain. The patient first developed the pain yesterday, and it has become increasingly worse. Over the last few hours, he has also developed bilious emesis. The patient has had no previous surgeries. Temperature is 37.8 C (100 F) and pulse is 110/min. Physical examination shows diffuse abdominal tenderness with guarding during palpation. Abdominal imaging reveals bowel wall thickening within a blind pouch connected to the ileum. A laparotomy is performed. During the procedure, a fibrous band is seen attaching the end of the pouch to the umbilicus. The walls of this pouch are most likely composed of which of the following?

Submucosa, mucosa, and muscular layers - This patient's outpouching from the ileum with a fibrous band connected to the umbilicus is characteristic of a Meckel diverticulum. This congenital anomaly is due to incomplete obliteration of the vitelline (omphalomesenteric) duct in utero; the duct normally serves as a communication between the yolk sac and the primitive midgut in early gestation. Meckel diverticulum is typically located in the small bowel 2 feet from the ileocecal valve and, if symptomatic, often presents by age 2 (ie, rule of 2s); however, it can cause complications at any age. Meckel diverticulum is a true diverticulum, consisting of all 3 layers of the intestinal wall: mucosa, submucosa, and muscularis. It often contains heterotopic gastric mucosa, which secretes hydrochloric acid and leads to the classic presentation of painless lower gastrointestinal bleeding (hematochezia) due to the ulceration of adjacent intestine. Patients may also develop diverticular inflammation with signs of acute intestinal obstruction (eg, abdominal pain, guarding, bilious emesis), as seen in this patient. Other complications of a Meckel diverticulum can include intussusception, volvulus, and bowel perforation.

A 25-year-old man with an insignificant medical history was playing soccer when he suddenly collapsed. Despite all attempts to save his life, he died. Autopsy is performed. Gross examination of the heart reveals a weight of 580 g (normal: <350). The cut surface of the heart is shown. If this patient had a preparticipation sports screening, cardiac auscultation would have likely revealed a murmur that increases in intensity after which of the following?

Sudden standing - This patient's heart shows asymmetric ventricular septal hypertrophy, a finding characteristic of hypertrophic cardiomyopathy (HCM). Dynamic left ventricular outflow tract (LVOT) obstruction is common in HCM and creates a harsh crescendo-decrescendo, systolic ejection murmur that is best heard along the lower left sternal border. Maneuvers or conditions that decrease LV preload (eg, abrupt standing, Valsalva strain phase, dehydration) or decrease LV afterload (eg, administration of an arterial vasodilator) decrease LV blood volume to worsen LVOT obstruction and increase the intensity of the HCM murmur. In contrast, maneuvers or conditions that increase LV preload (eg, passive leg raising, squatting) or increase LV afterload (eg, infusion of an arterial vasoconstrictor like the alpha-1 agonist phenylephrine, sustained handgrip) increase LV blood volume to alleviate LVOT obstruction and decrease the intensity of the HCM murmur.

A 64-year-old man comes to the emergency department due to chest discomfort and shortness of breath. Medical history includes hypertension, type 2 diabetes mellitus, and severe osteoarthritis of the left knee. The patient is an active smoker with a 20-pack-year smoking history. He undergoes dobutamine infusion with simultaneous cardiac imaging. The following is seen: Before infusion: normal left ventricular contractility, ejection fraction 60% During infusion: decreased apical contractility, ejection fraction 45% Five minutes after infusion: normal left ventricular contractility, ejection fraction 60% Which of the following best explains the observed findings?

Supply-demand mismatch due to fixed coronary stenosis - Stable angina results from fixed coronary artery stenosis in the setting of atherosclerotic coronary artery disease (CAD). Such a stenosis limits blood supply to the downstream myocardium, which, during periods of increased myocardial oxygen demand (eg, exercise), can cause a mismatch of oxygen supply and demand that manifests as anginal symptoms (eg, chest pain, shortness of breath). Pharmacologic stress testing can be useful in diagnosing atherosclerotic CAD. Dobutamine, a beta-1 agonist, increases heart rate and contractility to mimic the increase in myocardial oxygen demand that occurs with exercise. Myocardium that is unable to obtain sufficient blood flow to meet the increased oxygen demand typically demonstrates a transient decrease in contractility (ie, wall motion defect), often leading to reduced ejection fraction. This patient with a transient wall motion defect localized to the cardiac apex most likely has a fixed atherosclerotic lesion that limits blood flow to that area of myocardium.

A 26-year-old man with a history of excessive daytime sleepiness due to narcolepsy enrolls in a clinical trial evaluating a new stimulant medication. The patient was previously treated with modafinil, but his symptoms did not improve. It is explained to the patient that the new drug works by promoting the release of dopamine and norepinephrine from nerve terminals in the central nervous system. Adverse effects of the drug are due primarily to excess release of norepinephrine from peripheral autonomic nerve terminals. Which of the following autonomic efferents would be most affected by the use of this drug?

Sympathetic output to the detrusor muscle - The parasympathetic and sympathetic subdivisions of the autonomic nervous system regulate most visceral functions of the body via 2-neuron signal transmission. In the parasympathetic system, preganglionic neurons arise from cranial nerve nuclei (eg, CN III, VII, IX, X) and from the sacral spinal cord and release acetylcholine, which binds to postganglionic nicotinic receptors found within the parasympathetic ganglia in or near the walls of target organs. Postganglionic neurons also release acetylcholine, activating muscarinic receptors within the target organs (eg, bronchi, heart). In the sympathetic system, preganglionic neurons arise from the thoracolumbar spinal cord and release acetylcholine, which binds to postganglionic nicotinic receptors in the sympathetic chain and prevertebral ganglia (Choice C). Postganglionic neurons then release norepinephrine, activating alpha/beta receptors within target organs (including the bladder); however, there are 2 notable exceptions to this scheme. The chromaffin cells of the adrenal medulla release norepinephrine and epinephrine directly into the circulation after stimulation by acetylcholine released from sympathetic preganglionic neurons. The preganglionic and postganglionic sympathetic neurons that supply eccrine sweat glands are both cholinergic.

A 38-year-old woman suddenly collapses at home after experiencing severe chest pain and dies before reaching a hospital. Postmortem examination reveals an occluding thrombus overlying a ruptured atherosclerotic plaque on the left anterior descending artery. The patient is also found to have thickened mitral valve leaflets with multiple small vegetations on both valvular surfaces and fibrinoid necrosis of arterioles. Other findings include glomerular capillary basement membrane thickening with wire-loop changes. This patient was most likely suffering from which of the following conditions?

Systemic lupus erythematosus - Cardiovascular manifestations of SLE include accelerated atherosclerosis with increased risk of myocardial infarction. Small arteries and arterioles can also show evidence of vasculitis with fibrinoid necrosis in any tissue. Other cardiac features include pericarditis and less commonly verrucous (Libman-Sacks) endocarditis, characterized by thickened valve leaflets (typically mitral and/or aortic) with multiple small vegetations on both surfaces. The vegetations are composed of sterile platelet thrombi intertwined with strands of fibrin, immune complexes, and mononuclear cells. These vegetations are easily dislodged and can result in systemic embolization. Renal involvement in SLE most often manifests as diffuse proliferative glomerulonephritis, which is characterized by proliferative and necrotizing lesions with crescent formation during active disease. Light microscopy also classically shows diffuse thickening of the glomerular capillary walls with "wire-loop" structures due to subendothelial immune complex deposition.

A 19-year-old man comes to the office due to eye pain and blurry vision in both eyes for the last several days. He sustained an open globe injury to the right eye 3 months ago after being struck during an altercation and was treated with surgical repair and prophylactic antibiotics. The left eye was unaffected. At the patient's last follow-up appointment, visual acuity in the right eye had improved from 20/400 to 20/80. He is otherwise healthy. Temperature is 37.1 C (98.8 F). Examination is unremarkable apart from bilateral conjunctival injection and decreased visual acuity in both eyes. Analysis of vitreous samples from both eyes demonstrates multinucleated giant cells. Which of the following mechanisms is most likely causing this patient's current manifestations?

T-cell response to previously sequestered antigens - This patient with a traumatic injury to the right eye has developed granulomatous inflammation of both the injured and noninjured eye, a condition known as sympathetic ophthalmia. This occurs when there is a robust T-cell response to previously sequestered antigens in the eye, an area that displays immune privilege. Self-antigens located in sites with immune privilege can be recognized by T cells that escape negative selection in the thymus. Therefore, if these antigens are released into the lymphatic system through trauma, T cells may recognize these antigens as foreign and mount a response that can occur in both the injured eye and the contralateral eye.

A 26-year-old man with depressed mental status is brought to the emergency department by police. Due to fear of arrest, he swallowed a handful of pills as the officers approached him. On examination, the patient responds to painful stimuli but is somnolent. Pupils are <2 mm. Respirations are 6/min. Lungs are clear to auscultation. Which of the following additional findings are most likely present in this patient?

Temperature ↓ Blood pressure ↓ Tidal volume ↓ This patient has the classic triad of decreased respiratory rate, constricted pupils (ie, miosis), and depressed mental status, consistent with acute opioid intoxication. Opioids are derived from the opium poppy plant and act on multiple receptor types (eg, mu, kappa, delta) located throughout the brain, spinal cord, and gastrointestinal tract. In an overdose, overstimulation of these receptors causes numerous adverse effects, the most lethal one being respiratory depression. Respiratory depression is the best predictor of opioid intoxication. In addition to the classic triad, other findings include hypothermia, hypotension, and decreased tidal volume. Decreased bowel sounds and constipation are also common findings.

A 28-year-old man comes to the office to discuss hair loss. He has hair loss primarily around the temples, which began insidiously 3 years ago. The patient is concerned because several of his family members became almost completely bald by their late 30s. He otherwise feels healthy and has normal muscle strength, libido, and erectile function. The patient is married, has 2 children, and works as a charter airplane pilot. Physical examination shows moderate thinning of hair at the anterior scalp, temporal region, and vertex. An oral medication is prescribed to treat the hair loss. After the initiation of therapy, which of the following changes would most likely occur in this patient?

Testosterone: increased Dihydrotestosterone: decreased Estradiol: increased This patient has androgenetic alopecia (often called male pattern hair loss), which presents with progressive hair loss at the temples, frontal hairline, and vertex. Androgenetic alopecia is characterized by a shortened anagen hair growth phase in the affected portions of the scalp, leading to shorter, thinner hair shafts (follicular miniaturization) with increased apoptosis at the dermal papillae. As the name implies, androgenetic alopecia is driven by both inherited (polygenic inheritance) and hormonal factors. Dihydrotestosterone (DHT) appears to be the primary androgen responsible for hair loss. Although total androgen levels are similar in affected and unaffected men, patients with androgenetic alopecia have a greater effect of DHT due to increased local conversion of testosterone to DHT and increased concentrations of androgen receptors in affected areas of the scalp. Androgenetic alopecia can be treated with 5-alpha reductase inhibitors (eg, finasteride), which decrease conversion of testosterone to DHT in the tissues. However, by preventing conversion to DHT, they increase the level of testosterone, which is then available for conversion to estradiol by aromatase. This can lead to mild feminizing effects (eg, gynecomastia) in some patients. Other treatments for androgenetic alopecia include topical minoxidil, a vasodilator that appears to increase the vascularity of the dermal papillae.

A 33-year-old man comes to the emergency department due to fever, chills, cough, and shortness of breath over the last week. He smokes a pack of cigarettes a day, consumes 2 or 3 alcoholic beverages daily, and uses intravenous heroin 2 or 3 times per week. He is febrile and tachycardic. Blood cultures grow Staphylococcus aureus. Chest x-ray shows bilateral nodular lesions with areas of cavitation. The valve most likely affected in this patient can be best evaluated by auscultation at which of the following sites?

The left 5th intercostal space along the sternum - The cavitary pulmonary nodules on this patient's chest x-ray are consistent with septic emboli, which are a common complication of right-sided infective endocarditis (IE). The tricuspid valve is frequently affected in intravenous drug users; bacterial adherence may be encouraged by the inadvertent injection of particulate matter that damages the tricuspid valve endothelium. Staphylococcus aureus is the most common organism responsible for IE in intravenous drug users. A murmur of IE typically results from regurgitation of blood due to impaired closure of the affected valve. Although tricuspid valve endocarditis does not always cause a murmur, an early- or holo-systolic murmur of tricuspid regurgitation is sometimes appreciated. This murmur is best auscultated in the 4th or 5th intercostal space at the left lower sternal border (the tricuspid auscultation area), and it increases in intensity with maneuvers that increase right ventricular blood volume (eg, inspiration, supine leg raise).

A 4-day-old infant is brought to the emergency department due to abnormal movements. The patient has had intermittent episodes of tonic posturing over the past 3 hours as well as vomiting, poor feeding, and irritability for the past 2 days. The mother also reports that the infant's diapers smell like caramelizing sugar. There are no known medical conditions in the family, but the mother's sister died "sometime in the first year" of life from unknown causes. On examination, the patient is lethargic and demonstrates intermittent posturing and increased generalized muscle tone. Laboratory evaluation of plasma and urine samples confirms the suspected diagnosis. This patient is most likely deficient in an enzyme that requires which of the following substances?

Thiamine - This infant has symptoms typical of maple syrup urine disease (MSUD), an autosomal recessive disorder. MSUD is characterized by mutations in branched-chain alpha-keto acid dehydrogenase complex (BCKDC), an enzyme necessary for breaking down the branched-chain amino acids (BCAAs) leucine, isoleucine, and valine. Treatment for MSUD involves restricting BCAAs in the diet. BCKDC (in addition to pyruvate dehydrogenase and alpha-ketoglutarate dehydrogenase) requires 5 cofactors: thiamine, lipoate, and coenzymes A, FAD, and NAD. Because MSUD has a spectrum of phenotypes, patients who have only a mild reduction in BCKDC activity may improve with high-dose thiamine treatment; however, most patients still require lifelong dietary restriction.

A 68-year-old man is evaluated in the hospital due to a pleural effusion. He has a history of base of tongue squamous cell carcinoma with involvement of multiple left anterior cervical lymph nodes. Two days ago, the patient underwent surgery to remove the tumor and the anterior cervical lymph nodes. Today, he has increasing shortness of breath, and chest x-ray reveals a large left pleural effusion. Which of the following was most likely injured during this patient's surgical procedure?

Thoaracic duct - This patient with a pleural effusion 2 days after cervical lymph node removal most likely has a chylothorax due to intraoperative injury of the thoracic duct. The thoracic duct is the largest lymphatic vessel. It originates at about the level of T12 in the abdomen where the lumbar and intestinal lymph trunks converge. It travels superiorly through the mediastinum (posterior to the esophagus) and drains into the junction between the left subclavian and jugular veins in the lower neck. It is most commonly injured during thoracic procedures (eg, esophagectomy) but can be injured in neck procedures as well (eg, cervical lymph node removal). Injury can result in leakage of lymph into either the neck or the thorax because a transected duct may retract into the chest. The thoracic duct collects lymph from most of the body, including the abdominal viscera, the viscera of the left hemithorax, all tissue inferior to the umbilicus, and all left-sided tissues superior to the umbilicus (the right head, arm, and thorax drain into the much smaller right lymphatic duct).

Erythroblasts isolated from a bone marrow biopsy sample of a patient with neonatal jaundice are incubated in a medium containing radiolabeled glucose. The cells are unable to generate NADPH from glucose metabolism but are able to convert fructose-6-phosphate to ribose-5-phosphate, which is required for nucleic acid synthesis. Which of the following enzymes is essential for the latter conversion?

Transketolase - The pentose phosphate pathway (HMP shunt) generates NADPH for use in reductive reactions and ribose-5-phosphate, a precursor for the synthesis of nucleotides. The pathway consists of 2 types of reactions, oxidative (irreversible) and nonoxidative (reversible), both of which can function independently depending on cellular requirements. Activity of the nonoxidative reactions is governed by the cellular demand for ribose-5-phosphate. When ribose-5-phosphate is produced in excess, transketolase and transaldolase can produce the glycolytic intermediates glyceraldehyde-3-phosphate and fructose-6-phosphate for ATP generation. When ribose-5-phosphate demand exceeds the production capabilities of the oxidative pathway, the nonoxidative pathway functions in reverse and transketolase and transaldolase catalyze the conversion of fructose-6-phosphate and glycerldehyde-3-phosphate to ribose-5-phosphate.

A 36-year-old woman is evaluated for several months of tingling and numbness of the right hand and fingers that has progressed to dull, aching pain. The symptoms initially were intermittent but have become persistent, with the pain worsening overnight. The patient works as an office secretary and has had difficulty typing and holding the phone due to hand weakness. On examination, she has weakness of thumb opposition and atrophy of the thenar eminence. The patient attempted treatment with conservative measures but had no significant improvement and has now been referred for surgery. During surgery, incision of which of the following structures is most likely to improve this patient's symptoms?

Transverse carpal ligament - This patient has right hand pain and numbness, thenar atrophy, and weakness of thumb opposition characteristic of carpal tunnel syndrome (CTS). The carpal tunnel is an anatomic space in the wrist between the carpal bones and the transverse carpal ligament (flexor retinaculum). The median nerve and 9 tendons (eg, flexor digitorum superficialis and profundus, flexor pollicis longus) pass through the carpal tunnel to enter the hand. CTS occurs when increased pressure within the carpal tunnel (eg, compression, inflammation) causes median nerve damage. Often, CTS can be managed successfully with nonsurgical treatments (eg, wrist splint, glucocorticoid injection). However, patients with significant impairment and evidence of median nerve injury on electromyography are often treated with surgical decompression. A longitudinal incision through the transverse carpal ligament decreases the pressure on the median nerve, improving patients' symptoms.

A 32-year-old woman comes to the emergency department due to 4 weeks of worsening cough, shortness of breath, and low-grade fevers. She also has had an unintentional weight loss of 4.5 kg (10 lb) over the past 3 months. Temperature is 37.1 C (98.8 F), blood pressure is 120/70 mm Hg, pulse is 102/min, and respirations are 22/min. Pulse oximetry shows 86% on ambient air. Lung auscultation reveals bilateral crackles. There are no heart murmurs, and jugular venous pressure is normal. Chest x-ray reveals diffuse interstitial infiltrates. The patient undergoes bronchoscopy; methenamine silver staining of the bronchoalveolar lavage specimen is shown. Which of the following is the most appropriate pharmacotherapy for this patient's current condition?

Trimethoprim-sulfamethoxazole - Pneumocystis pneumonia is an AIDS-defining illness that is often seen in patients who were previously unaware of HIV infection. The underlying pathogen is an atypical fungus called Pneumocystis jirovecii. Healthy patients rapidly clear the inhaled organisms, but those with impaired cell-mediated immunity are at risk for a noninvasive infection that fills the alveoli and leads to slowly worsening pulmonary symptoms (eg, dry cough, shortness of breath), hypoxia, and a diffuse, bilateral interstitial infiltrate. Fever and weight loss are also usually present. P jirovecii cannot be cultured; therefore, definitive diagnosis requires identification of the organism in respiratory secretions. Methenamine silver stain is frequently used to identify the cell wall of the pathogen, which often appears as a crescent, a crushed ping-pong ball, or a circular ring around a clear center. First-line treatment with trimethoprim-sulfamethoxazole is usually curative.

A 32-year-old woman, gravida 1 para 0, at 35 weeks gestation comes to the emergency department due to absent fetal movement for the past 24 hours. The patient has had no prenatal care. Fetal ultrasonography confirms an intrauterine fetal demise. Autopsy examination of the fetus reveals incomplete separation of the cerebral hemispheres with a single ventricle, as shown. Which of the following is most closely associated with this fetal condition?

Trisomy 13 - Fetal brain development begins at week 3, when the ectodermal cells thicken and invaginate to form the neural tube, which differentiates into the primitive forebrain (prosencephalon), midbrain (mesencephalon), and hindbrain (rhombencephalon). The prosencephalon gives rise to the diencephalon, which develops into the thalamus and third ventricle, and telencephalon, which develops into the cerebral hemispheres and lateral ventricles. This fetus has an incomplete separation of the cerebral hemispheres with a single lateral ventricle consistent with holoprosencephaly, a condition in which the forebrain fails to completely divide into right and left hemispheres. Disease severity and midline facial malformations (eg, facial clefts, cyclopia) are dependent upon the degree of hemisphere division; mild forms can be asymptomatic whereas severe cases can be lethal (as in this case). Holoprosencephaly may be due to a variety of factors (eg, sonic hedgehog gene mutations); the most common associated chromosomal abnormality is trisomy 13 (Patau syndrome). Other midline defects of trisomy 13 include microcephaly, microphthalmia, cleft lip/palate, and omphalocele. Cardiac abnormalities are also common, and cutis aplasia (scalp defect) is characteristic. The majority of affected fetuses die in utero.

An 18-year-old man, who was recently started on risperidone therapy for schizophrenia, comes to the office due to involuntary deviation of his head. On examination, the patient's head is tilted toward the right side and the chin is rotated to the left. There is palpable tightening of the right sternocleidomastoid muscle. It is determined that the patient is experiencing a medication-induced dystonic reaction due to motor neuron hyperactivity. The persistent myocyte stimulation causes a substance to be released from the sarcoplasmic reticulum. This substance most likely binds to which of the following proteins to cause this patient's symptoms?

Troponin - The sarcoplasmic reticulum (SR) is a modified endoplasmic reticulum within skeletal muscle cells. The SR forms a network of tubules with terminal cisterns that are in close contact with the T tubules (cytoplasmic membrane invaginations), allowing the membrane depolarization signal to reach the SR. The Ca2+-ATPase pump in the SR membrane actively sequesters calcium to keep intracellular concentrations low. Cell membrane depolarization causes calcium release from the SR into the cytoplasm via the SR ryanodine receptor. Actin filaments, troponin complex (troponins C, T, and I), and tropomyosin form the thin filaments of muscle fibers. In the resting state, tropomyosin covers the myosin binding sites on the actin filaments. On release from the SR, calcium binds to troponin C on the thin filaments. This induces a conformation change in the troponin complex, causing it to displace tropomyosin and expose the myosin binding sites on the actin filaments. Myosin makes up the thick filaments of skeletal muscle. ATP is bound by the myosin head and cleaved to form ADP and inorganic phosphate ion, which are retained at the myosin head. When the myosin head binds to an actin filament, a conformational change causes the myosin to pull the actin filament, leading to muscle contraction and ADP release. A new ATP molecule then is bound to the myosin head, causing release of the actin filament. The cycle then repeats until calcium is displaced from troponin C and the myosin binding sites are again covered.

A new study is conducted to investigate the efficacy of intravenous immunoglobulin versus analgesics on pain alleviation in patients with idiopathic small fiber neuropathy. A total of 100 eligible patients who fulfilled the inclusion/exclusion criteria are assigned to one of two groups based on random selection by a computer. The method of assigning patients to treatment groups in this study is most likely intended to produce which of the following?

Two groups in which participants are similar in underlying characteristics - Randomization refers to the process of randomly (ie, by chance) assigning subjects to experimental groups (eg, treatment vs control group). Randomization prevents selection bias and potentially produces comparable groups by balancing the frequency of confounding variables (eg, sex, age, severity of disease) between experimental groups. Making experimental groups as similar as possible (except for the treatment assigned) cancels the effect that these characteristics may have on the outcome of interest. For example, having groups that are dissimilar (ie, unbalanced) with respect to severity of disease may underestimate or overestimate the effect of a treatment, depending on which group has a greater proportion of severe cases. The best way to account for the effects of these variables is to randomly assign subjects to experimental groups. Afterward, the groups are analyzed to ensure that they are statistically similar in underlying variables, with the results often reported in a baseline characteristics table (eg, weight, sex, age). Therefore, randomization increases the likelihood that any difference observed between groups is due exclusively to the treatment and not to other characteristics.

A 65-year-old woman comes to the emergency department with progressive fatigue and lower-extremity paresthesias. Medical history is otherwise insignificant and the patient takes no medications. Laboratory studies show a mean corpuscular volume of 112 fL. Upper gastrointestinal endoscopy shows atrophic mucosa in the gastric body and fundus and a normal-appearing antrum. Destruction of which of the following gastric layers is most likely responsible for this patient's symptoms?

Upper glandular layer - The combination of lower-extremity paresthesias, macrocytic red blood cells (RBCs), and gastric body and fundal atrophy is highly suggestive of pernicious anemia (PA). PA is an autoimmune disorder caused by the cell-mediated destruction of parietal cells, oxyntic (pale pink) cells found predominantly in the upper glandular layer of the gastric body and fundus. Over time, progressive parietal cell destruction leads to chronic atrophic gastritis (ie, autoimmune gastritis), characterized histologically by a CD4-predominate inflammatory infiltrate, oxyntic gland atrophy, and intestinal metaplasia (ie, replacement of normal epithelia with goblet cells). Parietal cells are responsible for the secretion of hydrochloric acid and intrinsic factor, a glycoprotein that is involved in the absorption of B12 (cobalamin, an essential nutrient). After a critical number of parietal cells are destroyed and hepatic stores of cobalamin are depleted, patients develop B12 deficiency. This results in a megaloblastic anemia, characterized by macro-ovalocytes (elevated mean corpuscular volume, oval-shaped RBCs) and hypersegmented neutrophils. Patients also develop neurologic dysfunction due to demyelination of the dorsal and lateral spinal tracts, resulting in paresthesias and ataxia.

A 3-year-old boy is brought to the office due to abnormal motor development. He was born at 40 weeks gestation and had an unremarkable perinatal course. The boy developed normally during the first year of life. However, for the past 2 years, he has had progressive bilateral leg stiffness and abnormal involuntary movements. His cognitive and motor development is also delayed. There is no significant family history of neurological or muscular disorders. The patient's height, weight, and head circumference are below the 3rd percentile. Examination shows bilateral spastic paresis of his lower extremities and frequent choreoathetoid movements. Comprehensive laboratory testing reveals significantly elevated arginine levels in plasma and cerebrospinal fluid. The deficient enzyme in this patient is normally involved in the production of which of the following?

Urea - This patient has features of arginase deficiency, including progressive development of spastic diplegia, abnormal movements, and growth delay in the setting of elevated arginine levels. Arginase is a urea cycle enzyme that produces urea and ornithine from arginine. Diagnosis is based on elevated arginine levels on plasma amino acid testing. Treatment of arginase deficiency consists of a low-protein diet devoid of arginine. Administration of a synthetic protein made of essential amino acids usually results in a dramatic decrease in plasma arginine concentration and an improvement in neurological abnormalities. Unlike other urea cycle disorders, patients with arginase deficiency have mild or no hyperammonemia.

A 65-year-old woman comes to the clinic for follow up of type 2 diabetes mellitus. Medical history is notable for hypertension, stable coronary artery disease, and heart failure with preserved ejection fraction. Blood pressure is 144/95 mm Hg. Physical examination shows no murmurs, and the lungs are clear on auscultation. Fasting glucose is 122 mg/dL and hemoglobin A1c is 7.8%. The remainder of the laboratory results, including hepatic and renal markers, are normal. The patient is given a prescription for canagliflozin. She should be counseled on which of the following potential complications prior to initiation of this medication?

Urinary infection - Sodium-glucose cotransporter 2 (SGLT2) is a high-capacity transport protein that reabsorbs 90% of filtered glucose in the proximal tubule. SGLT2 inhibitors (eg, canagliflozin, dapagliflozin) are oral antidiabetic agents that decrease renal reabsorption of glucose, leading to urinary glucose loss and decreased blood glucose levels. The decreased reabsorption of sodium and glucose also induces osmotic diuresis, leading to a decrease in blood pressure and a lower rate of hospitalization for heart failure. However, the glycosuria induced by SGLT2 inhibitors creates a more favorable environment for pathogenic bacteria and fungi in the genitourinary tract and perineum. This leads to an increased risk for urinary tract infections (eg, pyelonephritis), genital mycotic infections (eg, vaginitis), and necrotizing fasciitis of the perineum (Fournier gangrene). Other adverse effects include hypovolemia, which can lead to orthostatic hypotension and acute kidney injury.

A 27-year-old man is brought to the emergency department by his wife. She says that he has been "acting crazy" for the last 2 weeks. He has hardly slept for the past 7 days and instead has worked on miscellaneous projects around the house. The patient spent several thousand dollars on new power tools to accomplish these tasks. When questioned, his speech is rapid and frenzied. He feels "spectacular" and is creating an "architectural masterpiece." The patient has had 2 previous depressive episodes. Which of the following medications is the most appropriate agent for long-term management of this patient?

Valproate - This patient's euphoric mood, decreased need for sleep, hyperactivity, grandiosity, and pressured speech lasting more than a week are characteristic of an acute manic episode of bipolar disorder. Bipolar disorder is a highly recurrent illness requiring maintenance treatment with mood-stabilizing medications to decrease the risk of recurrent mood episodes. Preferred medications for bipolar maintenance treatment include lithium, the anticonvulsants valproate and lamotrigine, and the second-generation antipsychotic quetiapine.

A 34-year-old man is brought to the emergency department by his wife after she found him running around the yard in his underwear shouting, "I am going to change the world." She reports that he has a lot of energy despite hardly sleeping for the past 7 days, and that he recently quit his job to write a 500-page novel that he says "explains everything about the universe." The patient's history is significant for epilepsy with generalized seizures since childhood and a psychiatric hospitalization a year ago for a major depressive episode. Family history is significant for alcoholism and hypertension in his father and schizophrenia in his grandmother. Which of the following medications would be most helpful for long-term treatment of this patient's psychiatric condition as well as his seizure disorder?

Valproate - This patient's history of major depression along with his current manic symptoms (ie, decreased need for sleep, grandiosity, goal-directed activity) indicate a diagnosis of bipolar disorder. Valproate (valproic acid) is used as an anticonvulsant and mood stabilizer. It blocks voltage-gated sodium channels and enhances the availability of GABA. As a mood stabilizer, it is used in acute mania and maintenance treatment of bipolar disorder. As an anticonvulsant, it effectively treats absence, myoclonic, and generalized tonic-clonic seizures. It is the drug of choice for myoclonic seizures and a second-line agent for absence seizures (for which ethosuximide is preferred). Other anticonvulsants used as mood stabilizers in bipolar disorder include carbamazepine and lamotrigine. Carbamazepine is a CYP450 inducer that can cause lower levels of concomitant medications. Lamotrigine is associated with skin rash, which rarely can progress to life-threatening mucocutaneous reactions (eg, Stevens-Johnson syndrome).

A 20-year-old woman comes to the physician because of multiple burns on her hands. She says that she gets burned quite often when she cooks, particularly while operating the oven. She also has burned herself when picking up her morning cup of coffee and when testing the water temperature in the shower. The patient is concerned because she cannot feel when objects are "really hot" and often does not notice that she has burns until she looks at her hands. She has no significant past medical history, no recent illnesses, and no sick contacts. Her surgical history is significant only for removal of impacted wisdom teeth 1 year ago. The patient has never smoked, drinks alcohol occasionally, and does not use illicit drugs. Her family history is significant for multiple sclerosis in her mother. Examination shows diminished pinprick and temperature sensation across the upper back, shoulders, and arms bilaterally. Light touch, position, and vibration sense are preserved. Examination of the lower extremities is unremarkable. Damage to which of the following spinal cord areas is most likely responsible for this patient's symptoms?

Ventral white commissure - This patient most likely has syringomyelia, which classically presents with disproportionate loss of pain and temperature sensation (dissociated anesthesia) involving the arms and hands. In syringomyelia, a cerebrospinal fluid-filled cavity called a syrinx usually forms in the cervical region of the spinal cord. These cavities can enlarge over time and destroy adjacent portions of the spinal cord. The areas most commonly damaged are the ventral white commissure and ventral horns. The ventral white commissure is located just anterior to the gray commissure. It is the area of decussation for the fibers of the lateral spinothalamic tract, which transmits pain and temperature sensation from peripheral receptors to the somatosensory cortex. Destruction of the ventral white commissure leads to loss of pain and temperature sensation bilaterally over the affected dermatomes (starting 1 to 2 levels below the lesion, as the axons briefly ascend in the zone of Lissauer before decussating). Touch, vibration, and position senses are preserved as the dorsal columns remain intact.

A 34-year-old man is evaluated in the clinic due to persistent buzzing in his right ear for the past 6 months. The patient also has decreased hearing on the right side. He has had no fever, ear pain, runny nose, sore throat, or cough. The patient does not use cotton swabs and has not been exposed to loud noise. He tried using over-the-counter oral pseudoephedrine with no improvement in his symptoms. On physical examination, there is no sinus tenderness, and mucous membranes are moist without erythema or exudate. Otoscopic examination reveals normal-appearing external auditory canals bilaterally with pinkish-grey tympanic membranes. MRI of the brain reveals an intracranial tumor. This patient's lesion has most likely arisen from which of the following structures?

Vestibulocochlear nerve - This patient's clinical presentation is suggestive of acoustic neuroma, a Schwann cell-derived tumor that typically arises from the vestibular portion of the vestibulocochlear nerve (CN VIII). The vestibulocochlear nerve exits the brainstem from the pontomedullary junction, lateral to the facial nerve, and enters the cerebellopontine angle (between the cerebellum and lateral pons). Patients with acoustic neuroma usually have ipsilateral sensorineural hearing loss and tinnitus (ear ringing/buzzing) due to impairment of the cochlear portion of the nerve. Damage to the vestibular component may also result in vertigo, dysequilibrium, and nystagmus. Most cases present unilaterally, but bilateral acoustic neuromas are frequently seen in patients with neurofibromatosis type 2.

An 18-year-old man comes to the office for a follow-up visit. He was seen in the emergency department 3 months ago because of panic attacks related to cocaine use. The patient subsequently stopped using cocaine and has had no recurrent anxiety symptoms. He graduated from high school, started a new job, and has been considering applying to community college. At today's visit, the patient reports that he used cocaine at a friend's house this past weekend. He says, "I really messed up. I feel like a failure." The patient has no history of illness. He is in no physical discomfort. Which of the following is the most appropriate statement by the physician at this time?

You've been making real progress in quitting. What can we learn from this temporary setback? - This patient says he feels "like a failure" because he used cocaine again following a 3-month period of abstinence. Focusing on the negative consequences of cocaine use (which the patient is already aware of) may be perceived as shaming and judgmental. The best approach is to help the patient feel understood by acknowledging his feelings of disappointment but reframing his relapse as a temporary setback he can surmount. Principles of motivational interviewing can be used to help this patient get back on track and strengthen his belief that he can keep making progress. Motivational interviewing is a nonjudgmental, collaborative, patient-centered approach that enhances the patient's sense of self-confidence and self-efficacy. By emphasizing the patient's past success in maintaining abstinence for a period of time, the physician supports the patient's belief in his ability to learn from this setback and make changes. Open-ended discussion of what contributed to using cocaine again from the patient's perspective can then be used to guide the patient to make a specific plan to achieve and implement change.

A 44-year-old man comes to the emergency department due to sudden-onset shortness of breath that started a few hours ago. Last week, he sustained a crush injury after a motorcycle drove over his left foot, and he has been immobilized since then due to pain. The patient drinks alcohol occasionally and does not use tobacco or illicit drugs. Temperature is 36.1 C (97 F), blood pressure is 118/76 mm Hg, pulse is 102/min, and respirations are 28/min. BMI is 37 kg/m2. On physical examination, heart rhythm is regular with normal S1 and S2. The lungs are clear to auscultation bilaterally. The dorsum of the left foot has resolving ecchymoses and swelling compared to when he was evaluated last week. The right calf is swollen and tender. Which of the following arterial blood gas values are most likely to be present in this patient?

pH 7.57 PaO2 (mmHg) 65 PaCO2 (mmHg) 26 Serum HCO3- (mEq/L) 23 Sudden-onset dyspnea in the setting of recent calf swelling is strongly suggestive of pulmonary embolism (PE). Dyspnea is the most common symptom of PE and risk factors include obesity, smoking, and prolonged immobilization. The embolus typically arises from a deep venous thrombus in the lower extremities (causes calf swelling) and lodges itself within the arterial blood supply of the lungs. PE occludes or significantly reduces blood flow to a portion of the pulmonary parenchyma, resulting in increased dead space ventilation and redistribution of pulmonary blood flow away from the affected segments. This leads to increased ventilation/perfusion (V/Q) mismatch in the remainder of the lungs with consequent hypoxemia. Stimulation of central respiratory drive occurs in response to dyspnea and inflammatory mediators released by ischemic pulmonary tissue. Hyperventilation does not significantly improve arterial oxygenation because hemoglobin is nearly fully saturated with O2 in areas of normal V/Q ratio, allowing little capacity for high V/Q regions to increase blood O2 content much further. In contrast, CO2 removal is more directly dependent on ventilation, and high V/Q regions have large capacity to exhale additional CO2. Therefore, the hyperventilation leads to hypocapnia (low PaCO2) and respiratory alkalosis. The serum HCO3- is near normal in the acute setting, but metabolic compensation with renal bicarbonate loss takes place over the next 72 hours.


Set pelajaran terkait

CH 7 Innovation and Entrepreneurship

View Set

Social Psychology: Chapter 6- Attitudes

View Set

UNIT 3 EXAM Intracranial Regulation + Infection Exemplars: Seizures and Meningitis

View Set

Spanish 1 Semester 2 Culture review

View Set